[obm-l] RE: [obm-l] dúvida

2004-07-03 Por tôpico Rogério Moraes de Carvalho
Olá Leandro,

O esboço de uma figura facilita bastante o entendimento da resolução
desta questão. Eu vou descrever o esboço que eu fiz para que você possa
entender a minha resolução.


INÍCIO DE UMA RESOLUÇÃO POSSÍVEL:

Sejam C1 e C2 os centros das circunferências, tais que R1 = 5 m e R2 = 3 m.
A distância entre os centros C1C2 = 18 m  R1 + R2 = 8 m, portanto a
informação de que as circunferências são exteriores nem era necessária.

OBSERVAÇÕES:

O enunciado do problema não está preciso, pois ele afirma o seguinte:
Calcule de quanto se deve prolongar a distância dos centros para que se
encontre a TANGENTE COMUM EXTERNA.. Existem 4 retas tangentes comuns às
duas circunferências, sendo que duas delas se cruzam num ponto interno do
segmento C1C2 e as duas outras se encontram no prolongamento do segmento
C1C2 no sentido de C1 para C2. Sendo assim, o problema não poderia usar o
artigo definido a para se referir à tangente, pois ela não é única. Mas,
como o problema fala de um prolongamento, pode-se reduzir a apenas 2
tangentes possíveis. O termo TANGENTE EXTERNA também não está preciso. O
problema quer sugerir que seja o ponto de encontro das retas tangentes
comuns que se encontram na reta suporte do segmento que une os centros das
circunferências, mas que não seja interno a este segmento. Ou seja, um ponto
pertencente à reta suporte do segmento C1C2 e EXTERNO a ele.

Segue uma sugestão de modificação no enunciado para que ele fique mais
preciso: Calcule de quanto se deve prolongar a distância dos centros das
circunferências para que se encontre o ponto de intersecção com as retas
tangentes comuns num ponto externo ao segmento que tem extremidades nos
centros..

CONTINUAÇÃO DA RESOLUÇÃO POSSÍVEL:

Considere T1 e T2 os pontos de tangência com as circunferências de centros
C1 e C2, respectivamente, pertencentes a uma das duas retas tangentes comuns
que não se encontram num ponto interno do segmento que une os centros das
circunferências. Considere também o ponto P como a intersecção desta reta
tangente com o prolongamento do segmento C1C2 no sentido de C1 para C2 e que
a medida C2P (medida do prolongamento) é dada por x.

Triângulo C1T1P ~ Triângulo C2T2P (Critério AA~)
C1T1P = C2T2P (ângulos retos, uma vez que T1 e T2 são pontos de tangência)
T1PC1 = T2PC2 (ângulo em comum)

Logo, teremos que:
C1P/C2P = C1T1/C2T2
(18 + x)/x = 5/3
18/x + 1 = 5/3
18/x = 2/3
2x = 54
x = 27 m

RESPOSTA: 27 m


Rogério Moraes de Carvalho
Consultor e Instrutor de Tecnologias da Informação
[EMAIL PROTECTED]
-Original Message-
From: [EMAIL PROTECTED] [mailto:[EMAIL PROTECTED] On
Behalf Of leandro-epcar
Sent: sábado, 3 de julho de 2004 11:40
To: obm-l
Subject: [obm-l] dúvida

  Duas circunferências de raios 5 m e 3 m sao 
exteriores .Sabendo que a distancia entre os seus 
centros é de 18 m .calcule de quantos se deve prolongar 
a distancia dos centros para que se encontre a TANGENTE 
COMUM EXTERNA 


resposta : 27 m
===
 o que seria a  tangente comum externa ,o que ele 
quer dizer com isso e como se calcula ?
==
atenciosamente
 leandro
 
__
Acabe com aquelas janelinhas que pulam na sua tela.
AntiPop-up UOL - É grátis!
http://antipopup.uol.com.br/



=
Instruções para entrar na lista, sair da lista e usar a lista em
http://www.mat.puc-rio.br/~nicolau/olimp/obm-l.html
=



=
Instruções para entrar na lista, sair da lista e usar a lista em
http://www.mat.puc-rio.br/~nicolau/olimp/obm-l.html
=


[obm-l] RE: [obm-l] Função quadrática

2004-07-02 Por tôpico Rogério Moraes de Carvalho
From: [EMAIL PROTECTED] [mailto:[EMAIL PROTECTED] On
Behalf Of Daniel Silva Braz
Sent: sexta-feira, 2 de julho de 2004 12:09
To: [EMAIL PROTECTED]
Subject: Re: [obm-l] Função quadrática

Um retângulo tem dimensões x e y, entre x e y vale a
relação 2x + y = 
21. Calcular x e y e a área do retângulo
consequentemente, sabendo que é 
a maior possível.

xy deve ser o maior possivel tal que 2x+y = 21
(assumindo que x e y pertencem aos inteiros positivos
- caso contrario teriamos uma infinidade de solucoes)
isso ocorre para x=5 e y=11 = xy = 55

[]'s
Daniel

==

--- [EMAIL PROTECTED] escreveu:  Olá pessoal da
lista boa noite.
 
 Gostaria de saber se alguém poderia me ajudar na
 resolução de um problema de área de retângulo, pois
 gerou uma dúvida à resposta.
 
 Eis o problema:
 
 Um retângulo tem dimensões x e y, entre x e y vale a
 relação 2x + y = 21. Calcular x e y
 (consequentemente a área do retângulo), sabendo que
 sua área é a maior possível.
 
 Um abraço a todos, Marcelo.
 
 ---
 Inscreva-se na Maratona iBest para concorrer a
 prêmios!
 São 2 casas, 11 Ford EcoSport e 60 computadores.
 Participe já: http://maratona.ibest.com.br

=
 Instruções para entrar na lista, sair da lista e
 usar a lista em
 http://www.mat.puc-rio.br/~nicolau/olimp/obm-l.html

=
  





___
Yahoo! Mail agora com 100MB, anti-spam e antivírus grátis!
http://br.info.mail.yahoo.com/
=
Instruções para entrar na lista, sair da lista e usar a lista em
http://www.mat.puc-rio.br/~nicolau/olimp/obm-l.html
=




=
Instruções para entrar na lista, sair da lista e usar a lista em
http://www.mat.puc-rio.br/~nicolau/olimp/obm-l.html
=


[obm-l] RE: [obm-l] Função quadrática

2004-07-02 Por tôpico Rogério Moraes de Carvalho
Olá Daniel,

Você está enganando, pois não é necessário assumir que x e y são
inteiros positivos. Na realidade, x e y devem ser reais positivos uma vez
que representam as medidas dos lados de um retângulo. Também não está
correto dizer que teríamos uma infinidade de soluções se x e y não forem
inteiros positivos, pois a solução é única. Segue uma resolução possível.

RESOLUÇÃO POSSÍVEL:
2x + y = 21 = y = 21 - 2x (i)
S = x.y (ii) Área do retângulo

Substituindo (i) na (ii):
S = x.(21 - x) = S = -x^2 + 21x (iii)

S em função de x é uma função quadrática com coeficiente de x^2 negativo
(-1), portanto a concavidade da parábola tem a concavidade voltada para
baixo e a função admite um valor máximo. O ponto de máximo da função
(abscissa do vértice) é dado por:
x = -b/(2a) = x = 21/4 = 5,25 (iv)

Substituindo (iv) na (i):
y = 21 - 2.(21/4) = y = 21/2 = 10,5 (v)

Substituindo (iv) e (v) na (ii), teremos:
S = x.y = S = (21/4).(21/2) = S = 441/8 = 55,125

RESPOSTA: x = 21/4 = 5,25, y = 21/2 = 10,5 e S = 441/8 = 55,125


Abraços,

Rogério Moraes de Carvalho
Consultor e Instrutor de Tecnologias da Informação
[EMAIL PROTECTED]

-Original Message-
From: [EMAIL PROTECTED] [mailto:[EMAIL PROTECTED] On
Behalf Of Daniel Silva Braz
Sent: sexta-feira, 2 de julho de 2004 12:09
To: [EMAIL PROTECTED]
Subject: Re: [obm-l] Função quadrática

Um retângulo tem dimensões x e y, entre x e y vale a
relação 2x + y = 
21. Calcular x e y e a área do retângulo
consequentemente, sabendo que é 
a maior possível.

xy deve ser o maior possivel tal que 2x+y = 21
(assumindo que x e y pertencem aos inteiros positivos
- caso contrario teriamos uma infinidade de solucoes)
isso ocorre para x=5 e y=11 = xy = 55

[]'s
Daniel

==

--- [EMAIL PROTECTED] escreveu:  Olá pessoal da
lista boa noite.
 
 Gostaria de saber se alguém poderia me ajudar na
 resolução de um problema de área de retângulo, pois
 gerou uma dúvida à resposta.
 
 Eis o problema:
 
 Um retângulo tem dimensões x e y, entre x e y vale a
 relação 2x + y = 21. Calcular x e y
 (consequentemente a área do retângulo), sabendo que
 sua área é a maior possível.
 
 Um abraço a todos, Marcelo.
 
 ---
 Inscreva-se na Maratona iBest para concorrer a
 prêmios!
 São 2 casas, 11 Ford EcoSport e 60 computadores.
 Participe já: http://maratona.ibest.com.br

=
 Instruções para entrar na lista, sair da lista e
 usar a lista em
 http://www.mat.puc-rio.br/~nicolau/olimp/obm-l.html

=
  





___
Yahoo! Mail agora com 100MB, anti-spam e antivírus grátis!
http://br.info.mail.yahoo.com/
=
Instruções para entrar na lista, sair da lista e usar a lista em
http://www.mat.puc-rio.br/~nicolau/olimp/obm-l.html
=




=
Instruções para entrar na lista, sair da lista e usar a lista em
http://www.mat.puc-rio.br/~nicolau/olimp/obm-l.html
=


[obm-l] RE: [obm-l] Não conseguir

2004-06-11 Por tôpico Rogério Moraes de Carvalho








3) O
valor numérico  é igual a :



A) 1990  B)
1991  C) 1992    D) 1993 E) 1994

RESOLUÇÃO POSSÍVEL:

1990^2 - 1996 = (1990^2
- 4) - 1992 = (1990 + 2)(1990 - 2) - 1992 = 1992.1988 - 1992 = 1992.(1988 - 1)
= 1992.1987

1990^2 + 3980 - 3
= (1990^2 + 2.1990 + 1) - 4 = (1990 + 1)^2 - 4 = (1991 + 2)(1991 - 2) =
1993.1989



Portanto, a
expressão a ser calculada se reduz a:

[(1992.1987)(1993.1989)(1991)]/[(1987)(1989)(1992)(1993)]



Simplficando os
fatores comuns do numerador e do denominador, encontramos:

1991



RESPOSTA:
Alternatica B (1991)





Rogério Moraes de
Carvalho












--
Esta mensagem foi verificada pelo sistema de antivírus e 
 acredita-se estar livre de perigo.

image001.gif

RE: [obm-l] pares ordenados

2004-06-10 Por tôpico Rogério Moraes de Carvalho
1) Quantos sao os pares nao-ordenados de inteiros positivos tais que, em
cada par, a soma do produto dos numeros do par com a soma dos numeros do 
par com o modulo da diferenca dos numeros do par seja igual a 20? 

a)1 b)2 c)3 d)4 e)5 


RESOLUÇÃO POSSÍVEL:

Uma vez que os pares são não ordenados, teremos que (x, y) = (y, x), para
todos x e y inteiros positivos. A fim de eliminar soluções repetidas, como
por exemplo em: (4, 3) e (3, 4), vamos considerar os valores x e y em ordem
decrescente com x e y inteiros positivos, ou seja: x = y  0 (i).

De acordo com o enunciado devemos ter:
xy + (x + y) + |x - y| = 20
Pela condição (i): x = y = x - y = 0 = |x - y| = x - y, logo:
xy + x + y + x - y = 20
xy + 2x = 20
x(y + 2) = 20

Fazendo a = x e b = y + 2, teremos:
a.b = 20, onde x = a e y = b - 2, como x e y positivos (ii)

Partindo de (ii), encontramos as seguintes possibilidades:
a = 1 e b = 20: x = 1 e y = 18 - não satisfaz a condição (i)
a = 2 e b = 10: x = 2 e y = 8 - não satisfaz a condição (i)
a = 4 e b = 5: x = 4 e y = 3 - satisfaz a condição (i)
a = 5 e b = 4: x = 5 e y = 2 - satisfaz a condição (i)
a = 10 e b = 2: x = 10 e y = 0 - não satisfaz a condição (i)
a = 20 e b = 1: x = 20 e y = -1 - não satisfaz a condição (i)

Não há a necessidade de testar os valores tais que a e b são inteiros
negativos, pois neste caso teríamos x = a  0 e y = b - 2  0, o que não
satisfaz a condição (i).

RESPOSTA: Alternativa b (4 e 3, 5 e 2).



Abraços,

Rogério Moraes de Carvalho



=
Instruções para entrar na lista, sair da lista e usar a lista em
http://www.mat.puc-rio.br/~nicolau/olimp/obm-l.html
=


[obm-l] RE: [obm-l] Não conseguir

2004-06-10 Por tôpico Rogério Moraes de Carvalho








4)
O número  é igual a :

A)
371   B) 372   C)  373  D)   374    E) 375




RESOLUÇÃO POSSÍVEL:

Ao fatorar 324 encontramos: 324 = 2^2.3^4
= 4.3^4.

Na expressão a ser calculada temos
fatores do tipo: a^4 + 4.b^4, com b = 3.

Segue uma possível fatoração da expressão
a^4 + 4.b^4:
a^4 + 4.b^4 = (a^2 + 2.b^2)^2 - 2.a^2.(2.b^2) = (a^2 + 2.b^2)^2 - (2ab)^2 = (a^2
+ 2.b^2 - 2ab)(a^2 + 2.b^2 + 2ab) = [(a^2 - 2ab + b^2) + b^2][(a^2 - 2ab + b^2)
- b^2]
Ou seja, teremos que:
a^4 + 4.b^4 = [(a - b)^2 + b^2] [(a + b)^2 + b^2] (i)

Considerando b = 3 na identidade demonstrada
acima, teremos:
a^4 + 324 = [(a - 3)^2 + 3^2] [(a + 3)^2 + 3^2] (ii)

Aplicando (ii) em cada fator da expressão
a ser calculada, obtemos:
[(7^2 + 3^2)(13^2 + 3^2)(19^2 + 3^2)(25^2 + 3^2)(31^2 + 3^2)(37^2 + 3^2)(43^2 +
3^2)(49^2 + 3^2)(55^2 + 3^2)(61^2 + 3^2)]/ [(1^2 + 3^2)(7^2 + 3^2)(13^2 + 3^2)(19^2
+ 3^2)(25^2 + 3^2)(31^2 + 3^2)(37^2 + 3^2)(43^2 + 3^2)(49^2 + 3^2)(55^2 + 3^2)]

Simplificando os fatores comuns do numerador
e do denominador, obtemos:
(61^2 + 3^2)/(1^2 + 3^2) = 3730/10 = 373

RESPOSTA: Alternativa C

Abraços

Rogério Moraes de Carvalho.





--
Esta mensagem foi verificada pelo sistema de antivírus e 
 acredita-se estar livre de perigo.

image001.gif

[obm-l] RE: [obm-l] O Último Teorema de Fermat

2004-06-10 Por tôpico Rogério Moraes de Carvalho
Olá Nicolau,

Você sabe como eu posso conseguir a demonstração final do Teorema de
Fermat feita pelo Wiles com coautoria do Richard Taylor? Existe alguma
versão digital disponível para download na Internet?

Abraços,

Rogério Moraes de Carvalho
-Original Message-
From: [EMAIL PROTECTED] [mailto:[EMAIL PROTECTED] On
Behalf Of Nicolau C. Saldanha
Sent: quinta-feira, 10 de junho de 2004 15:55
To: [EMAIL PROTECTED]
Subject: Re: [obm-l] O Último Teorema de Fermat

On Thu, Jun 10, 2004 at 12:08:57AM -0400, [EMAIL PROTECTED] wrote:
 Pelo que sei Andrew Wiles provou este teorema, mas havia um erro e o
proprio 
 Andrew corrigiu posteriormente. Este matematico entrou ateh para o Guiness

 Book, por isso. Corrijam-me se estiver errado.

Está tudo certo; o segundo artigo tem o Richard Taylor como coautor,
que foi aluno do Wiles e colega meu de doutorado em Princeton.
Apenas talvez seja melhor dizer não que Wiles provou, mas que Wiles
*completou*
a demonstração do teorema. Se você pegar o paper do Wiles e tentar ler não
vai entender muita coisa (a menos que você saiba muito mais teoria dos
números do que eu): o trabalho dele se baseia em um monte de trabalhos
anteriores, inclusive muitos bastante recentes.

[]s, N.

=
Instruções para entrar na lista, sair da lista e usar a lista em
http://www.mat.puc-rio.br/~nicolau/olimp/obm-l.html
=




=
Instruções para entrar na lista, sair da lista e usar a lista em
http://www.mat.puc-rio.br/~nicolau/olimp/obm-l.html
=


RE: [obm-l] Problema 16 OBM - Nivel 3

2004-06-08 Por tôpico Rogério Moraes de Carvalho
Resolva no campo dos reais a equação:
sqr[x + 2.sqr(x - 1)] + sqr[x - 2.sqr(x - 1)] = 2


RESOLUÇÃO POSSÍVEL:

Condição de existência no campo dos reais:
x - 1 = 0 = x = 1

Considerando x = 1, podemos concluir que:
x + 2.sqr(x - 1) = [sqr(x - 1)]^2 + 2.sqr(x - 1) + 1 = [sqr(x - 1) + 1]^2
x - 2.sqr(x - 1) = [sqr(x - 1)]^2 - 2.sqr(x - 1) + 1 = [sqr(x - 1) - 1]^2

Logo, teremos:
sqr{[sqr(x - 1) + 1]^2} + sqr{[sqr(x - 1) - 1]^2} = 2
|sqr(x - 1) + 1| + |sqr(x - 1) - 1| = 2 (i)

x - 1 = 0 = sqr(x - 1) = sqr(0) = sqr(x - 1) + 1 = 1 = |sqr(x - 1) +
1| = sqr(x - 1) + 1 (ii)
A função sqr(X) é estritamente crescente em X, para todo X real não
negativo, logo: sqr(X1) - sqr(X2) = 0 = sqr(X1) = sqr(X2) = X1 = X2 e
sqr(X1) - sqr(X2)  0 = sqr(X1)  sqr(X2) = X1  X2, para todos X1, X2
reais não negativos. Sendo assim, analisando a expressão sqr(x - 1) - 1 =
sqr(x - 1) - sqr(1), concluímos que:
sqr(x - 1) - sqr(1)  0 = x - 1  1 = x  2
sqr(x - 1) - sqr(1) = 0 = x - 1 = 1 = x = 2
Logo:
|sqr(x - 1) - 1| = sqr(x - 1) - 1, para x = 2 e |sqr(x - 1) - 1| = 1 -
sqr(x - 1), para x  2 (iii)


Substituindo (ii) e (iii) em (i), teremos:

Para 1 = x  2:
sqr(x - 1) + 1 + 1 - sqr(x - 1) = 2 = 0.sqr(x - 1) = 0 (Satisfeita para
qualquer x no intervalo considerado, ou seja, 1 = x  2) (iv)

Para x = 2:
sqr(x - 1) + 1 + sqr(x - 1) - 1 = 2 = 2.sqr(x - 1) = 2 = [sqr(x - 1)]^2
= 1^2 (observe que os radicandos são não negativos) = x - 1 = 1 = x = 2
(v)

Unindo as soluções de (iv) e (v): S = [1, 2] = {x real | 1 = x = 2}

Resposta: S = [1, 2] = {x real | 1 = x = 2}


Abraços,


Rogério Moraes de Carvalho
-Original Message-
From: [EMAIL PROTECTED] [mailto:[EMAIL PROTECTED] On
Behalf Of Maurizio
Sent: segunda-feira, 7 de junho de 2004 19:22
To: [EMAIL PROTECTED]
Subject: [obm-l] Problema 16 OBM - Nivel 3

Olá
a questão 16 é assim:

[x+2(x-1)^1/2]1/2+[x-2(x-1)^1/2]1/2=2

Eu obtive essa resoluçãoi mas não está dando certo... Quem escrever 
alguma resolução ou indicar o erro da minha eu agradeço desde já
{[x+2(x-1)^1/2]1/2}^2+2{[x+2(x-1)^1/2]1/2.[x-2(x-1)^1/2]1/2]}+{[x-2(x-1)^1/2
]1/2}^2=4
x+2[x-1]^1/2+2{x^2-2[x-1]^1/2}^1/2+x-2[x-1]^1/2=4
2x+2[x^2-4(x-1)]^1/2=4
x=2
=
Instruções para entrar na lista, sair da lista e usar a lista em
http://www.mat.puc-rio.br/~nicolau/olimp/obm-l.html
=




=
Instruções para entrar na lista, sair da lista e usar a lista em
http://www.mat.puc-rio.br/~nicolau/olimp/obm-l.html
=


[obm-l] RE: [obm-l] Funçao Quadratica

2004-06-07 Por tôpico Rogério Moraes de Carvalho
Olá aryqueirozq,

Segue uma possível resolução para esta questão.


RESOLUÇÃO POSSÍVEL:

Observe que o enunciado do problema não garante que a != 0, portanto não
podemos garantir que se trata de uma função do segundo grau em x (y = f(x)).
Mas, como as alternativas falam em valores de máximo e mínimo, nós
precisamos analisar os possíveis valores de a. Temos que analisar os casos
tais que: a = 0, a  0 (a função admite valor mínimo) e a  0 (a função
admite valor máximo).

Como os pontos (1, 1), (2, m) e (m, 2) pertencem à curva representada pela
equação y = ax^2 + bx + c, teremos:
a.1^2 + b.1 + c = 1 = a + b + c = 1 (E1)
a.2^2 + b.2 + c = m = 4a + 2b + c = m (E2)
a.m^2 + b.m + c = 2 = m^2.a + m.b + c = 2 (E3)

Para encontrar a, b e c em função de m devemos resolver um sistema de 3
equações em relação às 3 incógnitas a, b e c. Porém, para a != 0 a equação
corresponde a uma parábola. 
Segue a resolução pelo método de eliminação de Gauss:
a + b + c = 1 (E1)
3a + b = m - 1 (E4 = E2 - E1)
(m^2 - 1)a + (m - 1)b = 1 (E5 = E3 - E1)

a + b + c = 1 (E1)
3a + b = m - 1 (E4)
[(m^2 - 1) - 3(m - 1)]a = 1 - (m - 1)^2 (E6 = E5 - (m - 1).E4)

a + b + c = 1 (E1)
3a + b = m - 1 (E4)
(m^2 - 3m + 2)a = -m^2 + 2m = (m - 1)(m - 2)a = -m(m - 2) (E6)

O enunciado garante que m != 2, logo podemos dividir ambos os membros da
equação E6 por (m - 2) != 0, obtendo:
(m - 1)a = -m (E7)


PARA m = 1:

(E7) (1 - 1)a = -1 = 0.a = -1 (Impossível no campo dos reais)


PARA m = 0:

(E7) (0 - 1)a = -0 = a = 0
(E4) 3.0 + b = 0 - 1 = b = -1
(E1) 0 + (-1) + c = 1 = c = 2
Neste caso, a função seria do primeiro grau em x: y = 0.x^2 + (-1).x + 2 =
y = -x + 2. A função é representa por uma reta não paralela aos eixos no
sistema de coordenadas cartesianas ortogonais e portanto não admite valor
mínimo nem valor máximo.


PARA m != 0 e m != 1 e m != 2:

(E7) (m - 1)a = -m = a = -m/(m - 1), pois m != 1

ESTUDO DE SINAL DA EXPRESSÃO -m/(m - 1):
   +  0   --2  -
--o-o-  -m
   -  -1   +2  +
---oo- m - 1
   -  0   +1   -2  -
--ooo- a = -m/(m - 1)

CONCLUSÃO:
A função admite valor máximo (a  0) para m  0 ou m  1 e m != 2.
A função admite valor mínimo (a  0) para 0  m  1.


RESPOSTA: Alternativa B

Rogério Moraes de Carvalho
-Original Message-
From: [EMAIL PROTECTED] [mailto:[EMAIL PROTECTED] On
Behalf Of aryqueirozq
Sent: domingo, 6 de junho de 2004 21:13
To: obm-l
Subject: [obm-l] Funçao Quadratica




No sistema  de coordenadas cartesianas ortogonais , a 
curva y=ax2 + bx + c passa pelos pontos (1,1) , ( 2,m) 
e (m, 2) , m é um número real diferente de 2. Sobre 
esta curva podemos afirmar que:

aEla admite um mínimo para todo m tal que ½m
3/2
b)Ela admite um mínimo para todo m tal que 0 m   
   1
c)Ela admite um máximo para todo m tal que – ½m   
1/2
d)Ela admite um máximo para todo m tal que ½m  3/2
e)Ela admite um máximo para todo m tal que0m   
  1

  agreço antecipadamente.
 
__
Acabe com aquelas janelinhas que pulam na sua tela.
AntiPop-up UOL - É grátis!
http://antipopup.uol.com.br/



=
Instruções para entrar na lista, sair da lista e usar a lista em
http://www.mat.puc-rio.br/~nicolau/olimp/obm-l.html
=




=
Instruções para entrar na lista, sair da lista e usar a lista em
http://www.mat.puc-rio.br/~nicolau/olimp/obm-l.html
=


RE: [obm-l] OBM 2004 - Nivel 3

2004-06-06 Por tôpico Rogério Moraes de Carvalho
Olá Vitor,

O cálculo da medida do lado AB é muito simples e direto, bastando
utilizar uma única vez o teorema de Stewart. A fim de tornar a questão um
pouco mais interessante, eu modifiquei o enunciado para pedir as medidas dos
outros três lados do quadrilátero, ou seja, AB, CD e DA e não somente AB.


QUESTÃO COM O ENUNCIADO EXTENDIDO:

As diagonais AC e BD de um quadrilátero ADCB se encontram em E. Se AE = 2,
BE = 5, CE = 10, DE = 4 e BC = 15/2, calcule os outros três lados do
quadrilátero: AB, CD e DA.


OBSERVAÇÃO:

Eu utilizarei o Teorema de Stewart, algumas vezes denominado Teorema de
Apollonius, para resolver esta questão. Segue um enunciado possível para o
Teorema de Stewart.


TEOREMA DE STEWART:

Seja ABC um triângulo qualquer e P um ponto interno do lado BC (AP é uma
ceviana interna relativa ao lado BC), então vale a seguinte relação:

AB^2/(BP.BC) + AC^2/(CP.CB) - AP^2/(PB.PC) = 1

(O Teorema de Stewart pode ser facilmente demonstrado pela aplicação da lei
dos co-senos nos triângulos APB e APC.)



RESOLUÇÃO POSSÍVEL DA QUESTÃO COM O ENUNCIADO EXTENDIDO:

No triângulo BAC, E é um ponto interno do lado AC, então:
BA^2/(AE.AC) + BC^2/(CE.CA) - BE^2/(EA.EC) = 1
BA^2/(2.12) + (15/2)^2/(10.12) - 5^2/(2.10) = 1
BA^2/24 + 15/32 - 5/4 = 1
BA^2/24 = 57/32
BA^2 = (9.19)/4
BA = 3.sqr(19)/2 (RESPOSTA DA QUESTÃO ORIGINAL)

No triângulo ADB, E é um ponto interno do lado DB, então:
AD^2/(DE.DB) + AB^2/(BE.BD) - AE^2/(ED.EB) = 1
AD^2/(4.9) + [3.sqr(19)/2]^2/(5.9) - 2^2/(4.5) = 1
AD^2/36 + 19/20 - 1/5 = 1
AD^2/36 = 1/4
AD^2 = 9
AD = 3

Os triângulos AEB e DEC são semelhantes pelo critério LAL, pois:
AEB = DEC (ângulos opostos pelo vértice)
AE/DE = 2/4 = 1/2
EB/EC = 5/10 = 1/2
Pela propriedade transitiva da igualdade: AE/DE = EB/EC
Da semelhança podemos concluir que:
BA/CD = 1/2 = AE/DE = EB/EC
CD = 2.BA = CD = 2.[3.sqr(19)/2] = CD = 3.sqr(19)

RESPOSTA DA QUESTÃO EXTENDIDA:

AB = 3.sqr(19)/2 (RESPOSTA DA QUESTÃO ORIGINAL), CD = 3.sqr(19) e AD = 3.


Atenciosamente,

Rogério Moraes de Carvalho
-Original Message-
From: [EMAIL PROTECTED] [mailto:[EMAIL PROTECTED] On
Behalf Of Vitor Dias
Sent: domingo, 6 de junho de 2004 01:23
To: [EMAIL PROTECTED]
Subject: [obm-l] OBM 2004 - Nivel 3

Fiz a prova...achei bem legal, e muito bem elaborada! foi um otimo
treinamento,
jah q vou prestar concurso para o CN e para EPCAr...(alem do que sou um
eterno apaixonado pela matematica hehe)
bem, vou deixar uma questao legal de geometria plana, tirada do livro
Challenging
Problems in Geometry, de Alfred S. Posamentier e Charles T. Salkind:

As diagonais AC e BD de um quadrilatero ADCB se encontram em E. Se AE=2,
BE=5, CE=10, DE=4 e BC=15/2, calcule AB.

Divirtam-se!!!


=
Instruções para entrar na lista, sair da lista e usar a lista em
http://www.mat.puc-rio.br/~nicolau/olimp/obm-l.html
=




=
Instruções para entrar na lista, sair da lista e usar a lista em
http://www.mat.puc-rio.br/~nicolau/olimp/obm-l.html
=


RE: [obm-l] o valor de x - continuacao

2004-06-06 Por tôpico Rogério Moraes de Carvalho
 - sqr(17)]/2}

Observe que a primeira igualdade deve ser válida, pois ambos os membros são
positivos, mas a segunda não é válida, pois o primeiro membro é negativo e o
segundo membro é positivo. Porém, se os números forem simétricos (opostos
aditivos) na segunda igualdade, então ao elevarmos ambos os membros ao
quadrado a igualdade se tornará verdadeira. Transformando o radical duplo
sqr{[9 +/- sqr(17)]/2} = sqr[9/2 +/- sqr(17/4)] em radicais simples,
teremos:
A = 9/2 e B = 17/4, logo: C = sqr(A^2 - B) = sqr[(9/2)^2 - 17/4] = sqr(81/4
- 17/4) = sqr(64/4) = sqr(16) = 4.
Logo: sqr{[9 +/- sqr(17)]/2} = sqr[(A + C)/2] +/- sqr[(A - C)/2] = sqr[(9/2
+ 4)/2] + sqr[(9/2 - 4)/2] = sqr(17/4) +/- sqr(1/4) = [sqr(17) +/- 1]/2.

Portanto, na igualdade (i) os membros são iguais e na igualdade (ii) os
membros são simétricos (opostos aditivos):
(i) [1 + sqr(17)]/2 = sqr{[9 + sqr(17)]/2} = [sqr(17) + 1]/2
(ii) [1 - sqr(17)]/2 = -[sqr(17) - 1]/2 != sqr{[9 - sqr(17)]/2} = [sqr(17) -
1]/2, mas {[1 - sqr(17)]/2}^2 = sqr{[9 - sqr(17)]/2}^2.


MOTIVO DE TERMOS ENCONTRADO A TERCEIRA RAIZ ELIMINADA: [1 + sqr(17)]/2

x = [1 + sqr(17)]/2
Esta solução foi obtida de (ix) y = 1 - x, logo: y = [1 - sqr(17)]/2

Substituindo os valores de x e y em (i) e (ii), teremos:
(i) y = sqr(5 - x) = [1 - sqr(17)]/2 = sqr{5 - [1 + sqr(17)]/2} = [1 -
sqr(17)]/2 = sqr{[9 - sqr(17)]/2}
(ii) x = sqr(5 - y) = [1 + sqr(17)]/2 = sqr{5 - [1 - sqr(17)]/2} = [1 +
sqr(17)]/2 = sqr{[9 + sqr(17)]/2}

A análise do motivo de termos encontrado a terceira raiz eliminada é
idêntica à análise do motivo de termos encontrado a segunda raiz eliminada,
uma vez que as igualdades (i) e (ii) da segunda raiz eliminada são idênticas
às igualdades (ii) e (i), respectivamente, da terceira raiz eliminada.


Atenciosamente,

Rogério Moraes de Carvalho
-Original Message-
From: [EMAIL PROTECTED] [mailto:[EMAIL PROTECTED] On
Behalf Of Claudio Buffara
Sent: sexta-feira, 4 de junho de 2004 13:13
To: [EMAIL PROTECTED]
Subject: Re: [obm-l] o valor de x - continuacao

on 04.06.04 06:32, Rogério Moraes de Carvalho at [EMAIL PROTECTED] wrote:

 Olá Claudio,
 
 Originalmente, eu resolvi esta questão usando a mesma idéia
 apresentada como quarta solução pelo Fabio, porém eu analisei as condições
 que devem ser satisfeitas em cada passo para possibilitar as
transformações
 no campo dos reais. Deste modo, eu consigo analisar a validade das
soluções
 encontradas. É importante ressaltar que na resolução de uma equação
 irracional no campo dos reais, a análise da condição de existência é tão
 importante quanto o fato de encontrar uma equação polinomial
equivalente.
 Pois, esta equivalência quase sempre é parcial, ou seja, geralmente apenas
 algumas raízes são compartilhadas.
 
Tudo bem. Concordo. Alias, uma das principais licoes desse problema eh
justamente essa: depois de resolver uma equacao onde coisas foram elevadas
ao quadrado, eh fundamental checar para ver se as solucoes encontradas sao,
de fato, solucoes da equacao original.

Mas voce nao respondeu a minha pergunta: de onde vieram as tres raizes
adicionais, especialmente a outra raiz positiva.

[]s,
Claudio.


=
Instruções para entrar na lista, sair da lista e usar a lista em
http://www.mat.puc-rio.br/~nicolau/olimp/obm-l.html
=




=
Instruções para entrar na lista, sair da lista e usar a lista em
http://www.mat.puc-rio.br/~nicolau/olimp/obm-l.html
=


RE: [obm-l] o valor de x - continuacao

2004-06-04 Por tôpico Rogério Moraes de Carvalho
Olá Claudio,

Originalmente, eu resolvi esta questão usando a mesma idéia
apresentada como quarta solução pelo Fabio, porém eu analisei as condições
que devem ser satisfeitas em cada passo para possibilitar as transformações
no campo dos reais. Deste modo, eu consigo analisar a validade das soluções
encontradas. É importante ressaltar que na resolução de uma equação
irracional no campo dos reais, a análise da condição de existência é tão
importante quanto o fato de encontrar uma equação polinomial equivalente.
Pois, esta equivalência quase sempre é parcial, ou seja, geralmente apenas
algumas raízes são compartilhadas.

PROBLEMA:

Resolva no campo dos reais s seguinte equação: sqr(5 - sqr(5 - x)) = x.


RESOLUÇÃO POSSÍVEL PARA O PROBLEMA:

Fazendo y = sqr(5 - x) (i), teremos:
x = sqr(5 - y) (ii)

Seguem as condições que permitem a equivalência das igualdades (i) e (ii)
mesmo após elevar ambos os membros ao quadrado.
Igualdade (i): y = 0 e 5 - x = 0 = x = 5 (iii)
Igualdade (ii): x = 0 e 5 - y = 0 = y = 5 (iv)

Das condições (iii) e (iv), chegamos a uma condição geral:
0 = x = 5 e 0 = y = 5 (v).

Se for satisfeita a condição geral (v), poderemos elevar ambos os membros
das igualdades (i) e (ii), ou seja:
y^2 = 5 - x = y^2 + x = 5 (vi)
x^2 = 5 - y = x^2 + y = 5 (vii)

Aplicando a propriedade transitiva da igualdade em (vi) e (vii), teremos:
y^2 + x = x^2 + y = y^2 - x^2 - y + x = 0 = (y - x)(y + x) - (y - x) = 0
= (y - x)(y - x - 1) = 0 = y = x (viii) ou y = 1 - x (ix)

Substituindo a (viii) na (vii), teremos:
x^2 = 5 - x = x^2 + x - 5 = 0 = x = [-1 +/- sqr(21)]/2
Verificando se as soluções satisfazem a condição geral (v):
x = [-1 - sqr(21)]/2 (Não satisfaz, pois x  0)
x = [-1 + sqr(21)]/2 (Satisfaz, pois 0 = x = 5. Observe que 0 = y = 5
uma vez que pela igualdade (viii) y = x.)

Substituindo a (ix) na (vii), teremos:
x^2 = 5 - (1 - x) = x^2 - x - 4 = 0 = x = [1 +/- sqr(17)]/2
Verificando se as soluções satisfazem a condição geral (v):
x = [1 - sqr(17)]/2 (Não satisfaz, pois x  0)
x = [1 + sqr(17)]/2 (Não satisfaz, pois apesar de satisfazer a primeira
parte da condição geral 0 = x = 5 não satisfaz a segunda, pois pela
igualdade (ix) y = 1 - x = y = 1 - [1 + sqr(17)]/2 = y = [1 - sqr(17)]/2,
ou seja, y  0.)

Resposta:
x = [-1 + sqr(21)]/2
Conjunto solução no campo dos reais: S = {[-1 + sqr(21)]/2}


Atenciosamente,

Rogério Moraes de Carvalho
-Original Message-
From: [EMAIL PROTECTED] [mailto:[EMAIL PROTECTED] On
Behalf Of Claudio Buffara
Sent: quinta-feira, 3 de junho de 2004 22:34
To: [EMAIL PROTECTED]
Subject: [obm-l] o valor de x - continuacao

Em todas as solucoes que o Fabio apresentou, aparecem as equacoes:
x^2 + x - 5 = 0
e
x^2 - x - 4 = 0

As raizes da primeira sao: (-1+raiz(21))/2  e  (-1-raiz(21))/2
As da segunda sao: (1+raiz(17))/2  e  (1-raiz(17))/2

Examinando a equacao original: sqrt(5 - sqrt(5 - x)) = x,
observamos que x e 5 - x precisam ser nao-negativos.
Ou seja, temos que ter 0 = x = 5.

Isso elimina as raizes (-1-raiz(21))/2 e (1-raiz(17))/2.

No entanto, verificamos que apenas (-1+raiz(21))/2 satisfaz a equacao
original.

O problema que eu proponho eh:
Explique porque (1+raiz(17))/2 nao satisfaz a equacao original.

[]s,
Claudio.

on 03.06.04 21:40, Fabio Dias Moreira at [EMAIL PROTECTED] wrote:

 
 biper said:
 Hoje recebi esta questão do meu colega, no iício pensei
 que fosse fácil, mas acabei me complicando, aí vai:
 
 Calcule o valor de x para:
 
 [5 - (5 - x)1/2]1/2 = x
 
 
 Eu desnenvolvendo caiu num sistema, será que é por aí
 mesmo?
 [...]
 
 Bom, eu não sei de qual sistema você está falando, mas existem várias
 soluções para este problema (eu suponho que você quis diser sqrt(5 -
 sqrt(5 - x)) = x).
 
 Primeira solução:
 
 Eu considero essa solução, enviada aqui para a lista pelo nosso colega
 Ralph, a mais bonita e natural de todas.
 
 Abra tudo:
 
 sqrt(5 - sqrt(5 - x)) = x =
 5 - sqrt(5 - x) = x^2 =
 sqrt(5 - x) = 5 - x^2 =
 5 - x = 25 - 10x^2 + x^4 =
 x^4 - 10x^2 + x + 20 = 0.
 
 Se essa equação puder ser resolvida sem apelar para a fórmula da equação
 do quarto grau, ela *tem* que poder ser fatorada. Se a gente soubesse
 algumas raízes, a gente até poderia fatorar o polinômio...
 
 Mas a gente sabe algumas dessas raízes! Não é difícil ver que sqrt(5 - x)
 = x = x = sqrt(5 - sqrt(5 - x)). Logo é razoável esperar que x^2 + x - 5
 divida o polinômio em que chegamos. E, de fato,
 
 x^4 - 10x^2 + x + 20 = (x^2 + x - 5)(x^2 - x - 4). Continuar daqui é
trivial.
 
 Segunda solução:
 
 Se você não vir esse fator, também é possível resolver o problema. É fácil
 ver que o polinômio não tem raízes raacionais. Se ele puder ser fatorado,
 ele *tem* que poder ser escrito como produto de dois polinômios de segundo
 grau, i.e.
 
 x^4 - 10x^2 + x + 20 = (x^2 + ax + b)(x^2 + cx + d).
 
 Abrindo o lado direito,
 
 x^4 - 10x^2 + x + 20 = x^4 + (a+c)x^3 + (b+d+ac)x^2 + (ad+bc)x + bd
 
 Logo temos que achar a, b, c, d inteiros tais que
 
 (1) a + c = 0
 (2) b + d + ac = -10
 (3) ad

[obm-l] RE: [obm-l] Números complexos e outro

2004-05-30 Por tôpico Rogério Moraes de Carvalho
Olá Pedro,

Os problemas 2 e 3 já foram resolvidos.

O problema 1 pode ser resolvido facilmente pela aplicação de dois teoremas,
um dos quais foi colocado no enunciado.

TEOREMA 1: Se r é o resto da divisão de a por b então o resto da divisão de
a^n por b é igual ao resto da divisão de r ^n por b.

TEOREMA 2: Se r é o resto da divisão de a por 9, então r é o resto da
divisão da soma dos algarismos de a por 9.


RESOLUÇÃO POSSÍVEL PARA O PROBLEMA 1:

Adote a seguinte notação: r(a, b) - resto da divisão de a por b.
Aplicando o teorema 2 sucessivas vezes, teremos:
r(5342177,9)= r(5+3+4+2+1+7+7,9)= r(29,9)=r(2+9,9)=r(11,9)=r(1+1,9)=2
Aplicando o teorema 1 e depois o teorema 2 sucessivas vezes:
r(5342177^8,9)=r(2^8,9)=r(256,9)=r(2+5+6,9)=r(13,9)=r(1+3,9)=4

Resposta: 4


Abraços,

Rogério Moraes de Carvalho

-Original Message-
From: [EMAIL PROTECTED] [mailto:[EMAIL PROTECTED] On
Behalf Of pedro rajão
Sent: sábado, 29 de maio de 2004 18:09
To: [EMAIL PROTECTED]
Subject: [obm-l] Números complexos e outro

Olá

Eis alguns exercícios :

1 ] Sejam a e b dois números naturais com b # 0 . Se r é oresto da divisão 
de a por b então o resto da divisão de a^n por b é igual ao resto da divisão

de r ^n por b . Utilizando este teorema , calcular o resto da divisão de 
[5342177]^8 por 9.

2 ]  ITA - As raízes de ordem 4 do número z=e^Pi*i/2  , onde i é a unidade 
imaginária , são [na forma trigonométrica] ?

3 ]  ITA - Seja S o conjunto dos números complexos que satisfazem, 
simultaneamente às equções
| z - 3 i | = 3 e | z + i | = | z - 2 - i |
O produto de todos os elememtos de S é iguaL a ?

_
MSN Messenger: converse com os seus amigos online.  
http://messenger.msn.com.br

=
Instruções para entrar na lista, sair da lista e usar a lista em
http://www.mat.puc-rio.br/~nicolau/olimp/obm-l.html
=




=
Instruções para entrar na lista, sair da lista e usar a lista em
http://www.mat.puc-rio.br/~nicolau/olimp/obm-l.html
=


[obm-l] RE: [obm-l] RE: [obm-l] Números Interessantes

2004-05-29 Por tôpico Rogério Moraes de Carvalho
Olá Qwert,

Você está certo. Eu realmente não atentei para o fato de que os 10
algarismos deveriam ser diferentes. Foi muita falta de atenção da minha
parte. É certo que o problema se torna um tanto quanto elementar se não for
colocada nenhuma condição sobre os 10 algarismos.

Eu também tinha feito um programa para conferir se o resultado que
eu havia encontrado estava correto, sendo que eu não havia considerado com
todos os 10 algarismos diferentes. Então, eu fiz uma pequena modificação no
programa para fazer a contagem do número de múltiplos de 1 com 10
algarismos diferentes. Segue o resultado apresentado pelo programa.

Múltiplos positivos de 1 com 10 algarismos: 810.009
Múltiplos positivos ou negativos de 1 com 10 algarismos: 1.620.018
Múltiplos positivos de 1 com 10 algarismos diferentes: 3.456
Múltiplos positivos ou negativos de 1 com 10 algarismos diferentes:
6.912

Assim que eu tiver com mais tempo, vou tentar resolver o problema
levando em consideração que os 10 algarismos são diferentes.

Abraços,

Rogério Moraes de Carvalho
-Original Message-
From: [EMAIL PROTECTED] [mailto:[EMAIL PROTECTED] On
Behalf Of Qwert Smith
Sent: sexta-feira, 28 de maio de 2004 22:28
To: [EMAIL PROTECTED]
Subject: RE: [obm-l] RE: [obm-l] Números Interessantes

Rogerio,
   vc repondeu quantos numeros tem 10 algarismos e sao multiplos de 1.
A perguinta pede so os multiplos com algarismos distintos.  Por exemplo o
primeiro multiplo de 1 que satisfaz essa conicao e 1023489765

A unica coisa que eu consegui pensar e que um numero de 10 algarismos
distintos e sempre multiplo de 9, logo da pra diminuir a conta pra quais os
multiplos de 9, so que isso inclui numeros como 99.

De qualquer forma rodei a manivela aki no meu laptop e ele cuspiu que
3456 multiplos de 1 sao numeros de 10 algarismos distintos.


Olá André,

   Eu já havia postado uma resolução possível para este problema. Veja
o link: 
http://www.mat.puc-rio.br/~nicolau/olimp/obm-l.200405/msg00793.html.

Atenciosamente,

Rogério Moraes de Carvalho
 
    Questão:  Quantos são os números com 10 algarismo diferentes entre
si e divisível por 1.
   Dizer que eles estão incluídos entre os números interessantes está
correto?

_
Get 200+ ad-free, high-fidelity stations and LIVE Major League Baseball 
Gameday Audio! http://radio.msn.click-url.com/go/onm00200491ave/direct/01/

=
Instruções para entrar na lista, sair da lista e usar a lista em
http://www.mat.puc-rio.br/~nicolau/olimp/obm-l.html
=




=
Instruções para entrar na lista, sair da lista e usar a lista em
http://www.mat.puc-rio.br/~nicolau/olimp/obm-l.html
=


RE: [obm-l] Geom. Plana

2004-05-29 Por tôpico Rogério Moraes de Carvalho
Olá Thor,

Segue uma resolução possível para esta questão.


RESOLUÇÃO POSSÍVEL:

Se os dois quadrados concêntricos têm os mesmos perímetros (P), então eles
são congruentes, pois terão os mesmos lados (L = P/4). Como o esboço da
figura é muito importante para facilitar a compreensão da resolução, segue a
descrição do mesmo.

Seja ABCD um quadrado de perímetro P, lado L (L = P/4) e centro O. Agora
obtenha o outro quadrado A'B'C'D' a partir da rotação de um ângulo BETA de
ABCD em torno da sua origem O no sentido horário, tal que 0  BETA  90°.
Nomeie os pontos de interseção dos dois quadrados como H[1], H[2], H[3],
..., H[8] no sentido horário partindo do ponto de interseção mais próximo de
A no segmento AB.

Segue a demonstração de que o ângulo BETA (AOA') de rotação do quadrado
ABCD deve ser igual a 45°.
Para isto, considere P o ponto de interseção do segmento AO com o lado D'A'
do quadrado A'B'C'D' e Q o ponto de interseção do segmento A'O com o lado AB
do quadrado ABCD. No quadrilátero PH[1]QO o ângulo PH[1]Q corresponde a um
dos ângulos internos de um octógono regular (dado do enunciado), então:
PH[1]Q = (8 - 2).180°/8 = 135°
PH[1]A + PH[1]Q = 180° = PH[1]A + 135° = 180° = PH[1]A = 45°
PAH[1] = 45° (ângulo agudo formado entre uma diagonal e um lado do quadrado
ABCD)
Pelo Teorema do Ângulo Interno: OPH[1] = PAH[1] + PH[1]A = OPH[1] = 90°
Analogamente, concluímos que H[1]QO = 90°
A soma dos ângulos internos do quadrilátero OPH[1]Q é igual a 360°,
portanto: OPH[1] + PH[1]Q + H[1]QO + QOP = 360° = 90° + 135° + 90° +
BETA = 360° = BETA = 45°

Observe que: AO = AP + PO (i)

AO: metade da diagonal do quadrado ABCD, portanto AO = L.sqr(2)/2 (ii)

AP: metade do lado do octógono regular (X/2), pois na dedução do ângulo de
rotação (BETA) nós concluímos que o triângulo APH[1] é retângulo isósceles.
Analogamente, podemos concluir que APH[8] é retângulo isósceles. Como o lado
AP é comum, podemos dizer que os triângulo APH[1] e APH[8] são congruentes
pelo critério ALA. Considerando X como a medida do lado do octógono regular
H[1]H[2]H[3]H[4]H[5]H[6]H[7]H[8], teremos AP = PH[1] = PH[8] = X/2 (iii)

PO: metade do lado do quadrado A'B'C'D', portanto PO = L/2 (iv)

Substituindo as igualdades (ii), (iii) e (iv) na igualdade (i), teremos:
L.sqr(2)/2 = X/2 + L/2 = X = [sqr(2) - 1].L
Como L = P/4: X = {[sqr(2) - 1].P}/4

Resposta: {[sqr(2) - 1].P}/4

Atenciosamente,

Rogério Moraes de Carvalho
__
From: [EMAIL PROTECTED] [mailto:[EMAIL PROTECTED] On
Behalf Of Thor
Sent: sexta-feira, 28 de maio de 2004 19:25
To: [EMAIL PROTECTED]
Subject: [obm-l] Geom. Plana

 
 
Dois quadrados concêntricos de perímetro P , cada , são interceptados de
modo que os pontos de interseção
de seus lados sejam os vértices de um octógono regular.Qual é o lado desse
octógono em funçao de P?
 
 
Tentei fazer , e cheguei na lei dos co-senos , e dai parei
 
    Agradeço desde de já.



=
Instruções para entrar na lista, sair da lista e usar a lista em
http://www.mat.puc-rio.br/~nicolau/olimp/obm-l.html
=


RE: [obm-l] Geom. Plana

2004-05-29 Por tôpico Rogério Moraes de Carvalho
Olá Osvaldo,

Não há dados suficientes no enunciado do problema que permitam que
você conclua de forma DIRETA que os triângulos ABC e IBJ são semelhantes. É
fácil e direto concluir que os ângulos do triângulo ABC são os seguintes:
ABC = 90°, BCA = 45° e CAB = 45°, uma vez que se trata de um triângulo
retângulo isósceles (AB = BC = L e ABC = 90°). Porém, apesar de podermos
concluir diretamente que no triângulo IBJ o ângulo IBJ = 90°, não se pode
concluir diretamente que BJI = 45° ou JIB = 45°. Sendo assim, não é
correto fazer a semelhança entre os triângulos ABC e IBJ pelo critério AA~,
a não ser que se prove antes que um dos ângulos agudos do triângulo IBJ é
igual a 45°. Uma possível demonstração está colocada na solução que eu
propus.

Atenciosamente,

Rogério Moraes de Carvalho
-Original Message-
From: [EMAIL PROTECTED] [mailto:[EMAIL PROTECTED] On
Behalf Of Osvaldo
Sent: sábado, 29 de maio de 2004 13:58
To: obm-l
Subject: RE: [obm-l] Geom. Plana

E ai Thór!


Creio que uma outra res. possível seja algo como esta:

Dois quadrados com mesmo perímetro são certamente 
congruentes.
Seja l o lado do quadrado, ambos os quadrados têm 
perímetro P, assim P=l/4

Faça o desenho. Sejam A,B,C,D os vértices do primeiro 
quadrado e sejam E,F,G,H os vértices do outro quadrado 
de tal forma que B está mais proxima de EF. Sejam I e J 
as intersecções de EF com os lados AB e BC, 
respectivamente; O o centro dos quadrados e X a 
intersecção de OB com o lado EF.

Trace a diagonal AC. Os triang. ABC e IBJ são 
semelhantes caso ~AA. Da proporção AC/OB=IJ/XB 
temos que IJ=2.XB=2.y, onde IJ é o lado do octógono 
regular. 

Observe que a diagonal do quadrado corresponde ao lado 
do quadrado somada com duas vezes y=XB, ou seja, l.sqrt
(2)=l+2.y= y=l.(sqrt(2-1))/2
Observe que o triang. ret. XBJ é isosceles, logo o lado 
do octógono corresponde a duas vezes y ou seja l.(sqrt
(2)-1)=
(P/4).(sqrt(2)-1)

Falow ai




 Olá Thor,
 
   Segue uma resolução possível para esta questão.
 
 
 RESOLUÇÃO POSSÍVEL:
 
 Se os dois quadrados concêntricos têm os mesmos 
perímetros (P), então eles
 são congruentes, pois terão os mesmos lados (L = 
P/4). Como o esboço da
 figura é muito importante para facilitar a 
compreensão da resolução, segue a
 descrição do mesmo.
 
 Seja ABCD um quadrado de perímetro P, lado L (L = 
P/4) e centro O. Agora
 obtenha o outro quadrado A'B'C'D' a partir da rotação 
de um ângulo BETA de
 ABCD em torno da sua origem O no sentido horário, tal 
que 0  BETA  90°.
 Nomeie os pontos de interseção dos dois quadrados 
como H[1], H[2], H[3],
 ..., H[8] no sentido horário partindo do ponto de 
interseção mais próximo de
 A no segmento AB.
 
 Segue a demonstração de que o ângulo BETA (AOA') de 
rotação do quadrado
 ABCD deve ser igual a 45°.
 Para isto, considere P o ponto de interseção do 
segmento AO com o lado D'A'
 do quadrado A'B'C'D' e Q o ponto de interseção do 
segmento A'O com o lado AB
 do quadrado ABCD. No quadrilátero PH[1]QO o ângulo PH
[1]Q corresponde a um
 dos ângulos internos de um octógono regular (dado do 
enunciado), então:
 PH[1]Q = (8 - 2).180°/8 = 135°
 PH[1]A + PH[1]Q = 180° = PH[1]A + 135° = 180° = 
PH[1]A = 45°
 PAH[1] = 45° (ângulo agudo formado entre uma 
diagonal e um lado do quadrado
 ABCD)
 Pelo Teorema do Ângulo Interno: OPH[1] = PAH[1] + 
PH[1]A = OPH[1] = 90°
 Analogamente, concluímos que H[1]QO = 90°
 A soma dos ângulos internos do quadrilátero OPH[1]Q é 
igual a 360°,
 portanto: OPH[1] + PH[1]Q + H[1]QO + QOP = 360° 
= 90° + 135° + 90° +
 BETA = 360° = BETA = 45°
 
 Observe que: AO = AP + PO (i)
 
 AO: metade da diagonal do quadrado ABCD, portanto AO 
= L.sqr(2)/2 (ii)
 
 AP: metade do lado do octógono regular (X/2), pois na 
dedução do ângulo de
 rotação (BETA) nós concluímos que o triângulo APH[1] 
é retângulo isósceles.
 Analogamente, podemos concluir que APH[8] é retângulo 
isósceles. Como o lado
 AP é comum, podemos dizer que os triângulo APH[1] e 
APH[8] são congruentes
 pelo critério ALA. Considerando X como a medida do 
lado do octógono regular
 H[1]H[2]H[3]H[4]H[5]H[6]H[7]H[8], teremos AP = PH[1] 
= PH[8] = X/2 (iii)
 
 PO: metade do lado do quadrado A'B'C'D', portanto PO 
= L/2 (iv)
 
 Substituindo as igualdades (ii), (iii) e (iv) na 
igualdade (i), teremos:
 L.sqr(2)/2 = X/2 + L/2 = X = [sqr(2) - 1].L
 Como L = P/4: X = {[sqr(2) - 1].P}/4
 
 Resposta: {[sqr(2) - 1].P}/4
 
 Atenciosamente,
 
 Rogério Moraes de Carvalho
 __
 From: [EMAIL PROTECTED] [mailto:owner-obm-
[EMAIL PROTECTED] On
 Behalf Of Thor
 Sent: sexta-feira, 28 de maio de 2004 19:25
 To: [EMAIL PROTECTED]
 Subject: [obm-l] Geom. Plana
 
  
  
 Dois quadrados concêntricos de perímetro P , cada , 
são interceptados de
 modo que os pontos de interseção
 de seus lados sejam os vértices de um octógono 
regular.Qual é o lado desse
 octógono em funçao de P?
  
  
 Tentei fazer , e cheguei na lei dos co-senos , e dai 
parei
  
     Agradeço desde de já

RE: [obm-l] Geom. Plana

2004-05-29 Por tôpico Rogério Moraes de Carvalho
Olá Osvaldo,

Observe que você está tirando conclusões baseadas somente no
desenho. O enunciado não fornece nenhuma informação que permita que você
conclua de maneira DIRETA que os segmentos XJ e OC são paralelos. Ao afirmar
que os segmentos XJ e OC são paralelos, você está afirmando de maneira
indireta que o quadrado EFGH pode ser obtido a partir do quadrado ABCD por
uma rotação de 45° em torno do seu centro.

Abraços,

Rogério Moraes de Carvalho
-Original Message-
From: [EMAIL PROTECTED] [mailto:[EMAIL PROTECTED] On
Behalf Of Osvaldo
Sent: sábado, 29 de maio de 2004 16:55
To: obm-l
Subject: RE: [obm-l] Geom. Plana

Okay, concordo!
Porém, não mencionei na minha solução por ME PARECER 
meio direto, desculpe.
Tipo, olhando para os ângulos XJB e OCB, concluímos que 
eles têm mesmo valor (ang. correspondentes ja que BC é 
comum e os segmentos XJ e OC são //), ou seja 45° , dai 
completo o angulo BIX, ou seja BIX+90+45=180 ou seja, 
BIX vale 45 tambem. 

Falow, até. 

 Olá Osvaldo,
 
   Não há dados suficientes no enunciado do 
problema que permitam que
 você conclua de forma DIRETA que os triângulos ABC e 
IBJ são semelhantes. É
 fácil e direto concluir que os ângulos do triângulo 
ABC são os seguintes:
 ABC = 90°, BCA = 45° e CAB = 45°, uma vez que se 
trata de um triângulo
 retângulo isósceles (AB = BC = L e ABC = 90°). 
Porém, apesar de podermos
 concluir diretamente que no triângulo IBJ o ângulo 
IBJ = 90°, não se pode
 concluir diretamente que BJI = 45° ou JIB = 45°. 
Sendo assim, não é
 correto fazer a semelhança entre os triângulos ABC e 
IBJ pelo critério AA~,
 a não ser que se prove antes que um dos ângulos 
agudos do triângulo IBJ é
 igual a 45°. Uma possível demonstração está colocada 
na solução que eu
 propus.
 
 Atenciosamente,
 
 Rogério Moraes de Carvalho
 -Original Message-
 From: [EMAIL PROTECTED] [mailto:owner-obm-
[EMAIL PROTECTED] On
 Behalf Of Osvaldo
 Sent: sábado, 29 de maio de 2004 13:58
 To: obm-l
 Subject: RE: [obm-l] Geom. Plana
 
 E ai Thór!
 
 
 Creio que uma outra res. possível seja algo como esta:
 
 Dois quadrados com mesmo perímetro são certamente 
 congruentes.
 Seja l o lado do quadrado, ambos os quadrados têm 
 perímetro P, assim P=l/4
 
 Faça o desenho. Sejam A,B,C,D os vértices do primeiro 
 quadrado e sejam E,F,G,H os vértices do outro 
quadrado 
 de tal forma que B está mais proxima de EF. Sejam I e 
J 
 as intersecções de EF com os lados AB e BC, 
 respectivamente; O o centro dos quadrados e X a 
 intersecção de OB com o lado EF.
 
 Trace a diagonal AC. Os triang. ABC e IBJ são 
 semelhantes caso ~AA. Da proporção AC/OB=IJ/XB 
 temos que IJ=2.XB=2.y, onde IJ é o lado do octógono 
 regular. 
 
 Observe que a diagonal do quadrado corresponde ao 
lado 
 do quadrado somada com duas vezes y=XB, ou seja, 
l.sqrt
 (2)=l+2.y= y=l.(sqrt(2-1))/2
 Observe que o triang. ret. XBJ é isosceles, logo o 
lado 
 do octógono corresponde a duas vezes y ou seja l.(sqrt
 (2)-1)=
 (P/4).(sqrt(2)-1)
 
 Falow ai
 
 
 
 
  Olá Thor,
  
  Segue uma resolução possível para esta questão.
  
  
  RESOLUÇÃO POSSÍVEL:
  
  Se os dois quadrados concêntricos têm os mesmos 
 perímetros (P), então eles
  são congruentes, pois terão os mesmos lados (L = 
 P/4). Como o esboço da
  figura é muito importante para facilitar a 
 compreensão da resolução, segue a
  descrição do mesmo.
  
  Seja ABCD um quadrado de perímetro P, lado L (L = 
 P/4) e centro O. Agora
  obtenha o outro quadrado A'B'C'D' a partir da 
rotação 
 de um ângulo BETA de
  ABCD em torno da sua origem O no sentido horário, 
tal 
 que 0  BETA  90°.
  Nomeie os pontos de interseção dos dois quadrados 
 como H[1], H[2], H[3],
  ..., H[8] no sentido horário partindo do ponto de 
 interseção mais próximo de
  A no segmento AB.
  
  Segue a demonstração de que o ângulo BETA (AOA') 
de 
 rotação do quadrado
  ABCD deve ser igual a 45°.
  Para isto, considere P o ponto de interseção do 
 segmento AO com o lado D'A'
  do quadrado A'B'C'D' e Q o ponto de interseção do 
 segmento A'O com o lado AB
  do quadrado ABCD. No quadrilátero PH[1]QO o ângulo 
PH
 [1]Q corresponde a um
  dos ângulos internos de um octógono regular (dado 
do 
 enunciado), então:
  PH[1]Q = (8 - 2).180°/8 = 135°
  PH[1]A + PH[1]Q = 180° = PH[1]A + 135° = 180° 
= 
 PH[1]A = 45°
  PAH[1] = 45° (ângulo agudo formado entre uma 
 diagonal e um lado do quadrado
  ABCD)
  Pelo Teorema do Ângulo Interno: OPH[1] = PAH[1] + 
 PH[1]A = OPH[1] = 90°
  Analogamente, concluímos que H[1]QO = 90°
  A soma dos ângulos internos do quadrilátero OPH[1]Q 
é 
 igual a 360°,
  portanto: OPH[1] + PH[1]Q + H[1]QO + QOP = 360° 
 = 90° + 135° + 90° +
  BETA = 360° = BETA = 45°
  
  Observe que: AO = AP + PO (i)
  
  AO: metade da diagonal do quadrado ABCD, portanto 
AO 
 = L.sqr(2)/2 (ii)
  
  AP: metade do lado do octógono regular (X/2), pois 
na 
 dedução do ângulo de
  rotação (BETA) nós concluímos que o triângulo APH
[1] 
 é retângulo isósceles.
  Analogamente, podemos

[obm-l] RE: [obm-l] RES: [obm-l] RE: [obm-l] Números Interessantes

2004-05-29 Por tôpico Rogério Moraes de Carvalho
Olá Douglas,

Com certeza. O Qwert já havia feito esta observação. Assim que eu
tiver com mais tempo disponível, eu vou tentar resolver o problema proposto,
ou seja, considerando todos os algarismos diferentes. De qualquer modo, eu
postei uma mensagem com a resposta do problema obtida por intermédio de um
programa.

Abraços,

Rogério Moraes de Carvalho
-Original Message-
From: [EMAIL PROTECTED] [mailto:[EMAIL PROTECTED] On
Behalf Of Douglas Ribeiro Silva
Sent: sábado, 29 de maio de 2004 22:11
To: [EMAIL PROTECTED]
Subject: [obm-l] RES: [obm-l] RE: [obm-l] Números Interessantes

Olá Rogério, eu dei uma olhada na sua resolução para o problema e não
discuto que a matemática que você usou está correta... Mas acho que a
intenção do problema proposto não foi chegar na resposta que você chegou.

A questão pede Quantos são os números com 10 algarismo diferentes entre
si e divisível por 1

Você achou todos os números inteiros divisíveis por 1 que tem 10
algarismos, mas não necessariamente algarismos distintos.

Por exemplo: 10 possui 5 zeros e 5 uns

O numero teria que ser do tipo 1896753042
Ao meu ver foi isso que a questão pediu...

Um abraço, Douglas Ribeiro

-Mensagem original-
De: [EMAIL PROTECTED] [mailto:[EMAIL PROTECTED] Em nome
de Rogério Moraes de Carvalho
Enviada em: sexta-feira, 28 de maio de 2004 06:48
Para: [EMAIL PROTECTED]
Assunto: [obm-l] RE: [obm-l] Números Interessantes

Olá André,

Eu já havia postado uma resolução possível para este problema. Veja
o link: http://www.mat.puc-rio.br/~nicolau/olimp/obm-l.200405/msg00793.html.

Atenciosamente,

Rogério Moraes de Carvalho

From: [EMAIL PROTECTED] [mailto:[EMAIL PROTECTED] On
Behalf Of Andre
Sent: quinta-feira, 27 de maio de 2004 23:59
To: [EMAIL PROTECTED]
Subject: [obm-l] Números Interessantes

Olá,
 
    Questão:  Quantos são os números com 10 algarismo diferentes entre
si e divisível por 1.
    Dizer que eles estão incluídos entre os números interessantes está
correto?



=
Instruções para entrar na lista, sair da lista e usar a lista em
http://www.mat.puc-rio.br/~nicolau/olimp/obm-l.html
=


=
Instruções para entrar na lista, sair da lista e usar a lista em
http://www.mat.puc-rio.br/~nicolau/olimp/obm-l.html
=



=
Instruções para entrar na lista, sair da lista e usar a lista em
http://www.mat.puc-rio.br/~nicolau/olimp/obm-l.html
=


RE: [obm-l] Geom. Plana

2004-05-29 Por tôpico Rogério Moraes de Carvalho
Osvaldo,

Para chegar a esta conclusão, você vai ter que levar em conta o
seguinte dado do enunciado do problema: ... os pontos de interseção
de seus lados sejam os vértices de um octógono regular.. Sendo assim, você
pode levar em consideração que todos os ângulos internos do octógono regular
são iguais a 135°. Usando esta informação, você pode comprovar as suas
conclusões de várias maneiras. Observe que você não tinha usado esta
informação, portanto você estava resolvendo um problema mais genérico como
se fosse um caso particular.

Obs.: Se estiver com tempo, dê uma analisada na resolução que eu propus. A
resolução parece extensa, mas isto ocorreu devido a eu ter explicado
detalhadamente todos os passos e cálculos. Ao analisar os passos da
resolução, você poderá concluir que a resolução é bem simples.

Abraços,

Rogério Moraes de Carvalho
-Original Message-
From: [EMAIL PROTECTED] [mailto:[EMAIL PROTECTED] On
Behalf Of Osvaldo
Sent: sábado, 29 de maio de 2004 21:50
To: obm-l
Subject: RE: [obm-l] Geom. Plana

E ai Rogério!

Poxa, acho que vc esta certo. Será que tem alguma 
maneira direta de se fazer isto? se tiver favor me 
mandar. valew!





 Olá Osvaldo,
 
   Observe que você está tirando conclusões 
baseadas somente no
 desenho. O enunciado não fornece nenhuma informação 
que permita que você
 conclua de maneira DIRETA que os segmentos XJ e OC 
são paralelos. Ao afirmar
 que os segmentos XJ e OC são paralelos, você está 
afirmando de maneira
 indireta que o quadrado EFGH pode ser obtido a partir 
do quadrado ABCD por
 uma rotação de 45° em torno do seu centro.
 
 Abraços,
 
 Rogério Moraes de Carvalho
 -Original Message-
 From: [EMAIL PROTECTED] [mailto:owner-obm-
[EMAIL PROTECTED] On
 Behalf Of Osvaldo
 Sent: sábado, 29 de maio de 2004 16:55
 To: obm-l
 Subject: RE: [obm-l] Geom. Plana
 
 Okay, concordo!
 Porém, não mencionei na minha solução por ME PARECER 
 meio direto, desculpe.
 Tipo, olhando para os ângulos XJB e OCB, concluímos 
que 
 eles têm mesmo valor (ang. correspondentes ja que BC 
é 
 comum e os segmentos XJ e OC são //), ou seja 45° , 
dai 
 completo o angulo BIX, ou seja BIX+90+45=180 ou seja, 
 BIX vale 45 tambem. 
 
 Falow, até. 
 
  Olá Osvaldo,
  
  Não há dados suficientes no enunciado do 
 problema que permitam que
  você conclua de forma DIRETA que os triângulos ABC 
e 
 IBJ são semelhantes. É
  fácil e direto concluir que os ângulos do triângulo 
 ABC são os seguintes:
  ABC = 90°, BCA = 45° e CAB = 45°, uma vez que se 
 trata de um triângulo
  retângulo isósceles (AB = BC = L e ABC = 90°). 
 Porém, apesar de podermos
  concluir diretamente que no triângulo IBJ o ângulo 
 IBJ = 90°, não se pode
  concluir diretamente que BJI = 45° ou JIB = 45°. 
 Sendo assim, não é
  correto fazer a semelhança entre os triângulos ABC 
e 
 IBJ pelo critério AA~,
  a não ser que se prove antes que um dos ângulos 
 agudos do triângulo IBJ é
  igual a 45°. Uma possível demonstração está 
colocada 
 na solução que eu
  propus.
  
  Atenciosamente,
  
  Rogério Moraes de Carvalho
  -Original Message-
  From: [EMAIL PROTECTED] [mailto:owner-obm-
 [EMAIL PROTECTED] On
  Behalf Of Osvaldo
  Sent: sábado, 29 de maio de 2004 13:58
  To: obm-l
  Subject: RE: [obm-l] Geom. Plana
  
  E ai Thór!
  
  
  Creio que uma outra res. possível seja algo como 
esta:
  
  Dois quadrados com mesmo perímetro são certamente 
  congruentes.
  Seja l o lado do quadrado, ambos os quadrados têm 
  perímetro P, assim P=l/4
  
  Faça o desenho. Sejam A,B,C,D os vértices do 
primeiro 
  quadrado e sejam E,F,G,H os vértices do outro 
 quadrado 
  de tal forma que B está mais proxima de EF. Sejam I 
e 
 J 
  as intersecções de EF com os lados AB e BC, 
  respectivamente; O o centro dos quadrados e X a 
  intersecção de OB com o lado EF.
  
  Trace a diagonal AC. Os triang. ABC e IBJ são 
  semelhantes caso ~AA. Da proporção AC/OB=IJ/XB 
  temos que IJ=2.XB=2.y, onde IJ é o lado do octógono 
  regular. 
  
  Observe que a diagonal do quadrado corresponde ao 
 lado 
  do quadrado somada com duas vezes y=XB, ou seja, 
 l.sqrt
  (2)=l+2.y= y=l.(sqrt(2-1))/2
  Observe que o triang. ret. XBJ é isosceles, logo o 
 lado 
  do octógono corresponde a duas vezes y ou seja l.
(sqrt
  (2)-1)=
  (P/4).(sqrt(2)-1)
  
  Falow ai
  
  
  
  
   Olá Thor,
   
 Segue uma resolução possível para esta questão.
   
   
   RESOLUÇÃO POSSÍVEL:
   
   Se os dois quadrados concêntricos têm os mesmos 
  perímetros (P), então eles
   são congruentes, pois terão os mesmos lados (L = 
  P/4). Como o esboço da
   figura é muito importante para facilitar a 
  compreensão da resolução, segue a
   descrição do mesmo.
   
   Seja ABCD um quadrado de perímetro P, lado L (L = 
  P/4) e centro O. Agora
   obtenha o outro quadrado A'B'C'D' a partir da 
 rotação 
  de um ângulo BETA de
   ABCD em torno da sua origem O no sentido horário, 
 tal 
  que 0  BETA  90°.
   Nomeie os pontos de interseção dos dois quadrados 
  como

[obm-l] RE: [obm-l] Números Interessantes

2004-05-28 Por tôpico Rogério Moraes de Carvalho
Olá André,

Eu já havia postado uma resolução possível para este problema. Veja
o link: http://www.mat.puc-rio.br/~nicolau/olimp/obm-l.200405/msg00793.html.

Atenciosamente,

Rogério Moraes de Carvalho

From: [EMAIL PROTECTED] [mailto:[EMAIL PROTECTED] On
Behalf Of Andre
Sent: quinta-feira, 27 de maio de 2004 23:59
To: [EMAIL PROTECTED]
Subject: [obm-l] Números Interessantes

Olá,
 
    Questão:  Quantos são os números com 10 algarismo diferentes entre
si e divisível por 1.
    Dizer que eles estão incluídos entre os números interessantes está
correto?



=
Instruções para entrar na lista, sair da lista e usar a lista em
http://www.mat.puc-rio.br/~nicolau/olimp/obm-l.html
=


RE: [obm-l] Soma...

2004-05-28 Por tôpico Rogério Moraes de Carvalho
Olá David,

Se você considerar S[n] como um polinômio de grau k em n (k inteiro
positivo), então:

S[n]=a[k].n^k+a[k-1].n^(k-1)+...+a[1].n+a[0], tais que a[0], a[1], ..., a[k]
são os coeficientes de S[n] e a[k]!=0.

S[n-1]=a[k].(n-1)^k+a[k-1].(n-1)^(k-1)+...+a[1].(n-1)+a[0]

Considerando a notação C(u, v)=u!/[v!(u-v)!], com u e v inteiros não
negativos e u = v, e aplicando o desenvolvimento do binômio de Newton nas
expressões (n-1)^p, com p pertencente a {1, 2, ..., k}, teremos:

S[n]-S[n-1] = {a[k]-C(k,0).a[k]}.n^k +
{a[k-1]+C(k,1).a[k]-C(k-1,0).a[k-1]}.n^(k-1) + ...
S[n]-S[n-1] = {a[k]-a[k]}.n^k + {a[k-1]+k.a[k]-a[k-1]}.n^(k-1) + ...
S[n]-S[n-1] = k.a[k].n^(k-1) + ...

Como, por hipótese, k é inteiro positivo e a[k]!=0, então k.a[k]!=0. Sendo
assim: grau{S[n]-S[n-1]} = k-1 (i)
Como: S[n]-S[n-1]=n^2 = grau{S[n]-S[n-1]} = 2 (ii)

Por (i) e (ii): k-1 = 2 = k = 3


Atenciosamente,

Rogério Moraes de Carvalho
-Original Message-
From: [EMAIL PROTECTED] [mailto:[EMAIL PROTECTED] On
Behalf Of David M. Cardoso
Sent: quarta-feira, 26 de maio de 2004 20:49
To: [EMAIL PROTECTED]
Subject: RES: [obm-l] Soma...


Extraindo dessa mensagem essa parte:

   Seja S[n] o polinômio que representa a soma dos 
 quadrados dos n primeiros inteiros positivos, então podemos 
 concluir que:
 S[n] = S[n - 1] + n^2 = S[n] - S[n - 1] = n^2 (i)
 
 Logo S[n] tem que ser um polinômio de grau 3, uma vez que na 
 diferença S[n]
 - S[n - 1] os termos de maior grau dos polinômios vão ser 
 cancelados.

Não entendi pq o dá pra inferir que o grau do polinomio é 3...
Será alguem pode explicar isso?
 

 -Mensagem original-
 De: [EMAIL PROTECTED] 
 [mailto:[EMAIL PROTECTED] Em nome de Rogério Moraes 
 de Carvalho
 Enviada em: quarta-feira, 19 de maio de 2004 10:25
 Para: [EMAIL PROTECTED]
 Assunto: RE: [obm-l] Soma...
 
 Olá Crom,
 
   Muitos livros de Matemática apresentam uma possível 
 dedução da fórmula da soma das potências k-ésimas (k inteiro 
 positivo) dos n primeiros inteiros positivos pelo método que 
 você apresentou parcialmente, ou seja, usando o 
 desenvolvimento do binômio de Newton (x + 1)^(k + 1). Ao 
 aplicar o somatório com x variando de 1 até n a ambos os 
 membros da igualdade, os termos de grau (k + 1) podem ser 
 cancelados, com exceção de (n + 1)^(k + 1) no primeiro membro 
 da igualdade e 1^(k + 1) = 1 no segundo membro da igualdade. 
 Porém, para descobrir a fórmula da soma das potências 
 k-ésimas, nós precisamos conhecer todas as fórmulas das somas 
 das potências com expoente de 1 até (k - 1). Sendo assim, nós 
 encontramos uma fórmula de recorrência para deduzir a soma 
 das potências k-ésimas dos n primeiros inteiros positivos, 
 porém o processo vai ficando muito longo à medida que os 
 expoentes vão crescendo.
 
   A seguir, eu apresento um método que pode ser utilizado 
 para encontrar a soma das potências k-ésimas dos n primeiros 
 inteiros positivos de forma direta. Neste método, não há a 
 necessidade de se conhecer as fórmulas das somas das 
 potências com expoente de 1 até (k - 1)
 
 
 DEDUÇÃO POSSÍVEL:
 
   Seja S[n] o polinômio que representa a soma dos 
 quadrados dos n primeiros inteiros positivos, então podemos 
 concluir que:
 S[n] = S[n - 1] + n^2 = S[n] - S[n - 1] = n^2 (i)
 
 Logo S[n] tem que ser um polinômio de grau 3, uma vez que na 
 diferença S[n]
 - S[n - 1] os termos de maior grau dos polinômios vão ser 
 cancelados. Sendo assim, podemos escrever:
 S[n] = a.n^3 + b.n^2 + c.n + d
 O termo independente é 0, uma vez que S[0] não possui termos. 
 Portanto, d = 0.
 S[n] = a.n^3 + b.n^2 + c.n (ii)
 
 Substituindo a (ii) na (i):
 a.n^3 + b.n^2 + c.n - a.(n - 1)^3 - b.(n - 1)^2 - c.(n - 1) = n^2
 3a.n^2 - 3a.n + a + 2b.n - b + c = n^2
 3a.n^2 + (2b - 3a).n + (a - b + c) = n^2
 
 Pela identidade de polinômios, devemos ter:
 3a = 1 = a = 1/3
 2b - 3a = 0 = 2b - 1 = 0 = b = 1/2
 a - b + c = 0 = 1/3 - 1/2 + c = 0 = c = 1/6
 
 Substituindo a, b e c no polinômio (ii):
 S[n] = n^3/3 + n^2/2 + n/6
 
 Fatorando:
 S[n] = (2.n^3 + 3.n^2 + n)/6
 S[n] = [n(2n^2 + 3n + 1)]/6
 
 S[n] = [n(n + 1)(2n + 1)]/6
 
 Para o caso particular do problema apresentado, teremos:
 S[10] = (10.11.21)/6 = S[10] = 385
 
 
 Atenciosamente,
 
 Rogério Moraes de Carvalho
 
 From: [EMAIL PROTECTED] 
 [mailto:[EMAIL PROTECTED] On Behalf Of [EMAIL PROTECTED]
 Sent: quarta-feira, 19 de maio de 2004 01:21
 To: [EMAIL PROTECTED]
 Subject: [obm-l] Soma...
 
 Qual o valor de S=1^2+2^2+3^2+.+10^2?
 Usei para resolver esse problema a identidade (x+1)^3. Com efeito,
 2^3=(1+1)^3=1^3+3*1^2*1+3*1*1^2+1^1
 3^3=(2+1)^3=2^3+3*2^2*1+3*2*1^2+1
 --
 11^3=(10+1)^3=10^3+3*10^2*1+3*10*1^2+1.Isolando 
 convenientemente 3*1^2+3*2^2++3*10^2. descubro S. Minha 
 pergunta é: Existe um modo mais fácil de se achar soma de 
 quadrados perfeitos??
   Quem souber e puder responder, deixo meu agradecimento

RE: [obm-l] Soma...

2004-05-28 Por tôpico Rogério Moraes de Carvalho
Davi,

Disponha! Este é o método mais direto e elegante que eu conheço para
deduzir a fórmula do somatório das potências de expoente k, com k inteiro e
positivo, dos n primeiros inteiros positivos. Usando este método é possível
deduzir um algoritmo para calcular os coeficientes do polinômio que
representa o somatório com base no triângulo de Pascal.

Abraços,

Rogério Moraes de Carvalho

-Original Message-
From: [EMAIL PROTECTED] [mailto:[EMAIL PROTECTED] On
Behalf Of David M. Cardoso
Sent: sexta-feira, 28 de maio de 2004 20:57
To: [EMAIL PROTECTED]
Subject: RES: [obm-l] Soma...


Oi, eu entendi... muito muito muito obrigado...
Achei esse negocio muito util.. não conhecia..
Obrigado de novo..

[]'s
David

 -Mensagem original-
 De: [EMAIL PROTECTED] 
 [mailto:[EMAIL PROTECTED] Em nome de Rogério Moraes 
 de Carvalho
 Enviada em: sexta-feira, 28 de maio de 2004 16:22
 Para: [EMAIL PROTECTED]
 Assunto: RE: [obm-l] Soma...
 
 Olá David,
 
   Se você considerar S[n] como um polinômio de grau k em 
 n (k inteiro positivo), então:
 
 S[n]=a[k].n^k+a[k-1].n^(k-1)+...+a[1].n+a[0], tais que a[0], 
 a[1], ..., a[k] são os coeficientes de S[n] e a[k]!=0.
 
 S[n-1]=a[k].(n-1)^k+a[k-1].(n-1)^(k-1)+...+a[1].(n-1)+a[0]
 
 Considerando a notação C(u, v)=u!/[v!(u-v)!], com u e v 
 inteiros não negativos e u = v, e aplicando o 
 desenvolvimento do binômio de Newton nas expressões (n-1)^p, 
 com p pertencente a {1, 2, ..., k}, teremos:
 
 S[n]-S[n-1] = {a[k]-C(k,0).a[k]}.n^k +
 {a[k-1]+C(k,1).a[k]-C(k-1,0).a[k-1]}.n^(k-1) + ...
 S[n]-S[n-1] = {a[k]-a[k]}.n^k + {a[k-1]+k.a[k]-a[k-1]}.n^(k-1) + ...
 S[n]-S[n-1] = k.a[k].n^(k-1) + ...
 
 Como, por hipótese, k é inteiro positivo e a[k]!=0, então 
 k.a[k]!=0. Sendo
 assim: grau{S[n]-S[n-1]} = k-1 (i)
 Como: S[n]-S[n-1]=n^2 = grau{S[n]-S[n-1]} = 2 (ii)
 
 Por (i) e (ii): k-1 = 2 = k = 3
 
 
 Atenciosamente,
 
 Rogério Moraes de Carvalho
 -Original Message-
 From: [EMAIL PROTECTED] 
 [mailto:[EMAIL PROTECTED] On Behalf Of David M. Cardoso
 Sent: quarta-feira, 26 de maio de 2004 20:49
 To: [EMAIL PROTECTED]
 Subject: RES: [obm-l] Soma...
 
 
 Extraindo dessa mensagem essa parte:
 
  Seja S[n] o polinômio que representa a soma dos quadrados dos n 
  primeiros inteiros positivos, então podemos concluir que:
  S[n] = S[n - 1] + n^2 = S[n] - S[n - 1] = n^2 (i)
  
  Logo S[n] tem que ser um polinômio de grau 3, uma vez que 
 na diferença 
  S[n]
  - S[n - 1] os termos de maior grau dos polinômios vão ser 
 cancelados.
 
 Não entendi pq o dá pra inferir que o grau do polinomio é 3...
 Será alguem pode explicar isso?
  
 
  -Mensagem original-
  De: [EMAIL PROTECTED]
  [mailto:[EMAIL PROTECTED] Em nome de Rogério Moraes de 
  Carvalho Enviada em: quarta-feira, 19 de maio de 2004 10:25
  Para: [EMAIL PROTECTED]
  Assunto: RE: [obm-l] Soma...
  
  Olá Crom,
  
  Muitos livros de Matemática apresentam uma possível dedução da 
  fórmula da soma das potências k-ésimas (k inteiro
  positivo) dos n primeiros inteiros positivos pelo método que você 
  apresentou parcialmente, ou seja, usando o desenvolvimento 
 do binômio 
  de Newton (x + 1)^(k + 1). Ao aplicar o somatório com x 
 variando de 1 
  até n a ambos os membros da igualdade, os termos de grau (k 
 + 1) podem 
  ser cancelados, com exceção de (n + 1)^(k + 1) no primeiro 
 membro da 
  igualdade e 1^(k + 1) = 1 no segundo membro da igualdade.
  Porém, para descobrir a fórmula da soma das potências k-ésimas, nós 
  precisamos conhecer todas as fórmulas das somas das potências com 
  expoente de 1 até (k - 1). Sendo assim, nós encontramos uma 
 fórmula de 
  recorrência para deduzir a soma das potências k-ésimas dos 
 n primeiros 
  inteiros positivos, porém o processo vai ficando muito 
 longo à medida 
  que os expoentes vão crescendo.
  
  A seguir, eu apresento um método que pode ser utilizado para 
  encontrar a soma das potências k-ésimas dos n primeiros inteiros 
  positivos de forma direta. Neste método, não há a necessidade de se 
  conhecer as fórmulas das somas das potências com expoente 
 de 1 até (k 
  - 1)
  
  
  DEDUÇÃO POSSÍVEL:
  
  Seja S[n] o polinômio que representa a soma dos quadrados dos n 
  primeiros inteiros positivos, então podemos concluir que:
  S[n] = S[n - 1] + n^2 = S[n] - S[n - 1] = n^2 (i)
  
  Logo S[n] tem que ser um polinômio de grau 3, uma vez que 
 na diferença 
  S[n]
  - S[n - 1] os termos de maior grau dos polinômios vão ser 
 cancelados. 
  Sendo assim, podemos escrever:
  S[n] = a.n^3 + b.n^2 + c.n + d
  O termo independente é 0, uma vez que S[0] não possui termos. 
  Portanto, d = 0.
  S[n] = a.n^3 + b.n^2 + c.n (ii)
  
  Substituindo a (ii) na (i):
  a.n^3 + b.n^2 + c.n - a.(n - 1)^3 - b.(n - 1)^2 - c.(n - 1) = n^2
  3a.n^2 - 3a.n + a + 2b.n - b + c = n^2
  3a.n^2 + (2b - 3a).n + (a - b + c) = n^2
  
  Pela identidade de polinômios, devemos ter:
  3a = 1 = a = 1/3
  2b - 3a = 0 = 2b - 1 = 0 = b = 1/2 a - b + c = 0 = 
 1/3 - 1/2 + c

[obm-l] RE: [obm-l] Médias

2004-05-26 Por tôpico Rogério Moraes de Carvalho
RESOLUÇÃO POSSÍVEL:

Na resolução desta questão, nós precisamos supor que a velocidade com que
cada trabalhador (A, B e C) realiza um trabalho seja constante. A velocidade
com que o trabalho vai ser executado pode ser calculada pela razão entre o
trabalho (T) realizado e o tempo gasto para realizá-lo (t).

Considere como sendo T o trabalho específico citado no enunciado do
problema. Se v[X] e t[X] representam, respectivamente, a velocidade e o
tempo gasto para o trabalhador X executar o trabalho T, teremos:
v[A]=T/t[A] (unidade: (trabalho T)/dia)
v[B]=T/t[B] (unidade: (trabalho T)/dia)
v[C]=T/t[C] (unidade: (trabalho T)/dia)

Observe que a velocidade de realização do trabalho de um trabalhador
corresponde ao quanto de trabalho T que é executado a cada dia. Portanto,
para descobrir quanto de trabalho T dois trabalhadores juntos realizam a
cada dia, basta somar as quantias de trabalho T que cada um deles consegue
realizar individualmente por dia, ou seja, somar as suas velocidades de
realização do trabalho T.

Observe que:
v[X e Y]=v[X]+v[Y] = T/t[X e Y]=T/t[X]+T/t[Y] = 1/t[X]+1/t[Y]=1/t[X e Y]

Usando os dados do enunciado, teremos:
1/t[A]+1/t[B]=1/t[A e B] = 1/t[A]+1/t[B]=1/2  (i)
1/t[B]+1/t[C]=1/t[B e C] = 1/t[B]+1/t[C]=1/4  (ii)
1/t[A]+1/t[C]=1/t[A e C] = 1/t[A]+1/t[C]=5/12 (iii)

Adicionando, membro a membro, as igualdades (i), (ii) e (iii), teremos:
2.(1/t[A]+1/t[B]+1/t[C])=1/2+1/4+5/12
1/t[A]+1/t[B]+1/t[C]=[(6+3+5)/12]/2
1/t[A]+1/t[B]+1/t[C]=7/12 (iv)

 Substituindo a (ii) na (iv): 1/t[A]+1/4=7/12  = t[A]=3 dias
Substituindo a (iii) na (iv): 1/t[B]+5/12=7/12 = t[B]=6 dias
  Substituindo a (i) na (iv): 1/2+1/t[C]=7/12  = t[C]=12 dias

Resposta: Os trabalhadores A, B e C realizam o trabalho individualmente em,
respectivamente, 3, 6 e 12 dias.


Atenciosamente,

Rogério Moraes de Carvalho

From: [EMAIL PROTECTED] [mailto:[EMAIL PROTECTED] On
Behalf Of Fábio Bernardo
Sent: terça-feira, 25 de maio de 2004 22:33
To: OBM
Subject: [obm-l] Médias

A e B fazem um trabalho em 2 dias, B e C fazem o mesmo trabalho em 4 dias. A
e C fazem o mesmo trabalho em 12/5 dias. Em quantos dias eles fazem o
trabalho sozinhos.



=
Instruções para entrar na lista, sair da lista e usar a lista em
http://www.mat.puc-rio.br/~nicolau/olimp/obm-l.html
=


[obm-l] RE: [obm-l] Funçao composta

2004-05-24 Por tôpico Rogério Moraes de Carvalho
RESOLUÇÃO POSSÍVEL:

De acordo com o enunciado, a função f:A - A é tal que:
y = f(x) = x + 1, se x != 4
y = f(x) = 1, se x = 4

Uma vez que o conjunto A é finito é contém apenas 5 elementos, nós podemos
facilmente enumerar todos os pares ordenados (x, y) que satisfazem a lei de
formação de múltiplas sentenças apresentada.
f(0) = 0 + 1 = 1
f(1) = 1 + 1 = 2
f(2) = 2 + 1 = 3
f(3) = 3 + 1 = 4
f(4) = 1
f = {(0, 1), (1, 2), (2, 3), (3, 4), (4, 1)} (i)

Algumas conclusões sobre a função f:
- A função f não é injetora (ou injetiva), pois f(0) = f(4) = 1.
- A função f não é sobrejetora (ou sobrejetiva), pois Im(f) != CD(f) = A.


Analisando o conjunto de pares ordenados (i) que constituem a função f, nós
podemos resolver a equação (fofofof)(x) = 2 facilmente, como segue:

(fofofof)(x) = 2 = f((fofof)(x)) = 2 = (fofof)(x) = 1 = f((fof)(x)) = 1
= (fof)(x) = 0 ou (fof)(x) = 4 = f(f(x)) = 0 (impossível) ou f(f(x)) = 4
= f(x) = 3 = x = 2

Resposta: Alternativa c

Rogério Moraes de Carvalho
-Original Message-
From: [EMAIL PROTECTED] [mailto:[EMAIL PROTECTED] On
Behalf Of aryqueirozq
Sent: domingo, 23 de maio de 2004 21:17
To: [EMAIL PROTECTED]
Subject: [obm-l] Funçao composta






 Seja A = { 0,1, 2 ,3,4} e f ; A ®A uma função 
definida por f (x) = x + 1, se x eh diferente 4 e f( 4) 
= 1 . O número x pertence a A tal que ( f°f°f°f)(x) = 2 
eh:
a) 0b) 1  c) 2  d)3

   Agradeço.

 
__
Acabe com aquelas janelinhas que pulam na sua tela.
AntiPop-up UOL - É grátis!
http://antipopup.uol.com.br/



=
Instruções para entrar na lista, sair da lista e usar a lista em
http://www.mat.puc-rio.br/~nicolau/olimp/obm-l.html
=




=
Instruções para entrar na lista, sair da lista e usar a lista em
http://www.mat.puc-rio.br/~nicolau/olimp/obm-l.html
=


RE: [obm-l] colegio naval

2004-05-24 Por tôpico Rogério Moraes de Carvalho
Olá Leandro,

O somatório apresentado neste problema corresponde a um caso
particular da seguinte soma:
S[n] = 1/(a[1].a[2]) + 1/(a[2].a[3]) + ... + 1/(a[n-1].a[n]), com n inteiro
maior ou igual a 2 e a[1], a[2], a[3], ..., a[n] termos não nulos de uma
progressão aritmética. Um método possível de dedução da fórmula para o
cálculo de S[n] é idêntico ao que eu apresento para o caso geral da soma
deste problema em específico.

Na resolução abaixo, considere a notação abaixo.
S{k=1,k=n}(a[k]): Somatório de a índice k, com k variando de 1 até n.


RESOLUÇÃO POSSÍVEL:

Queremos encontrar a seguinte soma:
S=13/(2.4)+13/(4.6)+13/(6.8)+...+13/(50.52)
S=13.{1/[(2.1)(2.2)]+1/[(2.2)(2.3)]+1/[(2.3)(2.4)]+...+1/[(2.25)(2.26)]}

S=13.S[25] (i), onde a S[n] é dada por:

S[n]= 1/[(2.1)(2.2)]+1/[(2.2)(2.3)]+...+ 1/{(2n)[2(n+1)]}
S[n]=S{k=1,k=n}(1/{(2k)[2(k+1)]}) (ii), com n inteiro positivo.

Para decompor a fração 1/{(2k)[2(k+1)]} em frações parciais, podemos
encontrar A e B tais que:
1/{(2k)[2(k+1)]}=A/(2k)+B/[2(k+1)] (iii), com k inteiro positivo.
1/{(2k)[2(k+1)]}=[A(2k+2)+B(2k)]/{(2k)[2(k+1)]}
1/{(2k)[2(k+1)]}=[2(A+B)k+2A]/{(2k)[2(k+1)]}
1=2(A+B)k+2A, para todo k inteiro positivo.
Logo:
2A=1=A=1/2 (iv)
2(A+B)=0=B=-A=B=-1/2 (v)

Substituindo (iv) e (v) na (iii), concluímos que:
1/{(2k)[2(k+1)]}=1/(4k)-1/[4(k+1)] (vi)

Substituindo (vi) na (ii):
S[n]=S{k=1,k=n}(1/(4k)-1/[4(k+1)])
S[n]=S{k=1,k=n}(1/(4k))-S{k=1,k=n}(1/[4(k+1)])
S[n]=S{k=1,k=n}(1/(4k))-S{k+1=2,k+1=n+1}(1/[4(k+1)])
S[n]=S{k=1,k=n}(1/(4k))-S{k=2,k=n+1}(1/[4k])
S[n]=1/(4.1)+S{k=2,k=n}(1/(4k))-S{k=2,k=n}(1/[4k])-1/[4(n+1)]
S[n]=1/4-1/[4(n+1)]
S[n]=(n+1-1)/[4(n+1)]
S[n]=n/[4(n+1)] (vii)

Usando a fórmula (vii) na igualdade expressão (i), teremos:
S=13.S[25]=13.25/(4.26)=25/8

Resposta: Alternativa C


Atenciosamente,

Rogério Moraes de Carvalho
-Original Message-
From: [EMAIL PROTECTED] [mailto:[EMAIL PROTECTED] On
Behalf Of leandro-epcar
Sent: segunda-feira, 24 de maio de 2004 10:08
To: obm-l
Subject: [obm-l] colegio naval

  Alguem poderia me dar uma ideia nesta questao ,nao 
consigo achar uma sequencia ,nem mesmo calcular, esta 
questao.
   grato 
leandro



   Colegio naval   1994

  Sabendo-se que a seguinte identidade  (AX + BY)/XY =
A/Y + B/X é verdadeira para quaisquer números reais A,B,
X0,Y0,
  o valor de 13/(2*4)+ 13 /(4*6)+ 13/ ( 6*8) +...+13/
(50*52)

(A)25/16
(B)25/12
(C)25/8
(D)25/4
(E)25/2

 
__
Acabe com aquelas janelinhas que pulam na sua tela.
AntiPop-up UOL - É grátis!
http://antipopup.uol.com.br/



=
Instruções para entrar na lista, sair da lista e usar a lista em
http://www.mat.puc-rio.br/~nicolau/olimp/obm-l.html
=




=
Instruções para entrar na lista, sair da lista e usar a lista em
http://www.mat.puc-rio.br/~nicolau/olimp/obm-l.html
=


[obm-l] RE: [obm-l] Re:[obm-l] dúvida chara!

2004-05-23 Por tôpico Rogério Moraes de Carvalho
 e y, podemos encontrar x e
y em função de a e b:
a + b = (x + y) + (x - y) = a + b = 2x = x = (a + b)/2 (ii)
a - b = (x + y) - (x - y) = a - b = 2y = y = (a - b)/2 (iii)

Como x e y são naturais, então x = 0 e y = 0. Portanto:
x + y = 0 + 0 = a = 0. De acordo com a igualdade (i), a não pode ser 0,
logo a  0 (iv)
Como a.b  0 (i) e a  0 (iv), então b  0 (v)
y = 0 = -y = 0 = y = 0 e 0 = -y = y = -y = x + y = x - y =
a = b (vi)
Por (v) e (vi), concluímos que: a = b  0 (vii)

Sendo assim, devemos encontrar a e b inteiros tais que sejam satisfeitas as
seguintes condições:
a.b = 68 (ii)
a = b  0 (vii)
x = (a + b)/2 (ii) seja um número natural.
y = (a - b)/2 (iii) seja um número natural.

Analisando os divisores de 68, podemos concluir que existem apenas três
pares de valores de a e b que satisfazem as condições (ii) e (vii):
(a = 68 e b = 1) ou (a = 34 e b = 2) ou (a = 17 e b = 4)

Para a = 68 e b = 1:
x = (68 + 1)/2 = 69/2 NÃO é um número natural.
y = (68 - 1)/2 = 67/2 NÃO é um número natural.
Portanto, x = 69/2 e y = 67/2 NÃO é uma solução possível.

Para a = 34 e b = 2:
x = (34 + 2)/2 = 18 é um número natural.
y = (34 - 2)/2 = 16 é um número natural.
Portanto, x = 18 e y = 16 é uma solução possível.

Para a = 17 e b = 4:
x = (17 + 4)/2 = 21/2 NÃO é um número natural.
y = (17 - 4)/2 = 13/2 NÃO é um número natural.
Portanto, x = 21/2 e y = 13/2 NÃO é uma solução possível.

Único valor possível para (x + y)^2:
(x + y)^2 = (18 + 16)^2 = 34^2 = 1156

Resposta: Alternativa c



Atenciosamente,

Rogério Moraes de Carvalho
-Original Message-
From: [EMAIL PROTECTED] [mailto:[EMAIL PROTECTED] On
Behalf Of Osvaldo
Sent: domingo, 23 de maio de 2004 01:01
To: obm-l
Subject: [obm-l] Re:[obm-l] dúvida chara!

sejam x e y tais numeros, dai temos que
x^2-y^2=27

(x+y)(x-y)=27


a=x+y
b=x-y

Possiveis valores para a e b (x,y):

{(1,27),(3,9),(9,3),(27,1)}

Assim (x+y)^2=a^2

Temos então que todos os valores de (x+y)^2 pertencem a
{1, 9, 81, 729)

Logo um dos valores possiveis é 729
resposta c






 1)a diferença entre os quadrados de dois números 
naturais é 27.UM dos possíveis valores do quadrado da 
soma desses dois números:
 
 a)529
 b)625
 c)729
 d)841
 

Atenciosamente,

Engenharia Elétrica - UNESP Ilha Solteira
Osvaldo Mello Sponquiado 
Usuário de GNU/Linux


 
__
Acabe com aquelas janelinhas que pulam na sua tela.
AntiPop-up UOL - É grátis!
http://antipopup.uol.com.br/



=
Instruções para entrar na lista, sair da lista e usar a lista em
http://www.mat.puc-rio.br/~nicolau/olimp/obm-l.html
=



=
Instruções para entrar na lista, sair da lista e usar a lista em
http://www.mat.puc-rio.br/~nicolau/olimp/obm-l.html
=


[obm-l] RE: [obm-l] Múltiplo

2004-05-23 Por tôpico Rogério Moraes de Carvalho
Olá Júnior,

Segue uma resolução possível para o problema.

RESOLUÇÃO POSSÍVEL:

O número total de possibilidades de formar números de 3 algarismos distintos
com os dígitos 1, 2, 3, 6 e 7 é dado por:
#S = A(5, 3) = 5.4.3 = 60 (espaço amostral)

De acordo com o critério de divisibilidade por 3, um número inteiro é
divisível por 3 se e somente se a soma de seus algarismos é divisível por 3.
Sendo assim, os números formados somente serão múltiplos de 3 se a soma dos
algarismos for igual a 3. Podemos encontrar C(5, 3) = 10 combinações
possíveis de 5 algarismos distintos tomados 3 a 3, listadas a seguir:
 1) 1, 2, 3: soma = 6  (é múltiplo de 3)
 2) 1, 2, 6: soma = 9  (é múltiplo de 3)
 3) 1, 2, 7: soma = 10 (NÃO é múltiplo de 3)
 4) 1, 3, 6: soma = 10 (NÃO é múltiplo de 3)
 5) 1, 3, 7: soma = 11 (NÃO é múltiplo de 3)
 6) 1, 6, 7: soma = 14 (NÃO é múltiplo de 3)
 7) 2, 3, 6: soma = 11 (NÃO é múltiplo de 3)
 8) 2, 3, 7: soma = 12 (é múltiplo de 3)
 9) 2, 6, 7: soma = 15 (é múltiplo de 3)
10) 3, 6, 7: soma = 16 (NÃO é múltiplo de 3)

Para cada grupo de 3 algarismos cuja soma é igual a um múltiplo de 3 (4
grupos), podemos formar P(3) números com 3 algarismos diferentes:
#A = 4.P(3) = 4.3! = 4.6 = 24 (evento)

Num espaço com distribuição equiprovável, a probabilidade de o evento
ocorrer será dada por:
p = #A/#S = 24/60 = 2/5 = 40%

Resposta: 2/5 = 40%


Atenciosamente,

Rogério Moraes de Carvalho
-Original Message-
From: [EMAIL PROTECTED] [mailto:[EMAIL PROTECTED] On
Behalf Of Junior
Sent: sexta-feira, 21 de maio de 2004 08:36
To: [EMAIL PROTECTED]
Subject: [obm-l] Múltiplo

Dado o problema:

São formados numeros de 3 algarismos distintos usando-se os dígitos 1,2,3,6
e 7

Depois de formados, um desses numeros (de 3 algarismos) foi sorteado. Qual a
probabilidade dele ser um multiplo de 3?

-   Encontrei o espaço amostral A(5,3)
- Mas não consigo observar como ser multiplo de 3?

Alguem poderia me ajudar.

Desde ja agradeço.
=
Instruções para entrar na lista, sair da lista e usar a lista em
http://www.mat.puc-rio.br/~nicolau/olimp/obm-l.html
=




=
Instruções para entrar na lista, sair da lista e usar a lista em
http://www.mat.puc-rio.br/~nicolau/olimp/obm-l.html
=


RE: [obm-l]

2004-05-23 Por tôpico Rogério Moraes de Carvalho
Olá André,

Segue uma resolução possível para este problema.


RESOLUÇÃO POSSÍVEL:

Como o problema não informa sobre o sinal dos números, eu estarei
calculando o total de números positivos ou negativos que satisfazem o
enunciado. Para isto, basta calcular a quantidade (n) de múltiplos positivos
de 1 de 10 algarismos e, ao final, multiplicar esta quantidade por 2
(2n) para contar os múltiplos negativos também.

Para calcular a quantidade de múltiplos positivos e distintos de
1 com 10 algarismos, devemos encontrar o menor e o maior inteiros
positivos múltiplos de 1.

Cálculo do menor inteiro positivo com 10 algarismos e múltiplo de 1
---
Fazendo a divisão euclidiana do menor positivo de 10 algarismos (10)
por 1, encontramos:
10 = 9.1 + 1 = 10 - 1 = 9.1 =
10 - 1 + 1 = 9.1 + 1 =
10 = 90001.1 (i)
Logo, 10 é o menor inteiro positivo com 10 algarismos e múltiplo de
1.

Cálculo do maior inteiro positivo com 10 algarismos e múltiplo de 1
---
Fazendo a divisão euclidiana do maior positivo de 10 algarismos (99)
por 1, encontramos:
99 = 99.1 (ii)
Logo, 99 é o maior inteiro positivo com 10 algarismos e múltiplo de
1.

Cálculo da quantidade de inteiros positivos com 10 algarismos e múltiplos de
1
-
Por (i) e (ii), podemos concluir que a seqüência de inteiros positivos com
10 algarismos e múltiplos de 1 é dada por:
(10, 10 + 1, 10 + 2.1, ..., 99)

A seqüência acima é uma P.A. de primeiro termo a[1] = 10, último
termo a[n] = 99 e razão R = 1. Pela fórmula do termo geral de
uma P.A. podemos encontrar o número (n) de termos:
a[n] = a[1] + (n - 1).R = 99 = 10 + (n - 1).1
Substituindo as igualdades (i) e (ii), teremos:
99.1 = 90001.1 + (n - 1).1
99 = 90001 + n - 1
n = 99 - 9
n = 810009

Total de números positivos e negativos: 2n = 2.810009 = 1620018

Resposta: 1.620.018 números inteiros distintos, positivos ou negativos, com
10 algarismos são múltiplos de 1.


Atenciosamente,

Rogério Moraes de Carvalho

From: [EMAIL PROTECTED] [mailto:[EMAIL PROTECTED] On
Behalf Of Andre
Sent: sexta-feira, 21 de maio de 2004 00:16
To: [EMAIL PROTECTED]
Subject: [obm-l] 

Olá,
 
    Questão:  Quantos são os números com 10 algarismo diferentes entre
si e divisível por 1.
  Dizer que eles estão incluídos entre os números
interessantes está correto?



=
Instruções para entrar na lista, sair da lista e usar a lista em
http://www.mat.puc-rio.br/~nicolau/olimp/obm-l.html
=


RE: [obm-l] %

2004-05-23 Por tôpico Rogério Moraes de Carvalho
RESOLUÇÃO POSSÍVEL:

Seja x o preço inicial da mercadoria, então teremos:
Após o aumento de 20%: (100% + 20%).x = 1,2.x
Após o aumento de 12%: (100% + 12%).1,2.x = 1,12.1,2.x = 1,344.x

1,344.x = (100% + 34,4%).x (aumento de 34,4% nos dois meses)

Resposta: Alternativa C (34,4%)


Atenciosamente,

Rogério Moraes de Carvalho

-Original Message-
From: [EMAIL PROTECTED] [mailto:[EMAIL PROTECTED] On
Behalf Of elton francisco ferreira
Sent: domingo, 23 de maio de 2004 14:08
To: [EMAIL PROTECTED]
Subject: [obm-l] %

O valor de uma mercadoria em reais, subiu 20% no mês e
12% no mês seguinte. No intervalo desses dois meses, a
mercadoria subiu:

(A) 32%
(B) 32.4%
(C) 34.4%
(D) 34%
(E) 33% 



=
Instruções para entrar na lista, sair da lista e usar a lista em
http://www.mat.puc-rio.br/~nicolau/olimp/obm-l.html
=


[obm-l] RE: [obm-l] Re:[obm-l] RE: [obm-l] Re:[obm-l] dúvida chara!

2004-05-23 Por tôpico Rogério Moraes de Carvalho
Olá Osvaldo,

Não há a necessidade de formalidades, mesmo porque eu não sou Dr..
Eu gostaria de ressaltar o seguinte comentário que eu coloquei no início dos
meus comentários, caso não tenha ficado claro: A análise que eu apresento a
seguir corresponde a uma crítica de CARÁTER CONSTRUTIVO com relação à
resolução apresentada pelo Osvaldo. O objetivo desta análise não é depreciar
a resolução do Osvaldo, mas sim de mostrar que é necessário sermos rigorosos
nas resoluções de problemas de Matemática para não chegarmos a resultados 
incorretos. Muitas vezes podemos encontrar uma resposta correta para uma
questão resolvendo-a de maneira errada.

É importante ficar claro que mesmo para o conjunto dos números
inteiros a sua solução está incompleta. Observe que você não verificou se os
valores de a e b encontrados produzem valores inteiros de x e y.
Neste caso, você não utilizaria a condição a = b  0 e encontraria todos os
valores inteiros de a e b que reproduzem o produto. Veja o exemplo da
questão modificada para constatar que a sua resolução apresentará resultados
inválidos mesmo no conjunto dos números inteiros.

Atenciosamente,

Rogério Moraes de Carvalho
-Original Message-
From: [EMAIL PROTECTED] [mailto:[EMAIL PROTECTED] On
Behalf Of Osvaldo
Sent: domingo, 23 de maio de 2004 17:54
To: obm-l
Subject: [obm-l] Re:[obm-l] RE: [obm-l] Re:[obm-l] dúvida chara!

Desculpe-me se fui parcial Dr., porém equivoquei-me ao 
ler o enunciado da questão. Eu apenas fiz os calculos 
para os números inteiros e não naturais, ou seja, 
inclui algumas possibilidades a mais. 
Obrigado pela observação!





 Olá colegas da lista,
 
   Apesar da resolução apresentada pelo Osvaldo 
ter seguido um possível
 raciocínio correto para resolver esta questão, a 
análise dele está
 incompleta porque omite alguns passos muito 
importantes, o que pode nos
 levar a encontrar soluções inválidas. Neste problema 
especificamente, a
 resposta encontrada está correta, porém, se 
modificarmos o valor da
 diferença de quadrados de 27 para outro valor, então 
a resolução dele pode
 nos levar a resultados errados.
 
   A análise que eu apresento a seguir 
corresponde a uma crítica de
 caráter construtivo com relação à resolução 
apresentada pelo Osvaldo. O
 objetivo desta análise não é depreciar a resolução 
do Osvaldo, mas sim de
 mostrar que é necessário sermos rigorosos nas 
resoluções de problemas de
 Matemática para não chegarmos a resultados 
incorretos. Muitas vezes podemos
 encontrar uma resposta correta para uma questão 
resolvendo-a de maneira
 errada.
 
   Na resolução apresentada abaixo, considere 
que = significa
 implica e = significa maior ou igual a.
 
 
 QUESTÃO ORIGINAL:
 
 A diferença entre os quadrados de dois números 
naturais é 27. UM dos
 possíveis valores do quadrado da soma desses dois 
números:
 a)529
 b)625
 c)729
 d)841
 
 
 RESOLUÇÃO POSSÍVEL:
 
 Sejam x e y os dois números naturais, então devemos 
ter:
 x^2 - y^2 = 27 = (x + y)(x - y) = 27
 
 Adotando a = x + y e b = x - y, teremos:
 a.b = 27 (i) (Observe que o produto de a e b é 
positivo)
 Resolvendo o sistema de equações nas variáveis x e 
y, podemos encontrar x e
 y em função de a e b:
 a + b = (x + y) + (x - y) = a + b = 2x = x = (a 
+ b)/2 (ii)
 a - b = (x + y) - (x - y) = a - b = 2y = y = (a -
 b)/2 (iii)
 
 Como x e y são naturais, então x = 0 e y = 0. 
Portanto:
 x + y = 0 + 0 = a = 0. De acordo com a igualdade 
(i), a não pode ser 0,
 logo a  0 (iv)
 Como a.b  0 (i) e a  0 (iv), então b  0 (v)
 y = 0 = -y = 0 = y = 0 e 0 = -y = y = -y = 
x + y = x - y =
 a = b (vi)
 Por (v) e (vi), concluímos que: a = b  0 (vii)
 
 Sendo assim, devemos encontrar a e b inteiros tais 
que sejam satisfeitas as
 seguintes condições:
 a.b = 27 (ii)
 a = b  0 (vii)
 x = (a + b)/2 (ii) seja um número natural.
 y = (a - b)/2 (iii) seja um número natural.
 
 Analisando os divisores de 27, podemos concluir que 
existem apenas dois
 pares de valores de a e b que satisfazem as 
condições (ii) e (vii):
 (a = 27 e b = 1) ou (a = 9 e b = 3)
 
 Para a = 27 e b = 1:
 x = (27 + 1)/2 = 14 é um número natural.
 y = (27 - 1)/2 = 13 é um número natural.
 Portanto, x = 14 e y = 13 é uma solução possível.
 
 Para a = 9 e b = 3:
 x = (9 + 3)/2 = 6 é um número natural.
 y = (9 - 3)/2 = 3 é um número natural.
 Portanto, x = 6 e y = 3 é uma solução possível.
 
 Possíveis valores para (x + y)^2:
 (x + y)^2 = (14 + 13)^2 = 27^2 = 729
 (x + y)^2 = (6 + 3)^2 = 9^2 = 81
 
 Resposta: Alternativa c
 
 
 Observação: Pode parecer que os passos apresentados 
para deduzir as
 condições são desnecessários, mas são eles que 
garantem a validade das
 soluções encontradas.
 
 
 EXPLICAÇÃO DO MOTIVO DA RESOLUÇÃO APRESENTADA PELO 
OSVALDO SER INCOMPLETA:
 
 Na resolução são apresentados 4 valores possíveis 
para a e b (a,b):
 {(1,27),(3,9),(9,3),(27,1)}. Porém, (1,27) e (3,9) 
não satisfazem a condição
 (vii): a = b  0. Portanto, somente os pares (9,3) 
e (27,1) correspondem a
 possíveis

RE: [obm-l] colegio naval

2004-05-22 Por tôpico Rogério Moraes de Carvalho
Olá Leandro,

Apesar dos seus cuidados, acredito que você novamente cometeu um
erro de transcrição do enunciado da questão. Pelo menos nas alternativas,
uma vez que a alternativa A não faz sentido. Se o enunciado estiver correto,
então não existem valores de P e M que satisfazem todos os dados
apresentados. Vamos a uma resolução possível para este problema.


RESOLUÇÃO POSSÍVEL:

2MX - X + 5 = 3PX - 2M + P
(2M - 1 - 3P)X = P - 2M - 5

Para (2M - 1 - 3P) != 0, podemos concluir que:
- A equação é do primeiro grau na variável X.
- A equação admite uma única raiz dada por:
  X = (P - 2M - 5)/(2M - 1 - 3P)
  O que não satisfaz o enunciado do problema que apresenta duas raízes
distintas para a equação. Logo: (2M - 1 - 3P) = 0.


Para (2M - 1 - 3P) = 0 (i), podemos concluir que:
- A equação não é do primeiro grau, pois o coeficiente do termo X é nulo.
  0.X = P - 2M - 5 = P - 2M = 5 (ii)
- Resolvendo o sistema das equações (i) e (ii):
  -3P + 2M = 1 (i)
  P - 2M = 5 (ii)
  Adicionando, membro a membro, as equações (i) e (ii), teremos:
  -2P = 6 = P = -3
  Substituindo P na (ii): -3 - 2M = 5 = -2M = 8 = M = -4
- Observe que neste caso a equação se reduz a: 0.X = 0. Esta igualdade é
verdadeira para qualquer que seja o X complexo. Neste caso, existem
infinitas raízes complexas, inclusive 2^1/3 + 3^1/2 e 3^1/3 + 2^1/2. Porém,
a equação não é do primeiro grau, pois o coeficiente de X é nulo.

Analisando as alternativas:
(A) P^2 + M^2 = (-3)^2 + (-4)^2 = 25
(B) P.M = (-3).(-4) = 12
(C) M^P = (-4)^(-3) = 1/(-4)^3 = -1/64
(D) P^M = (-3)^(-4) = 1/(-3)^4 = 1/81
(E) P/M = (-3)/(-4) = 3/4


Resposta:

Rigorosamente, esta questão deveria ser anulada. Observe que, apesar de que
para P = -3 e M = -4 a equação admite qualquer número complexo como raiz,
portanto inclusive as duas raízes apresentadas no enunciado, ela não será de
primeiro grau como também é afirmado no enunciado. Logo, não existem valores
de P e M para os quais todas as afirmações do enunciado sejam verdadeiras.

Eu acredito que o item (A) desta questão deveria ser: P^2 + M^2 = 25. Se
for, ficará claro que a intenção do autor desta questão era de que a
alternativa (A) fosse a resposta. Porém, o autor da questão foi infeliz ao
informar: Sabe-se que a equação do primeiro grau na variável 'X': ...,
pois uma equação de primeiro grau admite uma única raiz complexa. Se o
início do enunciado fosse modificado para Sabe-se que a equação na variável
'X': ..., então poderíamos concluir corretamente que P^2 + M^2 = 25.


Atenciosamente,

Rogério Moraes de Carvalho
-Original Message-
From: [EMAIL PROTECTED] [mailto:[EMAIL PROTECTED] On
Behalf Of leandro-epcar
Sent: quarta-feira, 19 de maio de 2004 18:13
To: obm-l
Subject: [obm-l] colegio naval

colegio naval 93 

 Sabe-se que a equação do primeiro grau na variável 'X'
:2MX-X+5=3PX-2M+P admite as raízes   2^1/3 + 3^1/2  
e3^1/3 +  2^1/2.entre os parametros M e P vale a 
relação


(A) P^2 + M ^2
(B) PM = 6
(C) M^P=64
(D) P^M=32
(E) P/M=3/5

===
   Desta vez tomei cuidado em passar as questoes .
===
  Eu não estou compreendendo como uma equação do 
primeiro grau tem duas raízes .Se alguen souber 
pode por favor me explicar ou caso tenha uma 
incoerencia no enunciado desconsiderem a equação e me 
desculpem.
  Usando o teorema dos polinômios iguais teremos que a  

  2MX-X+5=3PX-2m+P  podemos transformar num sistema 
|=
|(2M-1)X + 5=0 
|(3P)X-2M+p=0 
|===
 teremos que 2M-1=3P  e  5=P-2M
  P = -3   e  M =-4
=
  e substituindo os valores de M e P na equação não 
aparece as raízes do enunciado.

  Agradeço desde já 
   LEANDRO GERALDO DA COSTA















   colegio naval 93
 
 Considere a equaçâo do primeiro grau em X : M^2X^3=M+9X
pode-se afirmar que a equação tem conjunto verdade 
unitário se:

(A) m=3
(B) m=-3
(C) m diferente de -3
(D) m diferente de 3
(E) m difernte de 3 e de -3

 
__
Acabe com aquelas janelinhas que pulam na sua tela.
AntiPop-up UOL - É grátis!
http://antipopup.uol.com.br/



=
Instruções para entrar na lista, sair da lista e usar a lista em
http://www.mat.puc-rio.br/~nicolau/olimp/obm-l.html
=




=
Instruções para entrar na lista, sair da lista e usar a lista em
http://www.mat.puc-rio.br/~nicolau/olimp/obm-l.html
=


[obm-l] RE: [obm-l] Nº Complexo

2004-05-22 Por tôpico Rogério Moraes de Carvalho
Olá Maurizio,

Eu acredito que você não tenha transcrito o enunciado corretamente
uma vez que estão faltando os parênteses no expoente da potência de base 2.
Observe que:
2^1/2/(1+i) = (2^1)/2/(1+i) = (2/2)/(1+i) = 1/(1+i)


Seguem duas resoluções possíveis para calcular o resultado da expressão:
[2^(1/2)/(1+i)]^93

Na resolução, eu estarei adotando a nomenclatura sqr(x) para representar a
raiz quadrada de x.

PRIMEIRA RESOLUÇÃO POSSÍVEL (Forma algébrica):
[sqr(2)/(1+i)]^93 =
[sqr(2)/(1+i)].[sqr(2)/(1+i)]^92 =
{[sqr(2).(1-i)]/[(1+i)(1-i)]}.[sqr(2)^2/(1+i)^2]^46 =
{sqr(2)/2-[sqr(2)/2].i}.[2/(2i)]^46 = 
{sqr(2)/2-[sqr(2)/2].i}.(1/i^46) =
Como i^46 = i^44.i^2 = 1.(-1) = -1, teremos:
{sqr(2)/2-[sqr(2)/2].i}.(-1) =
-sqr(2)/2+[sqr(2)/2].i

SEGUNDA RESOLUÇÃO POSSÍVEL (Forma trigonométrica):
[sqr(2)/(1+i)]^93 =
{[sqr(2).(1-i)]/[(1+i)(1-i)]}^93 = 
{sqr(2)/2-[sqr(2)/2].i}^93 = 
[cos(7pi/4)+i.sen(7pi/4)]^93 =
cos(93.7pi/4)+i.sen(93.7pi/4) =
cos(3pi/4+81.2pi)+i.sen(3pi/4+81.2pi) =
-sqr(2)/2+[sqr(2)/2].i

Resposta: [2^(1/2)/(1+i)]^93 = -sqr(2)/2+[sqr(2)/2].i


Atenciosamente,

Rogério Moraes de Carvalho
-Original Message-
From: [EMAIL PROTECTED] [mailto:[EMAIL PROTECTED] On
Behalf Of Maurizio
Sent: sábado, 22 de maio de 2004 12:45
To: [EMAIL PROTECTED]
Subject: [obm-l] Nº Complexo

Tou tentando resolver este aqui, que não é diferente dos outros que resolvi
de complexo mas não está dando certo:

Obrigado
MauZ

[2^1/2/(1+i)]^93

=
Instruções para entrar na lista, sair da lista e usar a lista em
http://www.mat.puc-rio.br/~nicolau/olimp/obm-l.html
=




=
Instruções para entrar na lista, sair da lista e usar a lista em
http://www.mat.puc-rio.br/~nicolau/olimp/obm-l.html
=


[obm-l] RE: [obm-l] aritimética

2004-05-21 Por tôpico Rogério Moraes de Carvalho
Que número aumenta em 2475 unidades quando
acrecenta-se dois zeros a sua direita?

POSSÍVEL RESOLUÇÃO:

Seja x o número procurado, então:
100x = x + 2475 = 99x = 2475 = x = 25

Resposta: 25 (Observe que 2500 = 25 + 2475)



Quais os possíveis restos da divisão de um número por 8?

POSSÍVEL RESOLUÇÃO:

Se a e b são dois inteiros, com b != 0, então existem e são únicos os
inteiros q e r tais que:
a = b.q + r e 0 = r  |b|

Portanto, numa divisão por 8 teremos: 0 = r  8

Resposta: r pertence ao conjunto {0, 1, 2, 3, 4, 5, 6, 7}.


Atenciosamente,

Rogério Moraes de Carvalho
-Original Message-
From: [EMAIL PROTECTED] [mailto:[EMAIL PROTECTED] On
Behalf Of elton francisco ferreira
Sent: quinta-feira, 20 de maio de 2004 23:48
To: [EMAIL PROTECTED]
Subject: [obm-l] aritimética

Que número aumenta em 2475 unidades quando
acrecenta-se dois zeros a sua direita?




Quais os possíveis restos da divisão de um número por 8?

__

Yahoo! Messenger - Fale com seus amigos online. Instale agora! 
http://br.download.yahoo.com/messenger/
=
Instruções para entrar na lista, sair da lista e usar a lista em
http://www.mat.puc-rio.br/~nicolau/olimp/obm-l.html
=




=
Instruções para entrar na lista, sair da lista e usar a lista em
http://www.mat.puc-rio.br/~nicolau/olimp/obm-l.html
=


[obm-l] RE: [obm-l] Exercício UFRJ

2004-05-21 Por tôpico Rogério Moraes de Carvalho
RESOLUÇÃO POSSÍVEL:

Uma vez que a velocidade do vento tem a mesma direção que a da velocidade do
avião e considerando que o sentido da velocidade do vento não seja
modificado, então o avião se deslocará no mesmo sentido do vento em metade
do percurso de ida e volta e no sentido contrário na outra metade.
Considerando T1 e T2 os tempos gastos nas duas metades do percurso de ida e
volta, lembrando que no Movimento Retilíneo e Uniforme (M.R.U.) v = d/T = d
= T/v, onde v representa o módulo da velocidade vetorial, d o módulo do
deslocamento vetorial e T o intervalo de tempo,teremos:

Ttotal = T1 + T2 (Ttotal = 4 h - autonomia de vôo)
4 = d/(300 + v) + d/(300 - v)
4 = d[1/(300 + v) + 1/(300 - v)]
4 = d(300 - v + 300 + v)/[(300 + v)(300 - v)]
4 = d(600)/(300^2 - v^2)
d = 4(300.300 - v^2)/600
d = 600 - v^2/150 (d em função de v)

Como d é uma função quadrática de v e o coeficiente de v^2 é negativo
(-1/150), podemos concluir que a função admite valor máximo, onde o ponto de
máximo é dado por: v = -b/(2a) = v = 0

Respostas:
a) d = 600 - v^2/150 (km, km/h)
b) A distância é máxima (600 km) quando não há vento (v = 0)

Rogério Moraes de Carvalho

From: [EMAIL PROTECTED] [mailto:[EMAIL PROTECTED] On
Behalf Of Alan Pellejero
Sent: sexta-feira, 21 de maio de 2004 12:26
To: [EMAIL PROTECTED]; [EMAIL PROTECTED]
Subject: [obm-l] Exercício UFRJ

Pessoal, gostaria de uma ajudinha nesse aqui:
 
1-)Um avião tem combustível para voar durante quatro horas.  Na presença de
um vento com a velocidade v km/h na direção e sentido do movimento, a
velocidade do avião é (300+v)km/h. Se o avião se desloca em sentido
contrário ao vento, sua velocidade é de (300-v)km/h.
Suponha que o avião se afaste a uma distância d do aeroporto e retorne ao
ponto de partida, consumindo todo o combustível,n e que durante todo o
trajeto a velocidade do vento seja constante e tenha a mesma direção que a
do movimento do avião.
a) Determine d como função de v;
b) Determine para que valor de v a distância d é máxima.
[]'s 
Alan Pellejero



Yahoo! Messenger - Fale com seus amigos online. Instale agora!



=
Instruções para entrar na lista, sair da lista e usar a lista em
http://www.mat.puc-rio.br/~nicolau/olimp/obm-l.html
=


[obm-l] RE: [obm-l] Exercício UFRJ

2004-05-21 Por tôpico Rogério Moraes de Carvalho
Olá Daniel,

Você está confundindo os conceitos de direção e sentido. Retas
paralelas compartilham da mesma direção e em cada direção há dois sentidos.

Como o problema não fala em mudança de SENTIDO na velocidade do
vento, então devemos considerar que o sentido da velocidade do vento é
mantida nos movimentos de ida e volta do avião.

Veja uma resolução possível para este problema numa mensagem
anterior que eu enviei.

Abraços,

Rogério Moraes de Carvalho

-Original Message-
From: [EMAIL PROTECTED] [mailto:[EMAIL PROTECTED] On
Behalf Of Daniel Silva Braz
Sent: sexta-feira, 21 de maio de 2004 13:37
To: [EMAIL PROTECTED]
Subject: Re: [obm-l] Exercício UFRJ

 --- Alan Pellejero [EMAIL PROTECTED]
escreveu:  Pessoal, gostaria de uma ajudinha nesse
aqui:
 1-)Um avião tem combustível para voar durante quatro
 horas.  Na presença de um vento com a velocidade v
 km/h na direção e sentido do movimento, a velocidade
 do avião é (300+v)km/h. Se o avião se desloca em
 sentido contrário ao vento, sua velocidade é de
 (300-v)km/h.
 Suponha que o avião se afaste a uma distância d do
 aeroporto e retorne ao ponto de partida, consumindo
 todo o combustível,n e que durante todo o trajeto a
 velocidade do vento seja constante e tenha a mesma
 direção que a do movimento do avião.

 a) Determine d como função de v;

considerando o tempo de voo como 4h
f(v) = 4(300+v)

 b) Determine para que valor de v a distância d é
 máxima.

Não existe máximo..qto mais rápido o vento mais longe
vc irá (já que o vento está sempre na direção do
movimento). Note apenas que você só pode usar 2h de
vôo para ir..pq precisará das outras 2h para voltar..

Alan, não estou muito certo se isso está correto..e
estou sem tempo de pensar um pouco mais..
mas..uma pergunta...e se o vento se mantivesse sempre
na mesma direção..ou seja..a favor na ida e contra na
volta..o q mudaria?

Daniel S. Braz


__

Yahoo! Messenger - Fale com seus amigos online. Instale agora! 
http://br.download.yahoo.com/messenger/
=
Instruções para entrar na lista, sair da lista e usar a lista em
http://www.mat.puc-rio.br/~nicolau/olimp/obm-l.html
=




=
Instruções para entrar na lista, sair da lista e usar a lista em
http://www.mat.puc-rio.br/~nicolau/olimp/obm-l.html
=


RE: [obm-l] Exponencial

2004-05-19 Por tôpico Rogério Moraes de Carvalho
Morgado,

Em grande parte das vezes é mais difícil decifrar o enunciado da
questão do que a própria questão. :)

Neste caso, eu somente consegui decifrar o enunciado porque já tinha
resolvido esta questão.

Abraços,

Rogério Moraes de Carvalho
-Original Message-
From: [EMAIL PROTECTED] [mailto:[EMAIL PROTECTED] On
Behalf Of Augusto Cesar de Oliveira Morgado
Sent: quarta-feira, 19 de maio de 2004 07:01
To: [EMAIL PROTECTED]
Subject: RE: [obm-l] Exponencial

Rogério, o que voce eh? egiptologo?
Parabens por ter decifrado.

==
Mensagem  enviada  pelo  CIP  WebMAIL  - Nova Geração - v. 2.1
CentroIn Internet Provider  http://www.centroin.com.br
Tel: (21) 2542-4849, (21) 2295-3331Fax: (21) 2295-2978
Empresa 100% Brasileira - Desde 1992 prestando servicos online


-- Original Message ---
From: Rogério Moraes de Carvalho [EMAIL PROTECTED]
To: [EMAIL PROTECTED]
Sent: Wed, 19 May 2004 01:54:32 -0300
Subject: RE: [obm-l] Exponencial

 Olá Fábio,
 
   Ficou muito difícil entender a questão com esta explicação da
 notação no meio do enunciado. De qualquer modo, eu já havia 
 resolvido esta questão anteriormente.
 
   Segue o enunciado e uma resolução possível.
 
 ENUNCIADO:
 Resolva no campo dos reais a seguinte equação exponencial:
 3^(x^2 + 1/x^2) = 81/3^(x + 1/x)
 
 RESOLUÇÃO:
 Condição de existência: x != 0
 
 Fazendo y = x + 1/x, teremos:
 y^2 = (x + 1/x)^2 = y^2 = x^2 + 2 + 1/x^2 = x^2 + 1/x^2 = y^2 - 2
 
 Portanto, representando a equação exponencial em função de y,
  teremos: 3^(y^2 - 2) = 3^4/3^y = 3^(y^2 - 2) = 3^(4 - y) = y^2 -
  2 = 4 - y = y^2 + y - 6 = 0 = y = -3 ou y = 2
 
 Para y = -3:
 x + 1/x = -3 = x^2 + 3x + 1 = 0 = x = [-3-sqr(5)]/2 ou x = [-
 3+sqr(5)]/2
 
 Para y = 2:
 x + 1/x = 2 = x^2 - 2x + 1 = 0 = (x - 1)^2 = 0 = x = 1
 
 Todas as soluções satisfazem a condição de existência.
 
 Resposta: S = {[-3-sqr(5)]/2, [-3+sqr(5)]/2, 1}
 
 Atenciosamente,
 
 Rogério Moraes de Carvalho
 
 From: [EMAIL PROTECTED] [mailto:[EMAIL PROTECTED] 
 On Behalf Of Fabio Contreiras Sent: sexta-feira, 14 de maio de 2004 23:00
 To: [EMAIL PROTECTED]
 Subject: [obm-l] Exponencial
 
 Tentei sair dessa equação mas naum deu em nada... alguem tem o bizu 
 aih hehe , Abraços! Fabio     3^x^2 ( 3 elevado à x ao quadrado ) + 
 1 / x^2 = { 81 / 3^[(x+1/x)] }       Valeu desde já!
 
 =
 Instruções para entrar na lista, sair da lista e usar a lista em
 http://www.mat.puc-rio.br/~nicolau/olimp/obm-l.html
 =
--- End of Original Message ---

=
Instruções para entrar na lista, sair da lista e usar a lista em
http://www.mat.puc-rio.br/~nicolau/olimp/obm-l.html
=



=
Instruções para entrar na lista, sair da lista e usar a lista em
http://www.mat.puc-rio.br/~nicolau/olimp/obm-l.html
=


RE: [obm-l] Soma...

2004-05-19 Por tôpico Rogério Moraes de Carvalho
Olá Crom,

Muitos livros de Matemática apresentam uma possível dedução da
fórmula da soma das potências k-ésimas (k inteiro positivo) dos n primeiros
inteiros positivos pelo método que você apresentou parcialmente, ou seja,
usando o desenvolvimento do binômio de Newton (x + 1)^(k + 1). Ao aplicar o
somatório com x variando de 1 até n a ambos os membros da igualdade, os
termos de grau (k + 1) podem ser cancelados, com exceção de (n + 1)^(k + 1)
no primeiro membro da igualdade e 1^(k + 1) = 1 no segundo membro da
igualdade. Porém, para descobrir a fórmula da soma das potências k-ésimas,
nós precisamos conhecer todas as fórmulas das somas das potências com
expoente de 1 até (k - 1). Sendo assim, nós encontramos uma fórmula de
recorrência para deduzir a soma das potências k-ésimas dos n primeiros
inteiros positivos, porém o processo vai ficando muito longo à medida que os
expoentes vão crescendo.

A seguir, eu apresento um método que pode ser utilizado para
encontrar a soma das potências k-ésimas dos n primeiros inteiros positivos
de forma direta. Neste método, não há a necessidade de se conhecer as
fórmulas das somas das potências com expoente de 1 até (k - 1)


DEDUÇÃO POSSÍVEL:

Seja S[n] o polinômio que representa a soma dos quadrados dos n
primeiros inteiros positivos, então podemos concluir que:
S[n] = S[n - 1] + n^2 = S[n] - S[n - 1] = n^2 (i)

Logo S[n] tem que ser um polinômio de grau 3, uma vez que na diferença S[n]
- S[n - 1] os termos de maior grau dos polinômios vão ser cancelados. Sendo
assim, podemos escrever:
S[n] = a.n^3 + b.n^2 + c.n + d
O termo independente é 0, uma vez que S[0] não possui termos. Portanto, d =
0.
S[n] = a.n^3 + b.n^2 + c.n (ii)

Substituindo a (ii) na (i):
a.n^3 + b.n^2 + c.n - a.(n - 1)^3 - b.(n - 1)^2 - c.(n - 1) = n^2
3a.n^2 - 3a.n + a + 2b.n - b + c = n^2
3a.n^2 + (2b - 3a).n + (a - b + c) = n^2

Pela identidade de polinômios, devemos ter:
3a = 1 = a = 1/3
2b - 3a = 0 = 2b - 1 = 0 = b = 1/2
a - b + c = 0 = 1/3 - 1/2 + c = 0 = c = 1/6

Substituindo a, b e c no polinômio (ii):
S[n] = n^3/3 + n^2/2 + n/6

Fatorando:
S[n] = (2.n^3 + 3.n^2 + n)/6
S[n] = [n(2n^2 + 3n + 1)]/6

S[n] = [n(n + 1)(2n + 1)]/6

Para o caso particular do problema apresentado, teremos:
S[10] = (10.11.21)/6 = S[10] = 385


Atenciosamente,

Rogério Moraes de Carvalho

From: [EMAIL PROTECTED] [mailto:[EMAIL PROTECTED] On
Behalf Of [EMAIL PROTECTED]
Sent: quarta-feira, 19 de maio de 2004 01:21
To: [EMAIL PROTECTED]
Subject: [obm-l] Soma...

Qual o valor de S=1^2+2^2+3^2+.+10^2?
Usei para resolver esse problema a identidade (x+1)^3. Com efeito,
2^3=(1+1)^3=1^3+3*1^2*1+3*1*1^2+1^1
3^3=(2+1)^3=2^3+3*2^2*1+3*2*1^2+1
--
11^3=(10+1)^3=10^3+3*10^2*1+3*10*1^2+1.Isolando convenientemente
3*1^2+3*2^2++3*10^2. descubro S. Minha pergunta é: Existe um modo mais
fácil de se achar soma de quadrados perfeitos??
  Quem souber e puder responder, deixo meu agradecimento.
   Crom



=
Instruções para entrar na lista, sair da lista e usar a lista em
http://www.mat.puc-rio.br/~nicolau/olimp/obm-l.html
=


RE: [obm-l] Exponencial

2004-05-18 Por tôpico Rogério Moraes de Carvalho
Olá Fábio,

Ficou muito difícil entender a questão com esta explicação da
notação no meio do enunciado. De qualquer modo, eu já havia resolvido esta
questão anteriormente.

Segue o enunciado e uma resolução possível.

ENUNCIADO:
Resolva no campo dos reais a seguinte equação exponencial:
3^(x^2 + 1/x^2) = 81/3^(x + 1/x)

RESOLUÇÃO:
Condição de existência: x != 0

Fazendo y = x + 1/x, teremos:
y^2 = (x + 1/x)^2 = y^2 = x^2 + 2 + 1/x^2 = x^2 + 1/x^2 = y^2 - 2

Portanto, representando a equação exponencial em função de y, teremos:
3^(y^2 - 2) = 3^4/3^y = 3^(y^2 - 2) = 3^(4 - y) = y^2 - 2 = 4 - y =
y^2 + y - 6 = 0 = y = -3 ou y = 2

Para y = -3:
x + 1/x = -3 = x^2 + 3x + 1 = 0 = x = [-3-sqr(5)]/2 ou x = [-3+sqr(5)]/2

Para y = 2:
x + 1/x = 2 = x^2 - 2x + 1 = 0 = (x - 1)^2 = 0 = x = 1

Todas as soluções satisfazem a condição de existência.

Resposta: S = {[-3-sqr(5)]/2, [-3+sqr(5)]/2, 1}



Atenciosamente,

Rogério Moraes de Carvalho

From: [EMAIL PROTECTED] [mailto:[EMAIL PROTECTED] On
Behalf Of Fabio Contreiras
Sent: sexta-feira, 14 de maio de 2004 23:00
To: [EMAIL PROTECTED]
Subject: [obm-l] Exponencial

Tentei sair dessa equação mas naum deu em nada... alguem tem o bizu aih hehe
, Abraços!
Fabio
 
 
3^x^2 ( 3 elevado à x ao quadrado ) + 1 / x^2 = { 81 / 3^[(x+1/x)] }
 
 
 
Valeu desde já!



=
Instruções para entrar na lista, sair da lista e usar a lista em
http://www.mat.puc-rio.br/~nicolau/olimp/obm-l.html
=


[obm-l] RE: _[obm-l]_somatório

2004-05-17 Por tôpico Rogério Moraes de Carvalho
Olá Gustavo,

Existem algumas soluções possíveis para encontrar uma fórmula para o
somatório apresentado.


UM POUCO DE TEORIA:

Alguns autores consideram que se os termos consecutivos de uma
progressão aritmética são multiplicados por potências de mesma base e
expoentes consecutivos, então teremos uma progressão aritmético-geométrica
(P.A.G.). Ou seja, uma seqüência do tipo:
a1, (a1 + R).q, (a1 + 2R).q^2, (a1 + 3R).q^3, ..., [a1 + (n - 1)R].q^(n - 1)
é uma progressão aritmético-geométrica de primeiro termo a1, com a1, a1 + R,
a1 + 2R, ..., a1 + (n - 1).R formando uma P.A. de razão R e q, q^2, q^3,
..., q^(n - 1) formando uma P.G. de razão q.

A seguir, eu apresento uma terceira solução possível usando
somatórios e suas propriedades. Esta técnica de resolução pode ser utilizada
para deduzir a fórmula geral da soma dos termos de uma P.A.G. e somente
utiliza a fórmula da soma dos termos de uma P.G. uma única vez.


NOTAÇÃO ADOTADA:
S{i=1,i=n}(a_i) significa somatório com i variando de 1 até n de a índice i.


OUTRA RESOLUÇÃO POSSÍVEL:
Seja S a soma a ser encontrada, então:
S = S{i=0,i=n-1}[(n-i).2^i]
2.S = S{i=0,i=n-1}[(n-i).2^(i+1)]
2.S = S{i+1=1,i+1=n}{[n+1-(i+1)].2^(i+1)}
2.S = S{i=1,i=n}[(n+1-i).2^i]
2.S = S{i=1,i=n}[(n-i).2^i+2^i]
2.S = S{i=1,i=n}[(n-i).2^i] + S{i=1,i=n}(2^i)
2.S = S - (n-0).2^0 + (n-n).2^n + 2.(2^n-1)/(2-1)
2.S - S = -n + 0 + 2^(n+1) - 2
S = 2^(n+1)-(n+2)

Atenciosamente,

Rogério Moraes de Carvalho

From: [EMAIL PROTECTED] [mailto:[EMAIL PROTECTED] On
Behalf Of Gustavo Baggio
Sent: segunda-feira, 17 de maio de 2004 01:13
To: [EMAIL PROTECTED]
Subject: Re:_[obm-l]_somatório

Valeu fábio dias, mas a solução do eduardo henrique me pareceu mas atraente
pois manipula direto no somatório. A sua me pareceu interessante também pelo
que olhei (maios ou menos).
Na verdade esse somatório saiu de uma relação de recorrência de um algoritmo
recursivo aparentemente inofensivo. Rodei o programa para alguns valores de
n e a fórmula é realmente válida.  
 
gustavo

Fabio Dias Moreira [EMAIL PROTECTED] wrote:

Eduardo Henrique Leitner said:
 On Sun, May 16, 2004 at 08:32:39PM -0300, Gustavo Baggio wrote:
 Alguém manja de alguma fórmula pra calcular direto o somatório de n *
 2^0 + (n - 1) * 2^1 + (n - 2)*2^2 + ... + 1*2^(n-1) ?
 Isso nada mais é do que somatório de i variando de 0 até (n-1) de (n
 - i)*(2^i).
 Por exemplo para n = 4 temos 4*1 + 3*2 + 2*4 + 1*8.

 Qualquer dica, enfim, tá valendo...
 [...]

 eis uma maneira:


 n * 2^0 + (n - 1) * 2^1 + (n - 2)*2^2 + ... + 1*2^(n-1) =
 = n[ 2^0 + 2^1 + 2^2 + ... + 2^(n-1) ] - { 1*2^1 + 2*2^2 + 3*2^3 + ... +
 (n-1)*2^(n-1) } =

 partindo do suposto que vc conhece a fórmula da soma dos n primeiros
 termos de uma PG:
 [...]
 resposta: 2^(n+1) - (n+2)
g! t; [...]

Se pudermos usar cálculo tem uma maneira mais direta:

x^0 + x^1 + x^2 + ... + x^n = [x^(n+1) - 1]/(x-1)

Derive os dois lados em relação a x:

1*x^0 + 2*x^1 + ... + n*x^(n-1) = d([x^(n+1) - 1]/[x-1])/dx

Finalmente, multiplique por x:

1*x^1 + 2*x^2 + ... + n*x^n = x * d([x^(n+1) - 1]/[x-1])/dx

O lado direito é facilmente derivado, pois é a derivada de um quociente.
De fato, não é muito difícil ver que ela vale
[n*x^(n+1)-(n+1)*x^n+1]/[x-1]^2. Substituindo x = 2,

1*2^1 + 2*2^2 + ... + n*2^n = 2 * [n*2^(n+1)-(n+1)*2^n+1]/[2-1]^2
1*2^1 + 2*2^2 + ... + n*2^n = n*2^(n+2) - (n+1)*2^(n+1) + 2.

Finalmente, voltando ao problema original,

n*2^0 + (n-1)*2^1 + ... + 1*2^(n-1) =
= n*[2^0+2^1+2^2+...+2^(n-1)] - (1*2^1 + 2*2^2 + ... +
(n-1)*2^(n-1)) =
= n*(2^n-1) - (n-1)*2^(n+1) + n*2^n - 2 =
= n*2^n - n - 2*n*2^n + 2^(n+1) + n*2^n - 2 =
= 2^(n+1) - (n+2).

Note que nós calculamos 1*2^1 ! + ... + n*2^n, mas queremos 1*2^1 + 2*2^2 +
... + (n-1)*2^(n-1), logo temos que trocar o n por n-1.

Outro problema legal nessa mesma linha é o problema 4 da OBM 2002, nível 3.

[]s,

-- 
Fábio ctg \pi Dias Moreira


=
Instruções para entrar na lista, sair da lista e usar a lista em
http://www.mat.puc-rio.br/~nicolau/olimp/obm-l.html
=


Yahoo! Messenger - Fale com seus amigos online. Instale agora!



=
Instruções para entrar na lista, sair da lista e usar a lista em
http://www.mat.puc-rio.br/~nicolau/olimp/obm-l.html
=


RE: [obm-l] cococolegio navalvalval

2004-05-14 Por tôpico Rogério Moraes de Carvalho
Olá Paulo,

No Web Site da Olimpíada Brasileira de Matemática, você poderá
encontrar o seguinte parágrafo que descreve a Lista de Discussão de
Problemas de Matemática Olímpica:
Está aberta uma lista de discussão de problemas de Matemática Olímpica.
A lista é inteiramente gratuita e encontra-se aberta a todos os alunos e
professores que quiserem participar.

Existem vários níveis de Olimpíadas de Matemática no Brasil e no
mundo. Eu acredito que os alunos que estão se preparando podem encontrar
nesta lista um ponto de apoio. Cada um deles vai ter seus pontos fortes e
fracos na Matemática, assim como qualquer um de nós. Devemos ter bom senso
na hora de avaliar o que pode ser omitido numa resolução por ser óbvio e o
que deve ser apresentado por não ser tão evidente.

Eu entendi perfeitamente todos os passos do seu raciocínio, porém
ele não é conclusivo, omite detalhes que não são nem um pouco elementares e
não é suficiente nem mesmo para resolver a questão original do Colégio Naval
com o enunciado incorreto.

Quem sabe você não pode esclarecer OBJETIVAMENTE as questões que eu
apresento nos comentários abaixo. Entenda objetivamente como: apresentado
resultados completos (não parciais).

1) Alguem errou o enunciado de uma questao e propos o calculo de 1/(M^2)  +

1/(N^3) quando deveria ter proposto 1/(N^3) + 1/(M^3).  Aqui, M e N sao
raizes de uma equacao do 2 grau.

COMENTÁRIO: Apresente uma solução elegante para o problema com o enunciado
errado. Quem sabe você não aplica a sua generalização do problema a este
caso particular. Porém, que a resolução seja mais simples do que substituir
M e N pelas raízes da equação.


2) Dado que (M+N)^P = M^P + ALGO MAIS + N^P = M^P + N^P = (M+N)^P - ALGO 
MAIS. Dai : 1/(N^P) + 1/(M^P) = [(M+N)^P - ALGO MAIS ]/(MN)^P e que (M+N)^P,
ALGO MAIS e (MN)^P podem ser efetivamente expressos em funcao de M+N e de MN
... ACABOU

COMENTÁRIO: Apresentar um raciocínio Matemático com ALGO MAIS no meio de
uma expressão me parece algo um tanto quanto chulo. Dizer que (M+N)^P, ALGO
MAIS e (MN)^P podem ser efetivamente expressos em funcao de M+N e de MN ...
ACABOU omite uma questão nem um pouco elementar. Na realidade (M+N)^P e
(MN)^P já estão em função de M+N, MN e P. Porém, o ALGO MAIS que foi
omitido corresponde aos termos centrais (excluindo somente os termos
extremos) do desenvolvimento do Binômio de Newton e não é nem um pouco
simples colocá-los em função de M + N, MN e P.
Como você colocou como elementar esta passagem, por favor, nos apresente o
seu raciocínio matemático e APRESENTE uma fórmula geral que permita calcular
M^P + N^P em função de M + N, MN e P.


3) O Passo 2) acima, numa lista de discussao de problemas de matematica 
olimpica, e equivalente a querer ensinar o PAI NOSSO pra Padre Aposentado.
Logo, trivial e sem graca.

COMENTÁRIO: Infelizmente, apesar de já ter resolvidos vários problemas de
Olimpíadas de Matemática, eu ainda não consegui encontrar uma maneira
SIMPLES de deduzir M^P + N^P em função de M + N, MN e P. Porque você não
compartilha a sua dedução elementar conosco.


4) O erro de enunciado do passo 1) propoe uma questao que - nao obstante 
poder apresentar alguma assimetria - constitui-se num problema digno de
figurar nesta nossa lista e e conforme a nossa tradicao. Em x^2 - 10x + 1 =
0, como encontrar, por exemplo, 1/(M^537) + 1/(N^601), onde M e N sao
raizes.

COMENTÁRIO: Eu estou ansioso para ver o valor de 1/(M^537) + 1/(N^601),
sendo M e N raízes da equação x^2 - 10x + 1 = 0. Qual é o resultado desta
expressão? Por favor, apresente as duas soluções possíveis para o problema.


Isso me parece uma questao interessante... O desenvolvimento abaixo pode 
ajudar de alguma forma:

COMENTÁRIO: Quer dizer que o desenvolvimento apresentado não resolve a
questão que você propôs? 


Sejam N e M raizes e A e B naturais positivos. Podemos admitir que:

M^A + N^A = C1 = Efetivamente calculavel em funcao de M+N e MN
M^B + N^B = C2 = Efetivamente calculavel em funcao de M+N e MN
Daqui : (M^A + N^B) + (M^B + N^A ) = C1 + C2 = efetivamente calculavel

COMENTÁRIO: Na realidade, M^A + N^A = C1 pode ser calculado em função de M +
N, MN e A e M^B + N^B = C2 pode ser calculado em função de M + N, MN e
B. Por favor, prove a sua afirmação MOSTRANDO a expressão de M^A + N^A em
função de M + N, MN e A.


COMENTÁRIO DO RESTANTE DO RACIOCÍNIO: Será que os jurados de uma Olimpíada
de Matemática aceitariam o desenvolvimento de um raciocínio matemático com a
omissão de vários passos essenciais e com expressões como ALGO MAIS e
efetivamente calculável? Deste modo, seria simples demonstrar um teorema,
bastando para isto omitir partes da demonstração e utilizar um ALGO MAIS
ou um efetivamente calculável.
Quem sabe o Morgado, que já foi membro do Jurado Internacional na 34ª
Olimpíada Internacional de Matemática, realizada em Istambul em 1993, não
poderia nos esclarecer esta dúvida.


Abraços,

Rogério Moraes de Carvalho
-Original Message-
From: [EMAIL

RE: [obm-l] Duvidas

2004-05-14 Por tôpico Rogério Moraes de Carvalho
RESOLUÇÃO POSSÍVEL:

Uma vez que as raízes pertencem a um intervalo real, somos obrigados a
concluir que elas são reais, pois não há relação de ordem no conjunto dos
números complexos.

Sejam x1 e x2 as raízes da função f, tais que x1 != x2. O ponto médio de x1
e x2 no eixo das abscissas pode ser calculado por xm = (x1 + x2)/2 = xm =
(-b/a)/2 = xm = -b/(2a) (abscissa do vértice da parábola).

Se x1 e x2 pertencem ao intervalo [-2, 3], então xm está no interior do
mesmo. Ou seja:
-2  -b/(2a)  3 = -2  -b/2  3  (x (-2))= -6  b  4

Resposta: Alternativa B

Rogério Moraes de Carvalho
-Original Message-
From: [EMAIL PROTECTED] [mailto:[EMAIL PROTECTED] On
Behalf Of aryqueirozq
Sent: sexta-feira, 14 de maio de 2004 14:45
To: [EMAIL PROTECTED]
Subject: [obm-l] Duvidas

  Aguém poderia me ajudar nesta questao:


   A funcao f( x) = x^2 + bx + c , com  b e c reais, 
tem duas raizes distintas pertencente ao intervalo
[- 2 , 3].Entao , sobre os valores de b e c , a única 
afirmativa correta eh
a)c9
b)-6b4
c)b-6
d)4b6
e)c-6


 
__
Acabe com aquelas janelinhas que pulam na sua tela.
AntiPop-up UOL - É grátis!
http://antipopup.uol.com.br/



=
Instruções para entrar na lista, sair da lista e usar a lista em
http://www.mat.puc-rio.br/~nicolau/olimp/obm-l.html
=




=
Instruções para entrar na lista, sair da lista e usar a lista em
http://www.mat.puc-rio.br/~nicolau/olimp/obm-l.html
=


RE: [obm-l] cococolegio navalvalval

2004-05-13 Por tôpico Rogério Moraes de Carvalho
Olá Leandro,

Eu não tenho acesso às provas dos exames passados do Colégio Naval,
porém eu acredito que o Pacini esteja correto. Principalmente, porque se o
problema estiver pedindo o valor da expressão 1/M^3 + 1/N^2, então não há
alternativa alguma que satisfaça o problema, mesmo que uma das soluções seja
aproximadamente igual a 970. Mas, se o problema estiver pedindo o valor da
expressão 1/M^3 + 1/N^3, então o resultado é igual a exatamente 970
(Alternativa A).


De qualquer modo, vamos supor que o problema tenha pedido o valor da
expressão 1/M^3 + 1/N^2. Neste caso, a expressão produz resultados
diferentes dependendo da escolha das raízes que representarão M e N,
respectivamente. Vamos à resolução braçal do problema neste caso.

**
CÁLCULO DA EXPRESSÃO 1/M^3 + 1/N^2
**
Calculando as raízes da equação X^2 - 10X + 1 = 0, encontramos:
X1 = 5 - 2.sqr(6) e X2 = 5 + 2.sqr(6)

Primeira solução:
-
Fazendo M = X1 e N = X2, teremos
1/M^3 + 1/N^2 = 178.[3 + sqr(6)] = 970,009174215... (aproximadamente 970)

Observação: Não se pode concluir que a alternativa A seja correta, pois além
do resultado 970 ser aproximado, esta é apenas uma das soluções possíveis.

Segunda solução:

Fazendo M = X2 e N = X1, teremos
1/M^3 + 1/N^2 = 178.[3 - sqr(6)] = 97,9908257852... (aproximadamente 98)

Conclusão: Se o enunciado original realmente for 1/M^3 + 1/N^2, então esta
questão deveria ser anulada uma vez que há duas soluções. Mesmo que a única
solução fosse 178.[3 + sqr(6)] não seria correto assinalar a alternativa A
como correta, a não ser que o enunciado deixasse explícito que se deseja o
valor aproximado da expressão. Ou seja, algo como:
Sendo M e N as raízes da equação X^2-10X+1=0 , o valor aproximado da
expressão 1/M^3 + 1/N^2 é:



Agora vamos à solução do problema supondo que a expressão pedida
seja 1/M^3 + 1/N^2.

**
CÁLCULO DA EXPRESSÃO 1/M^3 + 1/N^3
**
Segue a resolução que eu havia apresentado em uma mensagem anterior.

Colocando a expressão 1/M^3 + 1/N^3 em função da soma S = M + N = -b/a = 10
e do produto das raízes P = M.N = c/a = 1, teremos:

1/M^3 + 1/N^3 = (M^3 + N^3)/(M^3.N^3) = [(M + N).(M^2 - M.N + N^2]/[(M.N)^3]
= [(M + N).(M + N)^2 - 3.M.N]/[(M.N)^3] = [S.(S^2 - 3.P)]/(P^3) = [10.(10^2
- 3.1)]/(1^3) = 10.(100 - 3) = 970

Alternativa A



Quanto aos comentários do Paulo Santa Rita, eu não consegui entender
a intenção da questão proposta e nem a aplicação para calcular 1/M^3 + 1/N^2
no problema do Colégio Naval. Inclusive, tem alguns passos que eu não
consegui entender, como em:
(M+N)^3 = M^3 + 2(M^2)N + 3M(N^2) +N^2 = M^3 +3MN(M+N) + N^3
A expressão intermediária não deveria ser M^3 + 3(M^2)N + 3M(N^2) + N^3 ao
invés de M^3 + 2(M^2)N + 3M(N^2) +N^2? De qualquer modo, o resultado final
ficou correto.

Eu não consegui entender a generalização do problema e nem como esta
generalização pode ser aplicada para resolver este problema específico.

Abraços,

Rogério Moraes de Carvalho
[EMAIL PROTECTED]

-Original Message-
From: [EMAIL PROTECTED] [mailto:[EMAIL PROTECTED] On
Behalf Of leandro-epcar
Sent: quinta-feira, 13 de maio de 2004 12:08
To: obm-l
Cc: obm-l
Subject: RE: [obm-l] cococolegio navalvalval

pacini o enunciado é este e a resposta é a 
letra a



-- Início da mensagem original ---

  De: [EMAIL PROTECTED]
Para: [EMAIL PROTECTED]
  Cc: 
Data: Sun, 09 May 2004 15:26:55 +
 Assunto: RE: [obm-l] cococolegio navalvalval

 Ola Pessoal,
 
 Bendito erro de transcricao ! A solucao de um 
problema trivial acrescenta 
 muito pouco ao
 espirito e objetivos originais desta nossa lista.
 
 Sem duvida e verdade que devemos apresentar os 
enunciados tal como eles 
 realmente sao,
 mas o erro de transcricao abaixo abaixo parece 
sugerir um problema MAIS 
 CONFORME o elan
 olimpico que nos inspira ...
 
 Se M e N sao raizes da equacao X^2 - Bx + 1 = 0, B  
2, entao para P primo, 
 existe uma forma
 sintetica ( em funcao de B ) de exprimir
 
 f(M,N) = 1/(M^(P+1))  +  1/(N^P)  ?
 
 Se P=2, entao, claramente :
 
 (M+N)^2 = M^2 + 2MN + N^2 = M^2 + N^2 + 2, pois MN = 1
 (M+N)^3 = M^3 + 2(M^2)N + 3M(N^2) +N^2 = M^3 +3MN
(M+N) + N^3
 (M+N)^2 + (M+N)^3 - 3MN(M+N) - 2 = (M^2 + N^3) + (M^3 
+ N^2)
 
 No caso, M=1/N e M^P + 1/M^P =f(M+1/M) e M+1/M = B 
para todo P ...
 
 Um Abraco a Todos
 Paulo Santa Rita
 1,1222,090504
 
 -Original Message-
 From: [EMAIL PROTECTED] [mailto:owner-obm-
[EMAIL PROTECTED] On
 Behalf Of Pacini bores
 Sent: sábado, 8 de maio de 2004 18:49
 To: [EMAIL PROTECTED]
 Subject: Re: [obm-l] cococolegio navalvalval
 
 
 Olá  Leandro ,
 
 O enunciado  correto eh :
Sendo M e N as raízes da equação X^2-10X+1=0 , o
 valor da expressão  1/M^3 + 1/N^3 ,  ok ?
 
 []´s Pacini
 
 
 
   COLÉGIO NAVAL (1989)
  
 Sendo M e N as raízes da equação X^2-10X

RE: [obm-l] Duvidas

2004-05-12 Por tôpico Rogério Moraes de Carvalho
POSSÍVEL RESOLUÇÃO PARA A QUESTÃO 1:
Na realidade, esta questão não tem dados suficientes para a sua resolução,
uma vez que existem infinitas parábolas que passam pelos três pontos.

Pela simplicidade das duas questões passadas, é certo que o autor da questão
quer a parábola cuja diretriz seja horizontal.

Neste caso, podemos escrever: f(t) = a.t^2 + b.t + c, com a != 0.
f(1) = 3,00 = a + b + c = 3 (E1)
f(2) = 5,00 = 4a + 2b + c = 5 (E2)
f(3) = 1,00 = 9a + 3b + c = 1 (E3)

O sistema pode ser resolvido rapidamente pelo método de eliminação de Gauss:
a + b + c = 3 (E1)
3a + b = 2 (E2 - E1 = E4)
8a + 2b = -2 (E3 - E1 = E5)

a + b + c = 3 (E1)
3a + b = 2 (E4)
2a = -6 (E5 - 2E4 = E6)

(E6) a = -3
(E4) -9 + b = 2 = b = 11
(E1) -3 + 11 + c = 3 = c = -5

Logo: f(t) = -3.t^2 + 11.t - 5 = f(5/2) = 15/4 = 3,75

Resposta: Letra d (Rigorosamente, o problema não tem dados suficientes para
ser resolvido)


POSSÍVEL RESOLUÇÃO PARA A QUESTÃO 2:
Uma vez que o eixo de simetria é horizontal, a solução se torna muito
simples, pois para todo x real, as ordenadas das duas parábolas serão
simétricas em relação à reta y = -2. Sendo assim, -2 é ponto médio das
ordenadas das parábolas, para todo x real.

Sejam:
f(x) = 2 - x^2
g(x) (função da outra parábola)

-2 = [f(x) + g(x)]/2 = -4 = 2 - x^2 + g(x) = g(x) = x^2 - 6
a = 1, b = 0 e c = -6
8a + b + c = 2

Resposta: Letra c

Atenciosamente,

Rogério Moraes de Carvalho
Consultor e Instrutor de Tecnologias da Informação
[EMAIL PROTECTED]

-Original Message-
From: [EMAIL PROTECTED] [mailto:[EMAIL PROTECTED] On
Behalf Of aryqueirozq
Sent: terça-feira, 11 de maio de 2004 22:48
To: [EMAIL PROTECTED]
Subject: [obm-l] Duvidas



 Alguém poderia me ajudar nessas duas questoes.

  Agradeço desde de já.

1)Os dados experimentais da tabela a seguir 
correspondem às concentrações de uma substância química 
medida em intervalos de 1 segundo. Assumindo que a 
linha que passa pelos três pontos experimentais é uma 
parábola, tem-se que a concentração (em moles) após 2,5 
segundos é:

Tempo (s)Concentração (moles)
1   3,00
2   5,00
3   1,00

a) 3,60
b) 3,65
c) 3,70
d) 3,75
e) 3,80

2)O gráfico da função quadrática y=ax2+bx+c, x real, é 
simétrico ao gráfico da parábola y=2-x2 com relação à 
reta de equação cartesiana y= -2. Determine o valor de 
8a+b+c.
a) - 4
b) 1/2
c) 2
d) 1
e) 4







 
__
Acabe com aquelas janelinhas que pulam na sua tela.
AntiPop-up UOL - É grátis!
http://antipopup.uol.com.br/



=
Instruções para entrar na lista, sair da lista e usar a lista em
http://www.mat.puc-rio.br/~nicolau/olimp/obm-l.html
=




=
Instruções para entrar na lista, sair da lista e usar a lista em
http://www.mat.puc-rio.br/~nicolau/olimp/obm-l.html
=


RE: [obm-l] Trigonometria

2004-05-11 Por tôpico Rogério Moraes de Carvalho
Olá Jerry,

Existe um caminho bem curto para a resolução deste problema. Basta
agrupar as parcelas de funções trigonométricas apropriadas no numerador e no
denominador e transformá-las em multiplicações.

Na resolução, eu estarei usando as seguintes identidades trigonométricas:

sen(p) + sen(q) = 2.sen[(p+q)/2].cos[(p-q)/2], com p e q reais.
cos(p) + cos(q) = 2.cos[(p+q)/2].cos[(p-q)/2], com p e q reais.

As identidades trigonométricas acima, conhecidas como fórmulas de
prostaférese, de transformação em produto ou, ainda, de fatoração podem ser
demonstradas facilmente aplicando-se as identidades de soma e diferença de
arcos.

RESOLUÇÃO POSSÍVEL:
Seja E = (sen30°+sen40°+sen50°)/(cos30°+cos40°+cos50°), então:
E = [(sen50°+sen30°)+sen40°)/[(cos30°+cos50°)+cos40°]
E = {2.sen[(50°+30°)/2].cos[(50°-30°)/2]+sen40°)/
{2.cos[(50°+30°)/2].cos[(50°-30°)/2]+cos40°}
E = (2.sen40°.cos10°+sen40°)/(2.cos40°.cos10°+cos40°}
E = [sen40°(2.cos10°+1)]/[cos40°(2.cos10°+1)]
E = sen40°/cos40°
E = tg40° c.q.d.

Atenciosamente,

Rogério Moraes de Carvalho
Consultor e Instrutor de Tecnologias da Informação
[EMAIL PROTECTED]

From: [EMAIL PROTECTED] [mailto:[EMAIL PROTECTED] On
Behalf Of Jerry Eduardo
Sent: terça-feira, 11 de maio de 2004 18:35
To: [EMAIL PROTECTED]
Subject: [obm-l] Trigonometria

Alguem pode me dar uma dica de como 
resolver o exercicio abaixo:
 
Mostre que:
 
tg40 = (sen30 + sen40 + sen50) / (cos30 + cos40 + cos50)
 
Grato,
 
Jerry



=
Instruções para entrar na lista, sair da lista e usar a lista em
http://www.mat.puc-rio.br/~nicolau/olimp/obm-l.html
=


RE: [obm-l] cococolegio navalvalval

2004-05-09 Por tôpico Rogério Moraes de Carvalho
Olá pessoal,

Vamos tentar prestar mais atenção na hora de copiar os enunciados
das questões, afinal de contas muitas pessoas estão gastando os seus tempos
para resolvê-las.

Vamos à resolução desta questão.

RESOLUÇÃO POSSÍVEL:

A idéia é tentar colocar a expressão 1/M^3 + 1/N^3 em função da soma das
raízes S = M + N = -b/a = 10 e do produtos das raízes P = M.N = c/a = 1.

Seja E = 1/M^3 + 1/N^3, então:
E = (M^3 + N^3)/(M^3.N^3)
E = [(M + N).(M^2 - M.N + N^2]/[(M.N)^3]
E = [(M + N).(M + N)^2 - 3.M.N]/[(M.N)^3]
E = [S.(S^2 - 3.P)]/(P^3)
E = [10.(10^2 - 3.1)]/(1^3)
E = 10.(100 - 3)
E = 970

Resposta: Letra A

Rogério Moraes de Carvalho
Consultor e Instrutor de Tecnologias da Informação
[EMAIL PROTECTED]


-Original Message-
From: [EMAIL PROTECTED] [mailto:[EMAIL PROTECTED] On
Behalf Of Pacini bores
Sent: sábado, 8 de maio de 2004 18:49
To: [EMAIL PROTECTED]
Subject: Re: [obm-l] cococolegio navalvalval


Olá  Leandro ,

O enunciado  correto eh :
  Sendo M e N as raízes da equação X^2-10X+1=0 , o
valor da expressão  1/M^3 + 1/N^3 ,  ok ?

[]´s Pacini



 COLÉGIO NAVAL (1989)

   Sendo M e N as raízes da equação X^2-10X+1=0 , o
valor da expressão  1/M^3 + 1/N^2 é :

  (A) 970
  (B) 950
  (C) 920
  (D) 900
  (E) 870
===
  ,



=
Instruções para entrar na lista, sair da lista e usar a lista em
http://www.mat.puc-rio.br/~nicolau/olimp/obm-l.html
=




=
Instruções para entrar na lista, sair da lista e usar a lista em
http://www.mat.puc-rio.br/~nicolau/olimp/obm-l.html
=


RE: [obm-l] En:COLEGIO NAVAL

2004-05-09 Por tôpico Rogério Moraes de Carvalho
Olá Leandro,

Seria interessante que você já disponibilizasse o mais rápido
possível as provas que você tem do Colégio Naval para os participantes da
lista. Deste modo, você estaria facilitando o acesso às questões originais
por parte de todos e ficaria mais fácil poder ajudá-lo.

Abraços,

Rogério Moraes de Carvalho
[EMAIL PROTECTED]


-Original Message-
From: [EMAIL PROTECTED] [mailto:[EMAIL PROTECTED] On
Behalf Of leandro-epcar
Sent: sábado, 8 de maio de 2004 15:49
To: obm-l
Subject: Re: [obm-l] En:COLEGIO NAVAL

   Em breve colocarei as provas do colegio naval na 
internet.
   Sobre  LEANDRO-EPCAR eu diria em rumo À EPCAR
 ou o colegio naval.
   Esse ano vou fazer a prova e pretendo passar.

   ATENCIOSAMENTE

   LEANDRO 

-- Início da mensagem original ---

  De: quot;Alan Pellejeroquot; 
[EMAIL PROTECTED]
Para: [EMAIL PROTECTED]
  Cc: 
Data: Thu, 6 May 2004 18:42:58 -0300 (ART)
 Assunto: Re: [obm-l] En:COLEGIO NAVAL

 e ai cara, meu, eu ficaria agradecido se vc me 
mandasse
 vc faz mesmo epcar ou esse nick é só pra botar uma 
moral???
 hehe
 valeu cara,
 té mais!
 Alan Pellejero
 
 leandro-epcar [EMAIL PROTECTED] wrote:
 -- Início da mensagem original ---
 
 De: leandro-epcar leandro-
 [EMAIL PROTECTED]
 Para: obm-l [EMAIL PROTECTED]
 Cc: 
 Data: Wed, 28 Apr 2004 10:31:09 -0300
 Assunto: COLEGIO NAVAL
 
 de:leandro geraldo da costa.
 para:obm-l.mat.puc-rio.com.br
 assunto:geometria plana.
 data: 28 de abril de 2004.
 -- 
 
 PROVA DO COLEGIO NAVAL DE 2003.
 
 Num quadrilátero ABCD tem-se:AB=42, BC=48, CD=64,
 DA=49 e P é o ponto de interseção entre as diagonais
 AC e BD .Qual a razão entre os segmentos PA e PC ,sa-
 bendo-se que a diagonal BD é igual a 56.
 
 (A)7/8.
 (B)8/7.
 (C)7/6.
 (D)6/7.
 (E)49/64.
 --
 OBSERVAÇÃO:
 
 Desenhando com escala notei que a diagonal BD é 
 bissetriz de ADC, mas não tenho como provar. 
 
 --
 
 Tenho as provas de admissao do colegio naval de
 52 a 66 e 87 a 2003.
 se houver algum interesado ,entrem em contato . 
 
 

 __
 Acabe com aquelas janelinhas que pulam na sua tela.
 AntiPop-up UOL - É grátis!
 http://antipopup.uol.com.br/
 
 
 
 

__
 Acabe com aquelas janelinhas que pulam na sua tela.
 AntiPop-up UOL - É grátis!
 http://antipopup.uol.com.br/
 
 
 
 

=
 Instruções para entrar na lista, sair da lista e usar 
a lista em
 http://www.mat.puc-rio.br/~nicolau/olimp/obm-l.html
 

=
 
 
 -
 Yahoo! Messenger - Fale com seus amigos online. 
Instale agora!
 
__
Acabe com aquelas janelinhas que pulam na sua tela.
AntiPop-up UOL - É grátis!
http://antipopup.uol.com.br/



=
Instruções para entrar na lista, sair da lista e usar a lista em
http://www.mat.puc-rio.br/~nicolau/olimp/obm-l.html
=




=
Instruções para entrar na lista, sair da lista e usar a lista em
http://www.mat.puc-rio.br/~nicolau/olimp/obm-l.html
=


[obm-l] RE: [obm-l] álgebra

2004-05-05 Por tôpico Rogério Moraes de Carvalho
Eu acredito que você tenha escrito os termos da soma de modo errado, pois
não haveria necessidade de parênteses externos nos termos. Provavelmente, a
soma desejada é um caso particular da clássica apresentada após a notação.

Na resolução considere a seguinte notação:
S[i=a][i=b]{f(i)}: Somatório de f(i) com i variando de a até b.

Calcule a soma
S=1/[sqr(2)+sqr(1)]+1/[sqr(3)+sqr(2)]+1/[sqr(4)+sqr(3)]+...+
1/[sqr(n)+sqr(n-1)], como n inteiro maior que 1.

RESOLUÇÃO:
S = S[i=2][i=n]{1/[sqr(i)+sqr(i-1)]}
Multiplicando o numerador e o denominador do termo geral pelo termo
racionalizante, tem-se:
S = S[i=2][i=n]{1.[sqr(i)-sqr(i-1)]/[sqr(i)+sqr(i-1)].[sqr(i)-sqr(i-1)]}
S = S[i=2][i=n]{[sqr(i)-sqr(i-1)]/[i-(i-1)]}
S = S[i=2][i=n]{[sqr(i)-sqr(i-1)]}
S = S[i=2][i=n]{sqr(i)}-S[i=2][i=n]{sqr(i-1)}
S = S[i=2][i=n]{sqr(i)}-S[i-1=1][i-1=n-1]{sqr(i-1)}
S = S[i=2][i=n]{sqr(i)}-S[i=1][i=n-1]{sqr(i)}
S = S[i=2][i=n-1]{sqr(i)}+sqr(n)-sqr(1)-S[i=2][i=n-1]{sqr(i)}
S = sqr(n)-sqr(1)
S = sqr(n)-1

No caso particular deste problema:
n = 100: S = sqr(100)-1 = 10-1 = 9

Resposta: S = 9

Rogério Moraes de Carvalho
Consultor e Instrutor de Tecnologias da Informação
[EMAIL PROTECTED]

-Original Message-
From: [EMAIL PROTECTED] [mailto:[EMAIL PROTECTED] On
Behalf Of seanjr
Sent: quarta-feira, 5 de maio de 2004 20:17
To: [EMAIL PROTECTED]
Subject: [obm-l] álgebra 

Calcule a soma S= ( 1/sqrt ( 2 ) + sqrt ( 1 ) ) + ( 1/sqrt ( 
3 ) + sqrt (
2 ) ) + ( 1/sqrt ( 4 ) + sqrt ( 3 ) ) + ... + ( 1/sqrt ( 99 ) 
+ sqrt (
100 ) )

 
---
Acabe com aquelas janelinhas que pulam na sua tela.
AntiPop-up UOL - É grátis! 
http://antipopup.uol.com.br


=
Instruções para entrar na lista, sair da lista e usar a lista em
http://www.mat.puc-rio.br/~nicolau/olimp/obm-l.html
=




=
Instruções para entrar na lista, sair da lista e usar a lista em
http://www.mat.puc-rio.br/~nicolau/olimp/obm-l.html
=


RE: Re[2]: [obm-l] Geometria Plana - Desafio (?)

2004-05-03 Por tôpico Rogério Moraes de Carvalho
Eu já havia resolvido este problema e, se não me engano, ele caiu em
uma das provas do Colégio Naval. Porém, ao ler o enunciado fornecido pelo
Victor, eu estranhei a omissão da informação de que o segmento MN que divide
o trapézio em dois outros trapézios equivalentes é paralelo às bases AB e
CD. A fim de garantir que a ausência desta informação não garante a
unicidade do cálculo da medida do segmento MN em função de a e b, eu
formulei uma outra questão para utilizá-la como um contra-exemplo.


Vamos ao enunciado da questão que eu formulei baseando-me no problema
fornecido pelo Victor:

Seja ABCD um trapézio retângulo de bases AB = 1 e CD = 3 e cujo ângulo
interno formado entre o lado BC e a base CD é igual a 60°. Dados os pontos M
e N, pertencentes aos lados não-paralelos, tais que o segmento MN divide
esse trapézio em dois outros trapézios equivalentes, calcule MN para cada um
dos dois casos apresentados abaixo.

Primeiro caso: MN é perpendicular ao lado BC. (Resposta: sqr(5))
Segundo caso: MN forma um ângulo de 30° com o prolongamento da base AB no
sentido de B para A. (Resposta: sqr(10))


A fim de tentar garantir que as respostas apresentadas estão corretas, eu
desenvolvi mais de uma solução para cada caso, sendo que uma delas foi por
Geometria Analítica.

A resolução apresentada pelo Boromir corresponde a uma das soluções que eu
havia desenvolvido para o problema original, porém ela somente tem validade
se no enunciado for informado que o segmento MN é paralelo às bases AB e CD,
o que não foi o caso do problema proposto pelo Victor. Vamos reformular o
enunciado para que a resolução do Boromir seja válida e, na seqüência, eu
apresentarei uma resolução alternativa.



ENUNCIADO MODIFICADO:

Dado um trapézio ABCD de bases AB = a e CD = b e os pontos M e N
pertencentes aos lados não-paralelos. Se o segmento MN é paralelo às bases e
divide esse trapézio em dois outros trapézios equivalentes, calcule MN em
função dos lados AB = a e CD = b.


RESOLUÇÃO ALTERNATIVA:

Considere CD = b como a base maior e AB = a como a base menor, logo b  a,
MN = x, H a distância entre a AB e MN e h a distância entre MN e CD. Também
considere que o ponto M pertence ao lado DA e o ponto N ao lado BC.

Trace uma paralela ao lado DA passando pelo vértice B do trapézio, de modo a
interceptar o segmento MN no ponto E e a base CD no ponto F. Deste modo,
ABEM e MEFD são paralelogramos, conseqüentemente tem os lados opostos
congruentes. Portanto: ME = AB = a e DF = ME = a.
MN = ME + EN = x = a + EN = EN = x - a
DC = DF + FC = b = a + FC = FC = b - a

Triângulo BEN ~ Triângulo BFC (Critério AA~):
FC/EN = (H + h)/ H = (b - a)/(x - a) = 1 + h/H =
= (b - x)/(x - a) = h/H (i)

De acordo com os dados, os trapézios ABNM e MNCD são equivalentes, logo:
S[ABNM] = S[MNCD] = (1/2).(x + a).H = (1/2).(b + x).h =
= (x + a)/(b + x) = h/H (ii)

Aplicando a propriedade transitiva nas igualdades (i) e (ii):
(b - x)/(x - a) = (x + a)/(b + x) = x^2 - a^2 = b^2 - x^2 =
= 2x^2 = a^2 + b^2 = x = sqr[(a^2 + b^2)/2]

Resposta: MN = sqr[(a^2 + b^2)/2]



Aplicando a fórmula encontrada para resolver o problema do trapézio
retângulo com bases AB = 1 e CD = 3 e BCD = 60°, apresentado acima,
teremos:
MN = sqr[(1^2 + 3^2)/2] = sqr(10/2) = sqr(5)

Observe que o valor encontrado na aplicação da fórmula coincide com o valor
encontrado no primeiro caso, mas difere do valor encontrado no segundo caso.
Portanto, a informação de que MN é paralelo às bases é necessária para
garantir a unicidade do comprimento de MN em função de a e b, uma vez que
com diferentes inclinações podemos encontrar um valor diferente de sqr[(a^2
+ b^2)/2] para o comprimento do segmento MN.


Atenciosamente,

Rogério Moraes de Carvalho
Consultor e Instrutor de Tecnologias da Informação
[EMAIL PROTECTED]

-Original Message-
From: [EMAIL PROTECTED] [mailto:[EMAIL PROTECTED] On
Behalf Of boromir
Sent: sexta-feira, 30 de abril de 2004 01:09
To: [EMAIL PROTECTED]
Subject: Re[2]: [obm-l] Geometria Plana - Desafio (?)

Ajeitei o texto (embora não tenha usado nenhum caracter especial) eu tb
recebi a mensagem truncada.
[]'s MP



=
De:Fellipe Rossi [EMAIL PROTECTED]
Para:[EMAIL PROTECTED]
Assunto:Re: [obm-l] Geometria Plana - Desafio (?)

Boromir não consigo entender nada da mensagem
Talvez voce esteja usando mtos caracteres
especiais...

MEnsagem alterada:

Vamos considerar a  b. Seja ainda P o ponto de encontro dos prolongamentos
dos lados não paralelos DA e CB. Conforme o enunciado, [ABNM]=[NMDB] = S.
([figura] = área do figura)

Vamos considerar [APB]=K.
APB ~ MPN  (razão a/x, onde MN = x). A razão entre as áreas é o quadrado da
razão de semelhança, portanto (K+S)/K = (x/a)^2.

Ainda temos que
APB~DPC (razão a/b), portanto (K+2S)/K = (b/a)^2.

Escrevendo melhor as equações acima, temos:
1 + S/K = x²/a² - S/K = (x²-a²)/a²
1 + 2S/K = b²/a² - 2S/K = (b²-a²)/a²

Dividindo a segunda pela primeira equação temos:
2(x²-a²) = b²-a²
2x²=b²+a²
x = SQRT{(a²+b²)/2}

Se eu não errei

RE: [obm-l] Binomios... Duvida ( interessante )

2004-04-27 Por tôpico Rogério Moraes de Carvalho








Olá Fábio,



Segue uma solução possível
para este problema.



Sejam a[k] e b[p] os
termos gerais dos binômios de Newton (2x - 3)^4 e (x + 2)^5, respectivamente,
termos:

a[k] = BINOM(4, k).(2x)^k.(-3)^(4
- k), com k pertencente a {0, 1, 2, 3, 4}

b[p] = BINOM(5, p).x^p.2^(5
- p), com k pertencente a {0, 1, 2, 3, 4}



Na multiplicação das
potências dos binômios, teremos que todos os termos a[k] possíveis serão
multiplicados por todos os termos b[p] possíveis, por aplicação da propriedade
distributiva.

Portanto:

a[k].b[p] = BINOM(4, k).BINOM(5,
p).2^k.(-3)^(4 - k).2^(5 - p).x^(k + p)



Devemos encontrar todos
os pares (k, p) tais que k + p = 3: (0, 3), (1, 2), (2, 1) e (3, 0).



Para k = 0 e p = 3: BINOM(4,
0).BINOM(5, 3).2^0.(-3)^4.2^2.x^3 = 3240.x^3

Para k = 1 e p = 2: BINOM(4,
1).BINOM(5, 2).2^1.(-3)^3.2^3.x^3 = -17280.x^3

Para k = 2 e p = 1: BINOM(4,
2).BINOM(5, 1).2^2.(-3)^2.2^4.x^3 = 17280.x^3

Para k = 3 e p = 0: BINOM(4,
3).BINOM(5, 0).2^3.(-3)^1.2^5.x^3 = -3072.x^3



Adicionando os termos: 3240.x^3
+ (-17280.x^3) + 17280.x^3 + (-3072.x^3) = 168.x^3



Portanto, o coeficiente
de x^3 é igual a 168.



Abraços,





Rogério Moraes de Carvalho

Consultor
e Instrutor de Tecnologias da Informação

[EMAIL PROTECTED]











From: owner-[EMAIL PROTECTED] [mailto:owner-[EMAIL PROTECTED]] On Behalf Of Fabio Contreiras
Sent: terça-feira, 27 de abril de
2004 22:45
To: [EMAIL PROTECTED]
Subject: [obm-l] Binomios...
Duvida ( interessante )







1) Determinar o coeficiente de x^3 no desenvolvimento de (
2x - 3 )^4 . ( x + 2 )^5

















Alguem pode me explicar o caminho ?





Abração!










RE: [obm-l] DUVIDA - Trigonometria

2004-04-23 Por tôpico Rogério Moraes de Carvalho
Para que o enunciado desta questão fique preciso é importante citar não
existe x real, pois existe solução no campo dos complexos. Sendo assim,
segue a resolução da questão:
Mostre que não existe x real tal que sen(x). sen(2x).sen(3x) = 4/5.

--
Resolução:
--
Na resolução serão usadas as seguintes identidades trigonométricas, que são
conseqüências diretas das fórmulas de seno e co-seno da adição e subtração
de dois ângulos.
cos(a - b) - cos(a + b) = 2.sen(a).sen(b) (i)
sen(a + b) + sen(a - b) = 2.sen(a).cos(b) (ii)
sen(2a) = 2.sen(a).cos(a) (iii)

sen(x).sen(2x).sen(3x) = 4/5

Reorganizando os fatores do primeiro membro e multiplicando ambos os membros
da igualdade por 2, teremos:
[2.sen(3x).sen(x)].sen(2x) = 8/5

Aplicando a identidade (i) ao primeiro membro da igualdade, teremos:
[cos(3x - x) - cos(3x + x)].sen(2x) = 8/5
sen(2x).cos(2x) - sen(2x).cos(4x) = 8/5

Multiplicando ambos os membros da igualdade por 2, teremos:
2.sen(2x).cos(2x) - 2.sen(2x).cos(4x) = 16/5

Pelas identidades (ii) e (iii), teremos:
sen(2.2x) - [sen(2x + 4x) + sen(2x - 4x)] = 16/5
sen(4x) - sen(6x) - sen(-2x) = 16/5
sen(2x) + sen(4x) + sen(-6x) = 16/5 (iv)

O seno de um número real sempre está no intervalo de -1 até 1. Portanto,
podemos concluir que:
-1 = sen(2x) = 1 (v), qualquer que seja o x real
-1 = sen(4x) = 1 (vi), qualquer que seja o x real
-1 = sen(-6x) = 1 (vii), qualquer que seja o x real

Adicionando, membro a membro, as desigualdades (v), (vi) e (vii), teremos:
-3 = sen(2x) + sen(4x) + sen(-6x) = 3 (viii)

Pela igualdade (iv), devemos ter:
sen(2x) + sen(4x) + sen(-6x) = 16/5  3. Pela desigualdade (viii) podemos
concluir que a igualdade é impossível qualquer que seja o x real. Ou seja,
não existe x real tal que sen(x). sen(2x).sen(3x) = 4/5.

Atenciosamente,

Rogério Moraes de Carvalho
Consultor e Instrutor de Tecnologias da Informação
[EMAIL PROTECTED]

From: [EMAIL PROTECTED] [mailto:[EMAIL PROTECTED] On
Behalf Of João Silva
Sent: sexta-feira, 23 de abril de 2004 15:46
To: [EMAIL PROTECTED]
Subject: [obm-l] DUVIDA - Trigonometria

Alguem pode me ajudar nesta questão:
 
- Mostre que não existe x tal que sen x . sen 2x. sen 3x = 4 / 5


Yahoo! Messenger - Fale com seus amigos online. Instale agora!



=
Instruções para entrar na lista, sair da lista e usar a lista em
http://www.mat.puc-rio.br/~nicolau/olimp/obm-l.html
=


[obm-l] RE: [obm-l] Re:_[obm-l]_Equação_Trigonométrica!

2004-04-17 Por tôpico Rogério Moraes de Carvalho
Carlos,

A seguir, eu apresento a minha resolução para o problema.
Analisando-a você poderá encontrar alguns pequenos problemas na sua
resolução. Observe que as duas resoluções são bem similares.


Questão:

Resolva em R, a seguinte equação.
2.sen(x).|sen(x)| + 3.sen(x) = 2

**
Resolução:
**
Considerando y = sen(x), teremos:
2.y.|y| + 3.y = 2 = 2.y.|y| + 3.y - 2 = 0

Pela definição de módulo de um número real, podemos dizer que:
|y| = y, se y = 0
|y| = -y, se y  0

Para y = 0:

2.y^2 + 3.y - 2 = 0
discriminante = 9 + 16 = 25
y = (-3 - 5) / 4 = y = -2 (não satisfaz a condição y = 0)
ou
y = (-3 + 5) / 4 = y = 1/2 (satisfaz a condição y = 0)

Para y  0:
---
-2.y^2 - 3.y - 2 = 0 = 2.y^2 + 3.y + 2 = 0
discriminante = 9 - 16 = -7 (a equação não tem raízes reais)


Portanto, concluímos que um único valor de y satisfaz a equação 2.y.|y| +
3.y = 2: y = 1/2

Como y = sen(x), teremos:
sen(x) = 1/2 = x = +-pi/6 + 2k.pi, com k inteiro.

Resposta: S = {x pertencente a R | x = +-pi/6 + 2k.pi, com k inteiro}


Observações:

O conjunto solução:
S = {x pertencente a R | x = +-pi/6 + 2k.pi, com k inteiro}
é equivalente ao conjunto solução dado pelo seu livro:
V = {x pertencente a R | x = pi/6+2kpi ou x = 5pi/6+2kpi, com k inteiro}

Na realidade, a solução y = -2 não pode ser levada em consideração porque
não satisfaz a condição colocada inicialmente, ou seja, y = 0.

Mesmo que y = -2 fosse uma solução satisfatória, nós teríamos que:
sen(x) = -2, o que é impossível no campo dos reais.
Seja a função f: R - R, tal que f(x) = sen(x), nós teremos que o conjunto
imagem é dado por: Im(f) = [-1, 1], como você havia citado. Ou seja,
qualquer que seja o x real, sempre teremos -1 = sen(x) = 1. Portanto,
todas as soluções que estão fora deste intervalo devem ser desconsideradas,
como você observou no livro.

A sua conclusão colocada abaixo é correta.
sen(x) = -2 como,
sen(x) = -2.sen(pi/2)
Porém, esta outra forma de escrever não muda em nada a questão da
impossibilidade de se encontrar um x real tal que sen(x)=-2.sen(pi/2)=-2.

Apenas por curiosidade, a equação sen(x) = -2 tem solução no campo dos
números complexos (C). Porém, a resolução em C seria diferente desta
apresentada, uma vez que no conjunto dos números complexos não há relação de
ordem. Portanto, não teria sentido considerar y = 0 ou y  0 com y
complexo.


From: [EMAIL PROTECTED] [mailto:[EMAIL PROTECTED] On
Behalf Of Johann Peter Gustav Lejeune Dirichlet
Sent: sexta-feira, 16 de abril de 2004 13:36
To: [EMAIL PROTECTED]
Subject: Re: [obm-l] Re:_[obm-l]_Equação_Trigonométrica!

Se voce definir seno em complexos fica facil.Acho que e^it=cos t+ i*sen t

Rafael [EMAIL PROTECTED] wrote: 
Carlos,

Se sen(x)  0, então 2 sen^2(x) + 3 sen(x) - 2 = 0

D = 3^2 - 4*2*(-2) = 9 + 16 = 25
sen(x) = (-3 +- 5)/4 == sen(x) = -2 ou sen(x) = 1/2

Como sen(x)  0, então sen(x) = 1/2.

Logo, x = Pi/6 + 2*k*Pi ou x = 5Pi/6 + 2*k*Pi,
sendo k inteiro.

Se sen(x)  0, então: - 2 sen^2 + 3 sen(x) - 2 = 0

D = 3^2 - 4*(-2)*(-2) = 9 - 16 = -7

Por D  0, sabemos que as raízes dessa equação são valores para os quais
sen(x) é complexo não-real. Veja:

sen(x) = [-3 +- sqrt(7)*i] / (-4) ==
== sen(x) = [3 + sqrt(7)*i]/4 ou sen(x) = [3 - sqrt(7)*i]/4


Encontrar os valores de x que satisfazem a essas equações imagino que não
seja fácil, mas o exercício pede que você resolva em R. Assim, o conjunto
solução é aquele mesmo que você mencionou.


Abraços,

Rafael



---! -- Original Message -
From: Carlos Alberto
To: [EMAIL PROTECTED]
Sent: Friday, April 16, 2004 9:28 AM
Subject: [obm-l] Equação Trigonométrica!


Resolva em R, a seguinte equação.

2 . senx . |senx| + 3 . senx = 2

Desculpa a pertinência em enviar questão que foge do escopo da lista.
Mas não tenho muitos locais para recorrer.
Segue abaixo minha resolução que eu não considerei tanto correta.

Resolução.

|senx|  0 ou
|senx|  0

logo, para

|senx|  0
-2 sen^2 x + 3 sen x - 2 = 0

Considerando sen x = t ( * )

-2 t^2 + 3t - 2 = 0

9 - 16 = - 7 --- Não possui raízes reais, logo não convém.

|senx|  0

2 sen^2 x + 3 sen x - 2 = 0
Considerando sen x = t

2 t^2 + 3 t - 2 = 0
t' = - 2 (**)
t = 1/2 (***)

Substituindo (*) em (**) e (***) temos,

senx = 1/2
senx = sen pi/6

x = pi/6 + 2kpi ou
x = 5pi/6 + 2kpi

Bom... a! té aqui tudo bem!!!

A Solução do livro é:
V = { x pert R | x = pi/6 + 2kpi ou x = 5 pi/6 + 2kpi}

O que ocorre com o sen x = -2??

Reparei no livro que nºs  1 e nºs  1 são aparentemente desconsiderados.

O pq disso? Eu imaginei sendo que a imagem de sen x = [-1, 1].

Mas não sei é realmente isso que ocorre.

Pois por outro lado eu enxergaria
sen x = -2 como,
sen x = -2 . sen pi/2

Alguém poderia esclarecer minha dúvida, e conferir se eu fiz algo errado na
resolução.

Desde já agradeço a atenção!!!

[ ], s Carlos


=
Instruções para entrar na lista, 

[obm-l] RE: [obm-l] RE: [obm-l] Re:_[obm-l]_Equação_Trigonométrica!

2004-04-17 Por tôpico Rogério Moraes de Carvalho
Olá Carlos,

Conforme o Rafael atentou numa mensagem privada, eu cometi um erro
primário.

Na realidade, a solução:
S = {x pertencente a R | x = +-pi/6 + 2k.pi, com k inteiro}
NÃO é equivalente ao conjunto solução dado pelo seu livro:
V = {x pertencente a R | x = pi/6+2kpi ou x = 5pi/6+2kpi, com k inteiro}

Pois -pi/6 é congruente ao arco 2pi - pi/6 = 7pi/6 e não 5pi/6 = pi - pi/6.

Eu realmente imaginei a equação como cos(x) = 1/2 ao invés de sen(x) = 1/2,
como o Rafael comentou.

Substitua:
x = +-pi/6 + 2k.pi, com k inteiro
por: 
x = pi/6+2kpi ou x = 5pi/6+2kpi, com k inteiro
na minha solução.

Rafael, muito obrigado,

Rogério Moraes de Carvalho
[EMAIL PROTECTED]


-Original Message-
From: [EMAIL PROTECTED] [mailto:[EMAIL PROTECTED] On
Behalf Of Rogério Moraes de Carvalho
Sent: sábado, 17 de abril de 2004 04:50
To: [EMAIL PROTECTED]
Subject: [obm-l] RE: [obm-l] Re:_[obm-l]_Equação_Trigonométrica!

Carlos,

A seguir, eu apresento a minha resolução para o problema.
Analisando-a você poderá encontrar alguns pequenos problemas na sua
resolução. Observe que as duas resoluções são bem similares.


Questão:

Resolva em R, a seguinte equação.
2.sen(x).|sen(x)| + 3.sen(x) = 2

**
Resolução:
**
Considerando y = sen(x), teremos:
2.y.|y| + 3.y = 2 = 2.y.|y| + 3.y - 2 = 0

Pela definição de módulo de um número real, podemos dizer que:
|y| = y, se y = 0
|y| = -y, se y  0

Para y = 0:

2.y^2 + 3.y - 2 = 0
discriminante = 9 + 16 = 25
y = (-3 - 5) / 4 = y = -2 (não satisfaz a condição y = 0)
ou
y = (-3 + 5) / 4 = y = 1/2 (satisfaz a condição y = 0)

Para y  0:
---
-2.y^2 - 3.y - 2 = 0 = 2.y^2 + 3.y + 2 = 0
discriminante = 9 - 16 = -7 (a equação não tem raízes reais)


Portanto, concluímos que um único valor de y satisfaz a equação 2.y.|y| +
3.y = 2: y = 1/2

Como y = sen(x), teremos:
sen(x) = 1/2 = x = +-pi/6 + 2k.pi, com k inteiro.

Resposta: S = {x pertencente a R | x = +-pi/6 + 2k.pi, com k inteiro}


Observações:

O conjunto solução:
S = {x pertencente a R | x = +-pi/6 + 2k.pi, com k inteiro}
é equivalente ao conjunto solução dado pelo seu livro:
V = {x pertencente a R | x = pi/6+2kpi ou x = 5pi/6+2kpi, com k inteiro}

Na realidade, a solução y = -2 não pode ser levada em consideração porque
não satisfaz a condição colocada inicialmente, ou seja, y = 0.

Mesmo que y = -2 fosse uma solução satisfatória, nós teríamos que:
sen(x) = -2, o que é impossível no campo dos reais.
Seja a função f: R - R, tal que f(x) = sen(x), nós teremos que o conjunto
imagem é dado por: Im(f) = [-1, 1], como você havia citado. Ou seja,
qualquer que seja o x real, sempre teremos -1 = sen(x) = 1. Portanto,
todas as soluções que estão fora deste intervalo devem ser desconsideradas,
como você observou no livro.

A sua conclusão colocada abaixo é correta.
sen(x) = -2 como,
sen(x) = -2.sen(pi/2)
Porém, esta outra forma de escrever não muda em nada a questão da
impossibilidade de se encontrar um x real tal que sen(x)=-2.sen(pi/2)=-2.

Apenas por curiosidade, a equação sen(x) = -2 tem solução no campo dos
números complexos (C). Porém, a resolução em C seria diferente desta
apresentada, uma vez que no conjunto dos números complexos não há relação de
ordem. Portanto, não teria sentido considerar y = 0 ou y  0 com y
complexo.


From: [EMAIL PROTECTED] [mailto:[EMAIL PROTECTED] On
Behalf Of Johann Peter Gustav Lejeune Dirichlet
Sent: sexta-feira, 16 de abril de 2004 13:36
To: [EMAIL PROTECTED]
Subject: Re: [obm-l] Re:_[obm-l]_Equação_Trigonométrica!

Se voce definir seno em complexos fica facil.Acho que e^it=cos t+ i*sen t

Rafael [EMAIL PROTECTED] wrote: 
Carlos,

Se sen(x)  0, então 2 sen^2(x) + 3 sen(x) - 2 = 0

D = 3^2 - 4*2*(-2) = 9 + 16 = 25
sen(x) = (-3 +- 5)/4 == sen(x) = -2 ou sen(x) = 1/2

Como sen(x)  0, então sen(x) = 1/2.

Logo, x = Pi/6 + 2*k*Pi ou x = 5Pi/6 + 2*k*Pi,
sendo k inteiro.

Se sen(x)  0, então: - 2 sen^2 + 3 sen(x) - 2 = 0

D = 3^2 - 4*(-2)*(-2) = 9 - 16 = -7

Por D  0, sabemos que as raízes dessa equação são valores para os quais
sen(x) é complexo não-real. Veja:

sen(x) = [-3 +- sqrt(7)*i] / (-4) ==
== sen(x) = [3 + sqrt(7)*i]/4 ou sen(x) = [3 - sqrt(7)*i]/4


Encontrar os valores de x que satisfazem a essas equações imagino que não
seja fácil, mas o exercício pede que você resolva em R. Assim, o conjunto
solução é aquele mesmo que você mencionou.


Abraços,

Rafael



---! -- Original Message -
From: Carlos Alberto
To: [EMAIL PROTECTED]
Sent: Friday, April 16, 2004 9:28 AM
Subject: [obm-l] Equação Trigonométrica!


Resolva em R, a seguinte equação.

2 . senx . |senx| + 3 . senx = 2

Desculpa a pertinência em enviar questão que foge do escopo da lista.
Mas não tenho muitos locais para recorrer.
Segue abaixo minha resolução que eu não considerei tanto correta.

Resolução.

|senx|  0 ou
|senx|  0

logo, para

|senx|  0
-2 sen^2 x + 3 sen x - 2 = 0

Considerando sen x = t ( * )

-2 t^2

RE: [obm-l] Problemas com radicais - CORRIGINDO!!

2004-04-16 Por tôpico Rogério Moraes de Carvalho
Olá Rafael,

Pode ficar tranqüilo, pois eu não fiquei ofendido em momento algum.
Quando eu disse que não havia entendido o objetivo dos seus comentários, foi
exatamente por eles não terem fundamento. Porém, eu estou certo de que você
teve a melhor das intenções, pois o objetivo de um grupo de discussões é a
ajuda mútua. Se você achou que eu havia cometido um erro, então procedeu
corretamente.

Peço desculpas se fui rude em meus comentários, mas a intenção é que
ficasse claro para os participantes do grupo que eu havia procurado ser o
mais rigoroso e preciso possível nas minhas considerações. Ao atingirmos um
determinado nível em Matemática, eu acho muito importante que nós
questionemos tudo aquilo que lemos, afinal de contas há muitos erros em
livros de Matemática.

Para mim está tudo esclarecido e espero que este mal entendido não
atrapalhe as nossas discussões sobre Matemática. Pode se sentir a vontade
para comentar os meus e-mails que eu estarei sendo mais cordial daqui para
frente.

Abraços,

Rogério.
-Original Message-
From: [EMAIL PROTECTED] [mailto:[EMAIL PROTECTED] On
Behalf Of Rafael
Sent: sexta-feira, 16 de abril de 2004 04:01
To: OBM-L
Subject: Re: [obm-l] Problemas com radicais - CORRIGINDO!!

Caro Rogério,

Eu não consegui entender o que você não entendeu: qual seria o objetivo de
um comentário a não ser emitir uma opinião que pode ou não ter algum
fundamento? Não me queira mal, por favor. Você nem sequer precisaria ter
mutilado o meu minúsculo comentário para comentá-lo...
Não, você não escreveu que 'a' e 'b' deveriam ser distintos e, em momento
algum, disse que o havia feito. Salientiei, e espero que você tenha
compreendido, que o trecho escrito por você estava entre aspas.
Sim, você me deu um contra-exemplo sobre o qual eu não havia pensado e que
eu encaminharei para o autor do livro que escreveu esses absurdos.

Tudo esclarecido? Espero que sim.


Obrigado,

Rafael de A. Sampaio




- Original Message -
From: Rogério Moraes de Carvalho [EMAIL PROTECTED]
To: [EMAIL PROTECTED]
Sent: Thursday, April 15, 2004 12:36 PM
Subject: RE: [obm-l] Problemas com radicais - CORRIGINDO!!


Ola Rafael,

Eu realmente nao consegui entender o objetivo dos seus comentarios.
De qualquer modo, eu estou comentando-os parte por parte.


Seu comentario:
---
Se a e b são racionais distintos, então a^2 é racional e a^2 - b é
racional..

Meu comentario:
---
No texto que eu escrevi, eu nao afirmei em momento algum que a e b devem ser
distintos, mas simplesmente racionais. A sua conclusao sobre a^2 e a^2 - b
serem racionais e' obvia, mesmo que a e b sejam racionais iguais. Isto e'
consequencia da propriedade de fechamento das operacoes de adicao e
multiplicacao do conjunto dos numeros racionais.



Seu comentario:
---
Ora, se a^2 - b for racional, transformar-se-á sqrt[a +- sqrt(b)] numa soma
ou diferença de radicais duplos, pois sqrt(a^2 - b) será
irracional.

Meu comentario:
---
Esta sua conclusao nao tem o menor embasamento teorico. De qualquer modo,
segue um contra-exemplo bem simples que comprova que a sua conclusao e'
falsa. Suponha a = 5/2 (racional) e b = 4 (racional), entao teremos a^2 - b
= (5/2)^2 - 4 = 25/4 - 4 = 9/4. Sendo assim, sqrt(a^2 - b) = sqrt(9/4) =
3/2, que e' racional. Portanto, a sua conclusao de que sqrt(a^2 - b) será
irracional esta' errada.



Seu comentario:
---
Dessa forma, sqrt(a^2 - b) deve ser um número inteiro
não-negativo, ou ainda, natural. Por isso: a, b, sqrt(a^2 - b) são
*naturais*, com [a +- sqrt(b)] real positivo.

Meu comentario:
---
Esta conclusao tambem nao tem o menor embasamento teorico. A reducao de
radicais duplos em radicais simples nao exige que a, b e sqrt(a^2 - b) sejam
naturais. Vamos a um exemplo de reducao de radicais duplos em radicais
simples em que a, b e sqrt(a^2 - b) sao racionais nao inteiros.
No radical duplo sqrt(5/3 + sqrt(7/3)), temos a = 5/3, b = 7/3 e sqrt(a^2 -
b) = sqrt[(5/3)^2 - 7/3] = sqrt(25/9 - 7/3) = sqrt[(25 - 21)/9] = sqrt(4/9)
= 2/3.
Sendo assim, podemos converter o radical duplo para radical simples, como
segue:
sqrt(5/3 + sqrt(7/3)) = sqrt[(5/3 + 2/3) / 2] + sqrt[(5/3 - 2/3) / 2]
sqrt(5/3 + sqrt(7/3)) = sqrt(7/6) + sqrt(1/2)


De qualquer modo, eu agradeco pela sua atencao.

Abracos,

Rogério Moraes de Carvalho



=
Instruções para entrar na lista, sair da lista e usar a lista em
http://www.mat.puc-rio.br/~nicolau/olimp/obm-l.html
=



=
Instruções para entrar na lista, sair da lista e usar a lista em
http://www.mat.puc-rio.br/~nicolau/olimp/obm-l.html
=


RE: [obm-l] Problemas com radicais - CORRIGINDO!!

2004-04-15 Por tôpico Rogério Moraes de Carvalho
Ola Rafael,

Eu realmente nao consegui entender o objetivo dos seus comentarios.
De qualquer modo, eu estou comentando-os parte por parte.


Seu comentario:
---
Se a e b são racionais distintos, então a^2 é racional e a^2 - b é
racional..

Meu comentario:
---
No texto que eu escrevi, eu nao afirmei em momento algum que a e b devem ser
distintos, mas simplesmente racionais. A sua conclusao sobre a^2 e a^2 - b
serem racionais e' obvia, mesmo que a e b sejam racionais iguais. Isto e'
consequencia da propriedade de fechamento das operacoes de adicao e
multiplicacao do conjunto dos numeros racionais.



Seu comentario:
---
Ora, se a^2 - b for racional, transformar-se-á sqrt[a +- sqrt(b)] numa soma
ou diferença de radicais duplos, pois sqrt(a^2 - b) será
irracional.

Meu comentario:
---
Esta sua conclusao nao tem o menor embasamento teorico. De qualquer modo,
segue um contra-exemplo bem simples que comprova que a sua conclusao e'
falsa. Suponha a = 5/2 (racional) e b = 4 (racional), entao teremos a^2 - b
= (5/2)^2 - 4 = 25/4 - 4 = 9/4. Sendo assim, sqrt(a^2 - b) = sqrt(9/4) =
3/2, que e' racional. Portanto, a sua conclusao de que sqrt(a^2 - b) será
irracional esta' errada.



Seu comentario:
---
Dessa forma, sqrt(a^2 - b) deve ser um número inteiro
não-negativo, ou ainda, natural. Por isso: a, b, sqrt(a^2 - b) são
*naturais*, com [a +- sqrt(b)] real positivo.

Meu comentario:
---
Esta conclusao tambem nao tem o menor embasamento teorico. A reducao de
radicais duplos em radicais simples nao exige que a, b e sqrt(a^2 - b) sejam
naturais. Vamos a um exemplo de reducao de radicais duplos em radicais
simples em que a, b e sqrt(a^2 - b) sao racionais nao inteiros.
No radical duplo sqrt(5/3 + sqrt(7/3)), temos a = 5/3, b = 7/3 e sqrt(a^2 -
b) = sqrt[(5/3)^2 - 7/3] = sqrt(25/9 - 7/3) = sqrt[(25 - 21)/9] = sqrt(4/9)
= 2/3.
Sendo assim, podemos converter o radical duplo para radical simples, como
segue:
sqrt(5/3 + sqrt(7/3)) = sqrt[(5/3 + 2/3) / 2] + sqrt[(5/3 - 2/3) / 2]
sqrt(5/3 + sqrt(7/3)) = sqrt(7/6) + sqrt(1/2)


De qualquer modo, eu agradeco pela sua atencao.

Abracos,

Rogério Moraes de Carvalho
-Original Message-
From: [EMAIL PROTECTED] [mailto:[EMAIL PROTECTED] On
Behalf Of Rafael
Sent: quinta-feira, 15 de abril de 2004 10:52
To: OBM-L
Subject: Re: [obm-l] Problemas com radicais - CORRIGINDO!!

Rogério,

Farei apenas um comentário sobre as condições de redução dos radicais duplos
a radicais simples. Você escreveu: Dada a expressão com radicais duplos v(a
+ vb), com a e b racionais, vb irracional e a + vb positivo, (...)

Se a e b são racionais distintos, então a^2 é racional e a^2 - b é
racional. Ora, se a^2 - b for racional, transformar-se-á sqrt[a +- sqrt(b)]
numa soma ou diferença de radicais duplos, pois sqrt(a^2 - b) será
irracional. Dessa forma, sqrt(a^2 - b) deve ser um número inteiro
não-negativo, ou ainda, natural. Por isso: a, b, sqrt(a^2 - b) são
*naturais*, com [a +- sqrt(b)] real positivo.


Abraços,

Rafael de A. Sampaio





- Original Message -
From: Rogério Moraes de Carvalho [EMAIL PROTECTED]
To: [EMAIL PROTECTED]
Sent: Thursday, April 15, 2004 8:05 AM
Subject: RE: [obm-l] Problemas com radicais - CORRIGINDO!!


Olá Daniel,

Muitos dos problemas que envolvem expressões com radicais duplos podem ser
resolvidos facilmente quando são realizadas as reduções para expressões com
radicais simples equivalentes. Existe uma fórmula para a redução, mas o
importante é entender como deduzi-la, pois o raciocínio é muito simples.

Redução de radicais duplos em radicais simples equivalentes
---
Dada a expressão com radicais duplos v(a + vb), com a e b racionais, vb
irracional e a + vb positivo, queremos encontrar x1 e x2 racionais positivos
tais que: v(a + vb) = vx1 + vx2.

Observe que de acordo com as condições dadas, ambos os membros da igualdade
são positivos. Portanto, a fim de eliminar o radical duplo do primeiro
membro da igualdade, podemos elevar ambos os membros ao quadrado garantindo
que a volta continua válida.
[v(a + vb)]² = (vx1 + vx2)²
a + vb = x1 + 2vx1vx2 + x2
a + vb = (x1 + x2) + v(4.x1.x2)

Sendo a, b, x1 e x2 racionais e vb irracional, a igualdade somente vai ser
verdadeira se tivermos:
x1 + x2 = a
4.x1.x2 = b = x1.x2 = b/4

Portanto, x1 e x2 são raízes da seguinte equação quadrática:
x² - (x1 + x2)x + x1.x2 = 0 = x² - ax + b/4 = 0

Calculando o discriminante, encontramos:
? = (-a)² - 4.1.(b/4) = ? = a² - b

Sendo assim, a nossa expressão somente poderá ser reduzida a radicais
simples se o discriminante (a² - b) for um quadrado de um racional. Se esta
condição for satisfeita, teremos:
x1 = [-(-a) + v(a² - b)] / 2 = [a + v(a² - b)] / 2
x2 = [-(-a) - v(a² - b)] / 2 = [a - v(a² - b)] / 2
Ou vice-versa.

Conclusão:
A expressão com radicais duplos v(a + vb), com a e b racionais, vb
irracional e a + vb positivo, pode ser transformada em uma